MCQs Faculty of General Medicine VI year

1. Mitral valve according to it’s B. Indentation of the middle third structure also is called: of the by an enlarged A. Bicuspid left atrium B. Tricuspid C. Notching of the ribs C. Aortic D. Bounding, full D. Pulmonary E. pectoris E. No correct answer 5. Which arrhythmia develops most 2. How many cusps does the right oftenly in patient with mitral atrioventricular valve have? ? A. 2 A. Atrial fibrillation B. 3 B. Atrioventricular block C. 4 C. Supraventricular D. 5 D. Ventricular fibrillation E. 6 E. Wolff-Parkinson-White syndrome 3. Mitral stenosis in 99% of all cases is caused by: 6. Choose ECG signs typical for atrial A. Infective fibrillation: 1. Negative T wave; 2. B. Congenital mitral stenosis Absence of P wave; 3. S-T C. Myxoma elevation; 4. f waves; 5. Different D. Rheumatic heart disease R-R distance. E. Mitral annual calcification A. 1, 2, 3 B. 2, 3 4 4. After suffering a streptococcal throat C. 2, 4, 5 infection, a 12-year-old boy D. 1, 3, 5 develops cardiac symptoms that E. All are correct are attributed to rheumatic . Years later, at the age of 34 he is 7. Choose chest x-ray findings typical admitted to the hospital with for mitral stenosis: 1. Convexity pulmonary edema. Further from enlarged left atrial examination reveals a diastolic appendage 2. Straightening of left murmur at the apex and mitral heart border 3. Left ventricular stenosis is diagnosed. Before hypertrophy 4.Large aorta 5. surgical evaluation, which of the Small aorta following findings can be A. 1, 2, 3 attributed to mitral stenosis? B. 2, 3, 4 A. Large left ventricle C. All are correct D. 1, 2, 4 E. 1, 2, 5 2 B. Percutaneous aortic balloon 8. Clinical of valvuloplasty suffers from a mitral stenosis develops with the relatively high complication reduction of the area of mitral and recurrence rate valve orifice less then: C. Percutaneous aortic balloon A. 2 cm 2 valvuloplasty has a relatively B. 3 cm 2 low complication and C. 4 cm 2 recurrence rate D. 5 cm 2 D. Open mitral commissurotomy E. 7 cm 2 can be successfully performed if there is limited calcification, 9. Which instrumental examination leaflet stiffness, chordal fusion is a method of choice to make E. Commissurotomy carries up to diagnosis and specify valvular a 20% chance of reoperation heart disease? within 5 years and a 60% A. ECG chance at 10 years B. Holter monitoring C. Doppler ultrasound test 12. Which signs are typical for patients D. Coronarography with aortic stenosis? Choose E. Cardiac catheterization correct combination: 1. Dyspnea; 2. Orthopnea; 3. Angina pectoris; 10. Indications for surgical treatment of 4. Syncope; 5. Congestive heart patients with mitral stenosis are: 1. failure. Critical mitral stenosis (mitral A. 1, 2, 3 valve orifice area less then 1 cm 2); B. 2, 3, 4 2. Symptomatic patients; 3. C. 1, 2, 4 Systemic ; 4. Mitral D. 3, 4, 5 valve gradient 12-15 mmHg; 5. E. All are correct End-diastolic gradient 8-10 mmHg. 13. On “Austin Flint A. 1, 2, 3 murmur” is associated with: B. 2, 3, 4 A. Mitral stenosis C. 1, 2, 5 B. Aortic regurgitation D. 2, 3, 5 C. Aortic stenosis E. All are correct D. Mitral regurgitation E. Tricuspid regurgitation 11. Which statement is wrong concerning surgical treatment of 14. A 63-year-old woman fell while patients with mitral stenosis? crossing the street after her A. Percutaneous balloon Thursday afternoon bridge game. valvuloplasty is a new Attempts at resuscitation for cardiac technique used for minimally arrest by the emergency medical invasive treatment of patients service (EMS) team were with mitral stenosis; unsuccessful. The woman had previously been diagnosed as 3 having aortic stenosis and left infancy. His atrial rate on ECG is ventricular hypertrophy. In addition 450 bpm, and his ventricular rate is to these factors, which of the 160 bpm. His pulse rate is 88 bpm. following predisposes to sudden The left atrium is enlarged. Similar cardiac death? findings were noted 1 year ago, but A. Split first heart sound he declined to take any . B. Hypokalemia Treatment should entail which of C. Soft murmur at left of sternum the following? that varies with inspiration A. Continue with colonoscopy D. Failure of the central venous B. Continue with colonoscopy pressure (CVP) to rise more after administration of than 1 cm H2O with 30-second parenteral antibiotics pressure on the liver C. Immediate administration of (hepatojugular reflux) antibiotics and follow-up E. CVP of -1 cm H2O colonoscopy at a later date D. Immediate administration of 15. While lying on the examining table anticoagulation and digoxin before colonoscopy, a 68-year-old and follow-up colonoscopy at a electrician notes . The later date colonoscopy was scheduled as a E. Immediate electrocardioversion routine procedure following with a current of 300–400 J removal of a benign 1 year earlier. He had in 17. After experiencing progressive infancy. His atrial rate on ECG is chest for 2 months, a surgical- 450 bpm, and his ventricular rate is supply store owner undergoes a CT 160 bpm. His pulse rate is 88 bpm. scan that reveals a space-occupying The left atrium is enlarged. Similar lesion of the wall of the left atrium, findings were noted 1 year ago, but which was confirmed to be he declined to take any medication. myxoma. There is no evidence of Make the diagnosis? disease elsewhere. What would the A. Mitral stenosis, atrial fibrilation next line of treatment be? B. Mitral stenosis, AV block A. Excision of a myxoma C. Mitral regurgitation, performed with a bypass paroxysmal tachycardia procedure D. Tricuspid regurgitation, atrial B. Excision of a myxoma fibrilation performed without a bypass E. Aortic stenosis procedure C. Insertion of a pacemaker 16. While lying on the examining table D. Chemotherapy before colonoscopy, a 68-year-old E. Radiotherapy electrician notes palpitations. The colonoscopy was scheduled as a 18. Following a car accident, a 52-year- routine procedure following old lawyer complains of pain in the removal of a benign polyp 1 year left and back. After arrival earlier. He had rheumatic fever in of the EMS team, her pulse rate is 4 84 bpm, but of small volume. She patients with valvular heart states that she has some cardiac diseases: 1. Valvular calcification, condition but is uncertain of its 2. Cardiac chamber enlargement, 3. nature. Which is the most likely Low ejection fraction, 4. Diastolic cause of the small pulse volume? filling disorder, 5. Pulmonary A. Aortic regurgitation congestion. B. Mitral regurgitation A. 1, 2, 5 C. Aortic stenosis B. 1, 3, 4 D. Tricuspid regurgitation C. 1, 4, 5 E. Hyperthyroidism D. 2, 3, 5 E. 2, 3, 4 19. On auscultation “Opening snap sound” is associated with: 23. Echocardiography allow real-time A. Mitral stenosis assessment of all, EXCEPT: B. Aortic regurgitation A. Chamber size C. Aortic stenosis B. Wall thickness D. Mitral regurgitation C. Valve appearance E. Tricuspid regurgitation D. Valve motion E. Extrasystole presence 20. Choose ECG signs typical for mitral stenosis: 1. Atrial fibrillation; 24. Cardiac catheterization is used to 2. Left atrium enlargement; 3. Right measure (choose correct answer): atrium enlargement; 4. Right A. Valve structure ventricle hypertrophy. B. Valve motion A. 1, 2, 3 C. Valve gradients B. 2, 3, 4 D. Valve vegetations C. 1, 2, 4 E. Chamber size D. 1, 2 E. All are correct 25. Afterload reduction with vasodilator agents is applied to 21. The mechanisms that compensate enhance pressure gradients in hemodynamics derangements patients with valvular heart disease. associated with valvular heart Choose the right combination of disease include all, EXCEPT: disorders, in which afterload A. Atrial chamber enlargement reduction can improve circulation. B. Ventricular chamber 1. Mitral regurgitation, 2. Aortic enlargement insufficiency, 3. Aortic stenosis, 4. C. Myocardial hypertrophy Mitral stenosis. D. Increased adrenergic A. 1,2 stimulation B. 1,3 E. C. 1,4 D. 2,3 22. The routine posteroanterior and E. 2,4 lateral chest roentgenogram may provide nonspecific information in 5 26. Operation for aortic stenosis is indicated for all symptoms, EXCEPT: 30. In deciding the tactics of treatment A. LV hypertrophy of patients with severe limb B. Congestive heart failure ischemia of unknown origin, the C. Calculated valve area less optimal combination of the than 0.8 cm2 following instrumental methods of D. Angina pectoris investigation are: 1. E. Syncope Sphygmography 2. Aorto- arteriography 3. Thermography; 4. 27. Mitral stenosis can cause all Ultrasound dopplerography 5. following conditions, EXCEPT: Occlusive plethysmography. Select A. Atrial fibrillation the correct combination: B. Pulmonary А. 1,2,3; C. Right ventricular dilation В. 2,3; D. LV hypertrophy С. 4,5; E. Hemoptysis D. 2,4; Е. 1,3,5. 28. For acute occlusion of main of the extremities are typical: 1. 31. In case of femoral embolism, Paleness; 2. Pain; 3. Edema; 4. ischemia III-B degree (last stage – Paresthesia; 5. No pulse. Choose the limb contractures) the method of correct configuration of answers: choice of treatment is: А. 1,2,4,5; А. Emergency embolectomy; B. 4,5; В. Thrombolysis; C. 2,3; С. Anticoagulant therapy; D. 1,2,3; D. Symptomatic therapy; E. All correct. Е. Primary amputation of limb.

29. In the absence, in patients with 32. Cause of brachial artery embolism embolism of femoral artery, of a may be any disease, except: heart disease as the cause of А. Mitral stenosis; embolism should be suspected: 1. В. MI; Aortic 2. Iliac artery С. Heart aneurism; aneurysm, 3. Aorta atheromatosis D. Abdominal aorta aneurism; with wall surface clot 4. Е. Compression of subclavian of superior mesenteric artery with additional cervical rib. 5. Additional cervical rib, which compresses the subclavian 33. What changes of coagulation are artery. Select the correct typical for patients with acute combination of answers: ischemia of the lower extremity due А. 1,2,5; to arterial embolism: 1. В. 3,4,5; Hypercoagulation 2. С. 1,2,3; Hypocoagulation 3. Inhibition of D. 4,5; fibrinolysis, 4. Platelets aggregation E. All correct. 6 5. Activation of fibrinolysys. Select varicosities. On examination, a the correct combination of answers: is heard over the right groin. А. 1,4; What is the most likely diagnosis? В. 2,3,5; A. Femoral artery thrombosis С 1,3,4; B. Superficial venous D. 2,3; insufficiency Е. 3,4. C. Arteriovenous (AV) D. 34. The best instrument for E. Deep vein insufficiency embolectomy from big arteries is: А. Vessel ring; 38. A 60-year-old man with a history of В. Vacuum aspirator; atrial fibrillation is found to have a С. Fogarty catheter; cyanotic, cold right lower extremity. D. Dormia catheter; The embolus is most probably Е. Kelly forceps. originating from which of the following? 35. In the topical diagnosis of acute A. An atherosclerotic plaque arterial occlusion the most B. An abdominal informative method of C. Heart examination is: D. Lungs А. Sphygmography; E. Paradoxical embolus В. Thermography; С. Aorto-arteriography; 39. A 60-year-old man with a history of D. Ultrasound dopplerography; atrial fibrillation is found to have a Е. Plethysmography. cyanotic, cold right lower extremity. Which is the most 36. A 63-year-old man has had a common site at which an arterial cyanotic painful left foot fourth toe embolus lodges? for 2 days. The dorsalis pedis and A. Aortic bifurcation posterior tibial arteries are palpable B. Dorsalis pedis artery on both sides. There is no history of C. Tibial arteries cardiac or . What is D. Popliteal artery the most likely diagnosis? E. Iliac artery A. Cardiac embolus B Atheroembolism 40. A 60-year-old man with a history of C Lupus atrial fibrillation is found to have a D Digital cyanotic, cold right lower E Raynaud’s syndrome extremity. What is the most appropriate management? 37. A 45-year-old woman undergoes A. Embolectomy cardiac catheterization through a B. Lumbar sympathectomy right femoral approach. Two C. Bypass months later, she complains of right D. Amputation lower extremity swelling and notes E. Arteriography the appearance of multiple 7 41. A young patient sustains blunt D. paresthesias trauma to his right knee that results E. Muscle contracture in acute thrombosis of his popliteal artery. Which tissue is most 45. Which clinical sign suggests sensitive to ischemia? necrosis of the muscles in patients A. Muscle with acute extremity ischemia? B. Nerve A. Pain C. Skin B. Pallor D. C. Edema E. Bone D. Paralysis E. Muscle contracture 42. A young college student injures his left knee while playing football and 46. Which location IS NOT usual is unable to bear weight. The source for emboli in case of acute provisional x-ray report indicates extremity ischemia? that there are no fractures seen. He A. Mitral valve is discharged home but presents the B. Left atrium next morning to the emergency C. Aorta department with a severely swollen, D. Left ventricle painful left knee and severe pain in E. Right ventricle the foot. On examination, the foot is pale, cold, and pulseless. What is 47. Thrombosis may occur in the the most likely diagnosis? following settings: 1) A. Traumatic atherosclerosis, 2) B. Gastrocnemius muscle tear hypercoagulable states, 3) C. Traumatic hyperhydration states, 4) vascular D. Posterior knee dislocation with grafts, 5) systemic anticoagulants thrombosed popliteal artery use. E. Traumatic sciatic neuropathy A. 1,3,5 B. 1,2,4 43. Name the earliest sign in case of C. 3,4,5 acute extremity ischemia caused D. 1,2,5 by thromboembolism? E. 2,3,4 A. Pain B. Pallor 48. Which metabolic derangements are C. Edema related to reperfusion in patient D. Paralysis with acute lower-extremity E. Muscle contracture ischemia after revascularization? 1) acidosis, 2) hyperkalemia, 3) 44. Symptoms of acute arterial alkalosis, 4) hypernatriemia, 5) occlusion are all except: myoglobinuria. A. Muscle pain A. 1,2,5 B. Skin pallor B. 2,3,5, C. Increased pulse on peripheral C. 1,4,5 arteries D. 2,3,5 8 E. 1,3,4 increase collateral perfusion, 5) decrease severity of the pain. 49. The embolic occlusion should be A. 1,2 suspected in patients with all B. 2,5 following features, EXCEPT: C. 1,4 A. acute onset of the disease D. 2,3 B. prior history of embolism E. 1,4 C. atrial fibrillation D. history of intermittent 53. Which statement concerning acute limb ischemia is wrong? E. MI A. Thrombosis of peripheral arteries is most oftenly 50. In case of acute limb ischemia so- associated with atherosclerotic called “5 Ps” have been used as a plaque mnemonic to remember the B. The extent of collateral flow clinical presentation of the across the site of occlusion disease. What are they? determines the severity of A. palpable cord, painlessness, symptoms. pallor, pulselessness, paralysis C. Patients with long-standing B. paresthesia, painlessness, atherosclerotic lesions very pallor, pulselessness, palpabile quickly develop dramatic cord symptoms C. paresthesia, pain, palpable D. Emboli originate from the cord, pulse paradoxus, paralysis heart in more than 90% of cases D. palpable cord, pain, pallor, and normally lodge at the site of pulse paradoxus, painlessness an arterial bifurcation E. paresthesia, pain, pallor, E. In case of thrombosis pulselessness, paralysis symptoms may be less dramatic than embolic occlusion 51. What is the “gold standard” in evaluation of patient with acute 54. Which of the following is NOT a limb ischemia? classic risk factor for PE? A. Aorto-arteriography A. Recent surgery B. Doppler ultrasonography B. C. CT C. Oral contraceptives D. MRI D. Hypertension E. Coagulogram E. Malignancy

52. The goal of systemic 55. Predominant source of PE is: anticoagulation, in case of A. Deep lower extremity embolization, are: 1) decrease the B. Left atrium risk of thrombus propagation, 2) C. Portal vein prevent recurrent embolization, 3) D. Upper extremity veins resolve thrombotic formation, 4) E. Mitral valve

9 56. Rare sources of PE are: 1. Portal vein; 2. Right atrium; 3. Left 60. Which statement is wrong atrium; 4. Upper extremity veins; 5. concerning test for D-dimer? Superficial varicose lower extremity A. D-dimer is fibrin degradation veins. product A. 1, 3, 4 B. Positive D-dimer test confirms B. 1, 4, 5 with high probability the C. 2, 4, 5 diagnosis of PE D. 2, 3, 4 C. D-dimer can be falsely elevated E. 3, 4, 5 postoperatively and in the setting of , ARDS, MI 57. Pathogeneticaly for PE is typical: D. Negative D-dimer with 98% A. Decreas of central venous probability exclude the diagnosis pressure of PE in patients with low PE B. Increas of arterial probability C. E. D-dimer is indicative of any D. thrombosis E. Increas of central venous pressure 61. Which statement is wrong concerning the V/Q scan? 58. What are the most common signs A. Radiolabelled marker is injected and symptoms of PE? IV A. Back pain, , and a B. Patient inhale aerosolized pulsatile abdominal mass radiolabelled marker B. Dyspnea, pleuritic , C. For PE is typical – perfusion and defect and normal ventilation C. Productive cough, wheezing, and scan bilateral pedal edema D. For pneumonia is typical – D. Chest pain, syncope, arterial perfusion and ventilation defect hypertension E. For PE is typical – normal E. Arterial hypertension, pain in the perfusion scan and ventilation thigh defect

59. Choose the combination of tests, 62. Which test is a gold standard in which are performed in case of diagnosing PE? suspicion for PE to make A. D-dimer preliminary diagnosis (baseline B. Angiopulmonography tests): 1. Chest x-ray; 2. Leg C. Leg doppler doppler; 3. ECG; 4. Arterial blood D. CT-pulmonary angiorgaphy gases; 5. D-dimer. E. Echocardiography A. 1, 2, 3 B. 2, 3, 4 63. 72 year-old patient, with obesity C. 3, 4, 5 and superficial , was D. 1, 3, 4 admitted to the surgical department E. 2, 3, 5 with strangulated large incisional 10 . On the next day after 60 mm Hg, heart rate – 130 per operation patient was complaining minute. Choose the first step in the for pain in the left thigh. On 3 day treatment of this patient? after operation during attempt to A. Antibiotics + stabilization of stand up appeared: inspiratory hemodynamic dyspnea, dull chest pain, systolic BP B. Thrombolysis + stabilization of 60 mm Hg, heart rate – 130 per hemodynamic minute. Make the diagnosis? C. Heparin + stabilization of A. Massive PE hemodynamic B. Acute D. Warfarine + stabilization of C. Minor PE hemodynamic D. Septic E. LMWH + stabilization of E. Pneumonia hemodynamic

64. 72 year-old patient, with obesity 66. 72 year-old patient, with obesity and superficial varicose veins, was and superficial varicose veins, was admitted to the surgical department admitted to the surgical department with strangulated large incisional with strangulated large . On the next day after hernia. On the next day after operation patient was complaining operation patient was complaining for pain in the left thigh. On 3 day for pain in the left thigh. On 3 day after operation during attempt to after operation during attempt to stand up appeared: inspiratory stand up appeared: inspiratory dyspnea, dull chest pain, systolic BP dyspnea, dull chest pain, systolic BP 60 mm Hg, heart rate – 130 per 60 mm Hg, heart rate – 130 per minute. Which diagnostic test minute. Which method of treatment should be performed to confirm the would be most effective in this diagnosis? case? A. Chest x-ray A. Medical treatment B. V/Q scan B. Surgical embolectomy C. Leg doppler C. Thrombolysis D. Transesophageal D. Therapy of pulmonary edema echocardiography E. Heparin E. D-dimers 67. Which of the following 65. 72 year-old patient, with obesity management options may be used in and superficial varicose veins, was the treatment of a PE? admitted to the surgical department A. Anticoagulation with strangulated large incisional B. Thrombolysis hernia. On the next day after C. Inferior vena cava (IVC) filter operation patient was complaining D. Surgical or catheter for pain in the left thigh. On 3 day embolectomy after operation during attempt to E. All of the above stand up appeared: inspiratory dyspnea, dull chest pain, systolic BP 11 68. Choose the drug which is used for B. Acute myocardial infarction pathogenetical treatment of PE: C. Major (submassive) PE A. t-PA D. Minor PE B. Heparin E. Pneumonia C. Pentoxyphyllin D. Warfarin 71. 28 years old man who is in an E. Aspirin army, presented with sudden onset of and localised 69. A 30-year-old male is admitted to pleuritic chest pain. He also has the hospital after a motorcycle non-productive cough, with an accident that resulted in a fracture of episode of haemoptysis, low grade the right femur. The fracture is fever without chills and rigors and managed with traction. Three days rashes. He had previous history of later the patient becomes confused trauma to the knee and anterior and tachypneic. A petechial rash is cruciate ligament repair done 1 noted over the chest. Lungs are months ago. No history of clear to auscultation. Arterial blood pneumonia, recent air travel or long gases show PO 2 of 50, PCO 2 of 28, journey drive. No other medical and pH of 7.49. The most likely illness. Acoording to examination diagnosis is? 20% occlusion of pulmonary A. Unilateral pulmonary edema vasculature was find. Which B. Hematoma of the chest diagnostic test is most suitable to C. Fat embolism confirm the diagnosis? D. Pulmonary thromboembolism A. CT-pulmonary angiogram E. Early Staphylococcus aureus B. ECG pneumonia C. Chest x-ray D. Leg doppler 70. 28 years old man who is in an E. D-dimers army, presented with sudden onset of shortness of breath and localised 72. 28 years old man who is in an pleuritic chest pain. He also has army, presented with sudden onset non-productive cough, with an of shortness of breath and localised episode of haemoptysis, low grade pleuritic chest pain. He also has fever without chills and rigors and non-productive cough, with an rashes. He had previous history of episode of haemoptysis, low grade trauma to the knee and anterior fever without chills and rigors and cruciate ligament repair done 1 rashes. He had previous history of months ago. No history of trauma to the knee and anterior pneumonia, recent air travel or long cruciate ligament repair done 1 journey drive. No other medical months ago. No history of illness. Acoording to examination pneumonia, recent air travel or long 20% occlusion of pulmonary journey drive. No other medical vasculature was find. Make the illness. Acoording to examination diagnosis? 20% occlusion of pulmonary A. Massive PE vasculature was find. Which method 12 of treatment would be most A. 1,3,5 effective in this case? B. 2,4,6 A. Antibioticotherapy C. 1,4,5 B. Surgical embolectomy D. 1,4,6 C. Thrombolysis E. 2,3,6 D. Therapy of pulmonary edema E. Heparin + Warfarine 76. Virchow’s triad, consists of three components, which predispose a 73. Which of the following are patient to thrombosis. Name them: contraindications for thrombolysis? 1. Hypoventilation; 2. Endothelial 1. Significant trauma (< 2 months); injury; 3. Stasis of blood flow; 4. 2. Recent major surgery (< 2 Hyperthrombolysis; 5. Blood months); 3. Prolonged cardio- hypercoagulability pulmonary resuscitation ( > 10 A. 1,3,5 minutes); 4. Recent puncture of a B. 1,4,5 non-compressible vessel (< 10 C. 2,3,5 days); 5. Bleeding diathesis. D. 1,2,4 A. All of them E. 2,3,4 B. 1, 2, 3 C. 2, 3, 4 77. Which statement, concerning D. 3, 4, 5 , is NOT E. 1, 2, 5 TRUE? A. Embolism is acute if it occludes 74. The "classic" clinical presentation a vessel of pulmonary embolism includes B. An embolism is chronic if it is all, EXCEPT: eccentric and contiguous with A. abrupt onset the vessel wall B. acute pleuritic chest pain C. A pulmonary embolism is called C. shortness of breath massive when it involves both D. hypoxia pulmonary arteries E. dehydration D. Routine laboratory findings are nonspecific 75. Pulmonary embolism can cause E. US dopplerography remains the such pathological processes: 1. gold standard examination for Reduces the cross-sectional area of the diagnosis of pulmonary the pulmonary vasculature; 2. embolism Increases the cross-sectional area of the pulmonary vasculature; 3. 78. For massive pulmonary embolism Reduces pulmonary vascular is typical systolic arterial pressure resistance; 4. Increases pulmonary less then vascular resistance; 5. Increases the A. less than 90 mm Hg right ventricular afterload; 6. B. less than 100 mm Hg Reduces the right ventricular C. less than 110 mm Hg afterload. Choose the correct D. more then 120 mm Hg combination. E. more then 200 mm Hg 13 84. Massive pulmonary embolism is 79. What is NOT a risk factor for caused by occlusion of pulmonary pulmonary embolism: vasculature: A. Immobilization A. 10-30% B. Surgery and trauma B. < 10% C. Pregnancy C. < 30% D. Bone fracture D. 30-50% E. Alcohol abusing E. > 50%

80. The onset of pulmonary embolism 85. All problems that compromise may include all symptoms, blood flow listed below can cause acute EXCEPT: visceral ischemia, except? A. Syncope A. Acute embolic occlusion B. Arterial hypertention B. Acute thrombotic occlusion C. Productive cough C. Nonocclusive mesenteric D. Wheezing ischemia E. Decreasing level of consciousness D. Splanchnic artery aneurysm E. Mesenteric veins thrombosis 81. Patients with massive pulmonary embolism may develop all symptoms, 86. Which clinical sings are typical for EXCEPT early stages of acute visceral A. Arterial hypotension ischemia? B. Tachycardia A. Severe abdominal pain, C. Polyuria vomiting, , leukocytosis D. Pleuritic chest pain B. Mild abdominal pain, E. Tachypnea C. Pulsating abdominal mass 82. Minor pulmonary embolism is D. No typical clinical signs caused by occlusion of pulmonary E. Bloody stool, signs of vasculature: A. 50-70% B. 70-90% 87. Name later manifestations of acute C. < 30% visceral ischemia? D. 30-50% A. Severe abdominal pain, E. >90% vomiting, diarrhea, leukocytosis B. Mild abdominal pain, 83. Major pulmonary embolism is constipation caused by occlusion of pulmonary C. Pulsating abdominal mass vasculature: D. No typical clinical signs A. 50-70% E. Bloody stool, signs of B. 70-90% peritonitis C. < 30% D. 30-50% 88. Which examination is considered to E. > 50% be a gold standard for diagnosis of acute visceral ischemia? 14 A. Ultrasound B. Arteriosclerotic plaque B. CT C. Mesenteric artery aneurysm C. Selective mesenteric D. Embolus angiography E. Vasospasm D. Duplex ultrasonography E. MRI 93. A 66-year-old woman is admitted for hyperalimentation due to 89. Which examination is considered to malnutrition consequent to massive be the best for screening of acute small-bowel resection. What is the visceral ischemia caused by thrombotic most likely condition that leads to ischemia or ? the need to perform a massive A. Ultrasound resection? B. CT A. Autoimmune disease C. Selective mesenteric B. Mesenteric ischemia angiography C. Mesenteric adenitis D. Duplex ultrasonography D. E. MRI E. Pseudomyxoma peritonei

90. A 60-year-old man with a history of 94. Name the form of acute mesenteric atrial fibrillation is found to severe ischemia which has the highest mortality abdominal pain, vomiting, diarrhea, rate? WBC=23*10 9/l. The embolus is most A. Acute embolic occlusion probably originating from which of the B. Acute thrombotic occlusion following? C. Nonocclusive mesenteric A. An atherosclerotic plaque ischemia B. An abdominal aortic aneurysm D. Splanchnic artery aneurysm C. Heart E. Mesenteric vein thrombosis D. Lungs E. Paradoxical embolus 95. The intestine is viable in over 90% of patients if the duration of mesenteric 91. A 60-year-old man with a history of ischemia symptoms lasts: atrial fibrillation is found to severe A. 12 hours or less abdominal pain, vomiting, diarrhea, B. 24 hours or less WBC=23*10 9/l. What is the most C. 36 hours or less appropriate surgical treatment for this D. 48 hours or less patient? E. 72 hours or less A. Embolectomy B. Lumbar sympathectomy 96. Superior mesenteric artery C. Bypass surgery embolism can be caused by: 1. D. Intestine resection Aneurysm of the left ventricle after E. Heparinization myocardial infarction; 2. Atrial fibrillation; 3. Bacterial 92. Name the most often cause for endocarditis; 4. Right ventricle mesenteric thrombosis? hypertrophy; 5. Pulmonary artery A. Blunt abdominal trauma 15 stenosis. Choose correct What causes acute mesenteric combination: ischemia in this case? А. 1, 3, 4; A. Acute embolic occlusion of B. 1, 2, 3; superior mesenteric artery C. 2, 3, 4; B. Acute thrombotic occlusion D. 2, 3, 5; of superior mesenteric artery E. All are correct. C. Nonocclusive mesenteric ischemia 97. What can cause infarction of the D. intestine: 1. Superior mesenteric artery E. Mesenteric vein thrombosis embolism; 2. Superior and inferior mesenteric artery embolism; 3. 100. Which parts of the GI tract will be Superior mesenteric artery thrombosis; ischemic in case of thrombosis of the 4. Superior mesenteric vein orifice of superior mesenteric artery? thrombosis; 5. Prolonged spasm of K. and arteries. Choose the L. Stomach, duodenum and BEST combination. A. 1, 2, 3 M. Small intestine, cecum, B. 2, 3, 4 colon ascendance C. 3, 4, 5 N. Colon and D. All diseases can cause O. All parts of small and infarction intestine E. None of these diseases can cause intestine infarction 101. Which operations we can perform in case of embolic occlusion of 98. In a patient with superior superior mesenteric artery (choose the mesenteric artery embolism in stage of best combination): 1. Embolectomy; 2. (part of small intestine Embolectomy and resection of part of necrotised) should be performed the small intestine; 3. Embolectomy and following operation: left hemicolectomy; 4. Embolectomy F. Thrombectomy and right hemicolectomy; 5. Total G. Isolated embolectomy excision of ileum, and right H. Embolectomy and hemicolectomy. resection of necrotised P. 1, 2, 3 intestine Q. 2, 3, 4 I. Total colectomy R. 1, 4, 5 J. Periarterial sympathectomy S. 1, 2, 4 T. All operations can be 99. A 42-years-old patient, who suffers performed from mitral stenosis and atrial fibrillation, 6 hours ago appeared 102. Most often cause of acute severe abdominal pain, vomiting, mesenteric ischemia is: diarrhea. On examination: tenderness A. Embolisation to the in mesogastrium, negative Blumberg superior mesenteric artery sign. CBC: Leukocytes – 21*10 9/l. 16 B. Thrombosis of superior mesenteric artery 105. A 68-year-old man is admitted to C. Nonocclusive mesenteric the coronary care unit with an ischemia acute myocardial infarction. His D. Portal vein thrombosis postinfarction course is marked by E. Mesenteric vein thrombosis congestive heart failure and intermittent hypotension. On the 103. What can cause nonocclusive fourth hospital day, he develops mesenteric ischemia (NOMI)? 1. severe midabdominal pain. On Severe mesenteric vasoconstriction; 2. , blood Myocardial infarction; 3. Septic shock; pressure is 90/60 mm Hg and pulse 4. Hemorrhagic shock; 5. Cardiac is 110 beats/min and regular; the decompensation. abdomen is soft with mild U. 1, 2, 3 generalized tenderness and V. 2, 3, 5 distention. Bowel sounds are W. 2, 3, 4 hypoactive; stool hematest is X. None of these diseases positive. The next step in this cause NOMI patient’s management should be Y. All these diseases cause which of the following? NOMI A. Barium enema B. Upper gastrointestinal 104. In patients with acute mesenteric ischemia due to mesenteric C. Angiography embolism, which of the following D. Ultrasonography statements is correct? E. Celiotomy A. Most oftenly embolization to inferior mesenteric 106. Most oftenly chronic lower artery is observed extremities ischemia is caused by: B. Embolus most oftenly A. Buerger’s disease origins from right heart B. Atherosclerosis C. Thrombolytic therapy may C. Popliteal artery entrapment be attempted in patients D. Mucinous cystic degeneration without signs of bowel E. Fibrodysplasia infarction or gastrointestinal bleeding 107. All statements concerning D. Arteriography usually atherosclerosis are true EXCEPT: reveals the embolus lodged A. Atherosclerosis results in lipid at the orifice of the accumulation within the intimal superior mesenteric artery layer of blood vessels E. At the time of exploration B. Is the basis of most peripheral in case of superior vascular disease mesenteric artery C. Atherosclerosis affects most embolism, ischemia is oftenly young patients most severe in the left D. Is the major degenerative disease colon of arteries 17 E. Oftenly occurs as a result of the exercise-induced increase in aging process blood flow E. Is lower extremity muscular pain 108. Major risk factors for induced by exercise atherosclerosis are all EXCEPT: A. Cigarette smoking 112. Asymptomatic clinical stage of B. Hypertension chronic lower extremity ischemia C. Diabetes mellitus can be diagnosed by signs: 1. D. Hyperlipidemia Ulcers; 2. Muscles atrophy; 3. E. Alcohol consumption Hyperkeratosis; 4. Diminished hair and nail growth on affected limb 109. According to the traditional and digits; 5. Claudication. Choose Fontaine classification system for correct combination. lower extremity arterial occlusive A. 1, 2, 3 disease II stage means: B. 2, 3, 4 A. Asymptomatic C. 3, 4, 5 B. Claudication D. 1, 3, 5 C. Ischemic rest pain E. 2, 4, 5 D. Ischemic ulceration E. Ischemic necrosis 113. Critical limb ischemia is characterized by: 1. Rest pain; 2. 110. According to the traditional Fontaine stage II; 3. Pedal necrosis; Fontaine classification system for 4. Intermittent claudication; 5. lower extremity arterial occlusive Fontaine stages III and IV: disease III stage means: A. 1, 2, 3 A. Asymptomatic B. 1, 2, 4 B. Claudication C. 2, 3, 4 C. Ischemic rest pain D. 1, 3, 5 D. Ischemic ulceration E. 2, 3, 5 E. Ischemic necrosis 114. Which examination or test is used 111. Choose not correct statement as the most common tool to concerning intermittent diagnose peripheral vascular disease claudication: and stratify objectively the extent of A. Is clinically diagnosed as rest occlusive disease? pain A. Phlebography B. Is relieved with short periods of B. Duplex ultrasound rest C. Angiography C. Pain is located in the calves (less D. Echocardiography frequently in the buttocks or E. Ankle-brachial pressure index thighs) D. Is caused by arterial obstruction 115. Initial nonoperative treatment of proximal to affected muscle patients with peripheral artery beds, which limits the normal disease in the I-II stage according to Fontaine classification consists of 18 such components: 1. Adequate pain in both lower and upper antiplatelet therapy; 2. extremities, migratory superficial Anticoagulant therapy; 3. of the feet occurring a few Antibiotics; 4. Hypertension years ago. Physical examination control; 5. Treatment of findings are remarkable for absent hyperlipidemia. Choose best bilateral posterior tibial and dorsalis combination: pedis with palpable popliteal A. 1, 2, 3 pulses.What is the single most B. 1, 3, 4 important step in management? C. 1, 4, 5 A. Multiple toe amputations D. 2, 3, 4 B. Long-term anticoagulant therapy E. 3, 4, 5 C. Immediate operative intervention D. Angiography followed by bypass 116. Which antiplatelet agent is used surgery most oftenly? E. Cessation of smoking A. Aspirin B. Clopidogrel 119. A middle-aged man undergoes a C. Ticlopidine left below-knee amputation for left- D. Heparin foot gangrene secondary to arterial E. Pentoxyphyllin occlusive disease. Which of the following statements is true after the 117. A 27-year-old chronic smoker below-knee amputation? presents with ulceration of the tip A. There is less efficient function of the right second, third, and than after above-knee fourth toes. He gives a history of amputation recurrent pain in both lower and B. Stump prognosis can be judged upper extremities, migratory by transcutaneous oxygen superficial phlebitis of the feet monitoring occurring a few years ago. C. Poor prognosis is inevitable if Physical examination findings are Doppler fails to record a pulse at remarkable for absent bilateral that level posterior tibial and dorsalis pedis D. Is performed in patients with pulses with palpable popliteal occlusion of femoral artery pulses. Make the diagnosis? E. The level of transection is 5 cm A. Thromboangiitis obliterans above the medial malleolus (Buerger’s disease) B. Lupus vasculitis 120. A 70-year-old man with a long- C. Atheroembolism standing history of diabetes D. Raynaud’s syndrome develops gangrene of the right E. Atherosclerosis obliterans second toe. What is true of his diabetic foot? 118. A 27-year-old chronic smoker A. Dorsalis pedis and posterior presents with ulceration of the tip of tibial arteries pulses are always the right second, third, and fourth absent toes. He gives a history of recurrent 19 B. Gangrene of the toe always symptoms or signs of arterial requires urgent below-knee insufficiency? amputation A. Ischemic ulceration C. Arterial reconstruction is B. Ischemic neuropathy (not invariably required diabetic neuropathy) D. His right common femoral artery C. Claudication is most probably occluded or D. Nocturnal foot pain stenosed E. Toe gangrene E In pathogenesis of diabetic foot development big role play 124. Symptoms or signs of stenosis of lower extremity atherosclerotic occlusive disease of arteries and progression of the bifurcation of the abdominal peripheral neuropathy aorta (Leriche syndrome) include? A. Claudication of the buttock and 121. An elderly patient with ischemic thigh, impotence rest pain is found to have combined B. Causalgia of the lower leg aortoiliac and femoropopliteal C. Retrograde ejaculation occlusive disease. What is the D. Gangrene of the feet treatment of choice in this case? E. Dependent rubor of the feet A. Lumbar sympathectomy B. Femoropopliteal bypass 125. On which artery we measure pulse C. Aortofemoral and below the middle of inguinal femoropopliteal bypass ligament? D. Aortofemoral bypass A. A. iliaca interna E. Vasodilator therapy B. A. femoralis communis C. A. poplitea 122. A 72-year-old man complains of D. A. profunda femoris bilateral thigh and buttock E. A. iliaca externa claudication of several months duration. He was told by his 126. Choose all pathological processes physician that the angiogram which are responsible for the revealed findings indicating that he development of varicosae veins: 1. has Leriche syndrome. What does Venous valvular incompetence; 2. this patient have? Venous hypertension; 3. High A. Abdominal aortic aneurysm intraabdominal preassure; 4. B. Aortoiliac occlusive disease Endothelial function imparement; 5. C. Iliac artery aneurysm Smooth muscle function D. Femoropopliteal occlusive imparement. disease A. 1, 2, 3; E. Tibial occlusive disease B. 2, 3, 5; C. 1, 3, 5; 123. Conservative management rather D. 2, 4, 5; than reconstructive arterial surgery E. All are correct. is generally recommended for patients with which of the following 127. The greater saphenous vein arises: 20 A. Anterior to the medial malleolus; 132. Choose all etiological factors B. Posterior to the medial which can cause varicose veins: 1. malleolus; Pregnancy; 2. Chronic straining; 3. C. Anterior to the lateral malleolus; Prolonged standing; 4. Pelvic veins D. Posterior to the lateral malleolus; obstruction; 5. Obesity. E. Below the knee joint. A. 1, 2, 3; B. 2, 3, 5; 128. Most oftenly the greater C. 1, 3, 5; saphenous vein join: D. 2, 4, 5; A. External iliac vein; E. All are correct. B. Common femoral vein; C. Popliteal vein; 133. Choose symptoms which are D. Posterior tibial vein; typical for early stages of varicose E. Deep femoral vein. veins: 1. Ankle swelling; 2. Leg heaviness; 3. Cutaneous 129. The lesser saphenous vein arises: pigmentation; 4. Ulceration; 5. A. Anterior to the medial malleolus; Cosmetically displeasing dilated B. Posterior to the medial superficial veins. malleolus; A. 1, 2, 3; C. Anterior to the lateral malleolus; B. 2, 3, 4; D. Posterior to the lateral malleolus; C. 1, 2, 5; E. Below the knee joint. D. 2, 3, 4; E. All are correct. 130. Most oftenly the lesser saphenous vein join: 134. All statements about pain in A. External iliac vein; patients with varicose veins are true B. Common femoral vein; except: C. Popliteal vein; A. The most frequent type is limb D. Posterior tibial vein; heaviness; E. Anterior tibial vein. B. Ache that occurs after prolonged standing; 131. All statements about veins of C. The pain is usually felt over the lower extremities are true except: calf area; A. 90% of blood is returning to the D. Walking may increase the calf heart by deep vein system; ache associated with varicose B. The leg muscles act like a heart veins; for veins; E. Lying down, particularly with C. Backward floow of the blood in elevation of the limb, relieves veins is prevented by valves; limb heaviness within a short D. Blood flows from superficial to period of time. deep venous system by perforator veins; 135. Varicose veins are defined as E. system consists dilated palpable subcutaneous most oftenly from 3 veins. veins larger than: A. 4 mm; 21 B. 7 mm; B. 2,3; C. 10 mm; C. 3,4; D. 2 mm; D. 1, 2; E. 1 mm. E. All are correct.

136. Which statement is wrong 139. Name main principles of concerning venous imaging studies? nonsurgical treatment of not A. Duplex scanning is screening complicated varicose veins: 1. method for patients with Rest; 2. Exercises; 3. Leg varicose veins; elevation; 4. Elastic compression; B. For all patients with varicose 5. Lower leg. Choose best veins phlebography is indicated; combination: C. Phlebography and duplex A. 2, 3, 5; scanning provide detailed B. 1, 2, 5; anatomic information; C. 1, 3, 4; D. Phlebography is indicated in D. 2, 4, 5; complex cases (vein valve E. 1, 2, 4. transplantation or multiple re-do procedures); 140. A 21-year-old woman is referred E. Duplex scanning allows vein to your office because of multiple mapping, perforator mapping. lower extremity varicose veins. She has large varicosities in the 137. Venous disease of the legs can be distribution of the long saphenous classified according to the vein, she was never examined for severity, cause, site and specific varicose veins. What is the next step abnormality using the CEAP in management? classification. What C1 means A. A ligation and stripping according to this classification? operation; A. No visible or palpable signs of B. Ligation of both the long and venous disease; short saphenous system; B. Telangiectases, reticular veins, C. Sclerotherapy; malleolar flare; D. Duplex evaluation along with C. Varicose veins; clinical correlation as an essential D. Edema without skin changes; initial step; E. Skin changes ascribed to venous E. Compression stockings and disease anticoagulation therapy.

138. Specify most efficient and 141. A middle-age woman has right noninavasive examinations, which foot nonpitting edema, leg helps to identify the perforator heavyness. The diagnosis of incompetence: 1. Torniquet test. 2. chronic venous insufficiency is Duplex scanning. 3. Phlebography. made, stage C1. What is the 4. Arteriography. 5. March test. treatment of choice? Choose best combination: A. Vein stripping; А. 2,5; B. Pressure-gradient stockings; 22 C. Skin grafting; B. Sclerotherapy is not a D. Perforator vein ligation; reasonable treatment option E. Valvuloplasty. C. There is no association with deep venous insufficiency 142. A patient complains of an D. These veins will never bleed ulceration on the inner surface of E. Walking will likely improve her the lower third of the right shin. symptoms On xamination: the ulcer is round- shaped, up to 5 cm in diameter, 144. Development of chronic venous with sloping edges. On the inner insufficiency of lower extremities surface of this shin there are depends from the functional varicose veins. What complication condition of so-called "muscle appeared in this patient? pump". In relation to which muscle A. Elephantiasis group isused this term? B. Erysipelatous A. Buttocks C. Deep venous thrombosis B. Abdominal wall D. Varicose veins with trophic C. Foot ulceration D. Hip E. Popliteal artery thrombosis E. Shin

143. A 61-year-old woman comes to 145. Operation for varicose veins of the office complaining of ”spidery lower extremities allows to: 1. To veins”. Occasionally, she has a dull, overcome abnormal dump of blood achy feeling in her legs that usually from deep veins to superficial; 2. occurs at the end of the day. Remove varicose veins; 3. Restore Recently her shoes have begun to patency of deep veins; 4. Adjust feel tight at the end of the day. She femoral vein valve failure; 5. used to work as a bank teller, Remove trophicaly changed tissues. spending many hours on her feet Select best combination of answers: each day. She denies any shortness A. 3,4,5 of breath or difficulty walking up B. 1,2,4 stairs. There is no history of deep C. All are correct vein thrombosis. On physical D. 1,2,3 examination her legs are symmetric E. 2,3,4 in size, without evidence of trauma or skin breakdown. On the inner 146. Reticular veins are defined as aspect of her upper and lower thigh dilated nonpalpable subcutaneous are dilated superficial veins. There veins: are good dorsal pedalis pulses A. 1-4 mm; bilaterally, and motor and sensory B. 5-10 mm; examination is normal. The most C. 10-15 mm; accurate statement regarding her D. > 15 mm; condition is: E. < 1 mm. A. Compression stockings may provide relief 23 147. Teleangiectases are defined as Lung cancer; 3. Genitourinary dilated nonpalpable subcutaneous cancer; 4. CNS cancer; 5. veins: Gastrointestinal cancer. A. 1-4 mm; A. 2, 3, 4; B. 5-10 mm; B. 2, 3, 5; C. 10-15 mm; C. 1, 2, 3; D. > 15 mm; D. 3, 4, 5; E. < 1 mm. E. All are correct

148. Specify factors, which plays 152. In women, especially in young important role in the pathogenesis age, deep vein thrombosis can be of deep vein thrombosis: 1. Stasis; caused by: 1. Pregnancy; 2. 2. Hypocoagulability; 3. Abortion; 3. Oral contraceptives; 4. Hypercoagulability; 4. Vein wall Hormonal replacement therapt. (endothelial) injury; 5. Allergic A. All are correct; reaction. B. 1, 2, 3; A. 1, 2, 3; C. 2, 3, 4; B. 1, 3, 4; D. 1, 2, 4; C. 1, 2, 4; E. 1, 3, 4. D. 2, 3, 5; E. All are correct. 153. Name the signs of acute deep vein thrombosis: 1. Thigh pain; 2. 149. Where deep vein thrombosis most Massive leg swelling; 3. Pale colour oftenly begins: of the thigh skin; 4. Early leg A. Foot; paralysis; 5. Leg . B. Anlke; A. 1, 2, 3; C. Calf; B. 2, 3, 4; D. Knee; C. 3, 4, 5; E. Thigh D. 1, 2, 5; E. 2, 3, 5 150. Choose the risk factors for the development of deep vein 154. Homans’ sign – is: thrombosis: 1. Advanced age; 2. A. Thigh swelling; Immobilization; 3. Active cancer; 4. B. Leg cyanosis; Orthopedic procedures; 5. Primary C. Pain with passive dorsiflexion of hypercoagulable states. the foot; A. 1, 2, 3; D. Tenderness of the calf; B. 2, 3, 4; E. Prominent superficial veins in C. 1, 3, 5; case of deep vein thrombosis D. 2, 3, 5; E. All are correct. 155. Which statement concerning D- dimer are not true? 151. What are most suspicios A. D-dimers are products of the for the development of deep vein degradation of cross-linked thrombosis: 1. Thyroid cancer; 2. fibrin by plasmin; 24 B. D-dimer blood levels reflect the E. Assay of fibrin/fibrinogen presence of intravascular fibrin; products C. D-dimer is sensitive for the diagnosis of venous 158. Name the most dangerous thromboembolism complication of deep vein D. D-dimer measurement is most thrombosis: valuable as an adjunct to other A. Venous ulcers; diagnostic modalities (duplex B. Pulmonary embolism; scanning etc) C. Phlebitis; E. Increased level of D-dimer is D. Valvular incompetency; highly specific for deep vein E. Obliteration of lower extremity thrombosis deep veins.

156. Four days after undergoing 159. A middle-aged man known to subtotal gastrectomy for stomach have is admitted cancer, a 58-year-old woman with upper gastrointestinal bleeding. complains of right leg and thigh During his hospital stay, he pain, swelling and redness, and has develops DVT of the left lower tenderness on examination. The extremity. What is the most diagnosis of deep vein thrombosis is appropriate management? entertained. What is the initial A. Anticoagulants screening test to establish the B. Observation diagnosis? C. Thrombolytic therapy A. Venography D. Inferior vena cava (IVC) filter B. Venous duplex ultrasound E. Aspirin C. Impedance plethysmography D. Radio-labeled fibrinogen 160. An 18-year-old man develops a E. Assay of fibrin/fibrinogen painful, swollen left leg while products training for the Kiev Marathon. There is tenderness in the left calf 157. Four days after undergoing and ecchymosis is present. No signs subtotal gastrectomy for stomach of varicose veins on both legs, cancer, a 58-year-old woman active and passive movements in the complains of right leg and thigh left knee joint are not painfull. What pain, swelling and redness, and is the most likely diagnosis? has tenderness on examination. A. Cellulitis; The diagnosis of deep vein B. Deep vein thrombosis; thrombosis is entertained. What is C. Superficial ; the most efficient and sensitive D. Tear of the plantaris muscle; (gold standard) test to establish the E. Popliteal artery embolism diagnosis? A. Venography 161. Choose the best combination of B. Venous duplex ultrasound deep vein thrombosis prevention C. Impedance plethysmography methods in the patient of high risk D. Radio-labeled fibrinogen group: 1. Early activation of the 25 patient after surgical procedure; 2. patients; 5. Young age of the Low-molecular weight heparins; 3. patient with deep vein thrombosis. Graduated-compression stockings; A. 2, 3, 4; 4. Intermittent leg compression; 5. B. 1, 2, 3; Regional anesthesia (intraspinal or C. 3, 4, 5; epidural anesthesia). D. 1, 2, 5; A. 1, 2, 3; E. 1, 3, 5 B. 2, 3, 4; C. 3, 4, 5; 165. Prophylactic regimens of D. 1, 2, 4; documented benefit in decreasing E. All are correct the risk of POSToperative thromboembolism include: 162. Treatment of deep vein A. Early activation and ambulation thrombosis is directed towards all B. External pneumatic compression factors except: devices placed on the upper A. To prevent death from extremities pulmonary embolism; C. Long bedrest after operation B. To prevent recurrent deep D. Leg elevation for 24 h vein thrombosis; postoperatively C. To prevent development of E. Dipyridamole therapy for 48 h varicose veins; postoperatively D. To prevent the post- thrombotic syndrome; 166. Patients with phlebographically E. To prevent proximal confirmed deep vein thrombosis of propagation of the thrombosis the calf: A. Can expect asymptomatic 163. Which thrombolytic agent is used recovery if treated promptly with for the treatment of deep vein anticoagulants thrombosis? B. May be effectively treated with A. Aspirin; low-dose heparin B. Recombinant tissue C. May be effectively treated with plasminogen activator; pneumatic compression C. Heparin stockings D. Fondaparinux (Arixtra); D. May be effectively treated with E. Pentoxyphyllin acetylsalicylic acid E. Are at risk for significant 164. Specify idications for inferior pulmonary embolism vena caval interruption: 1. Contraindications for thromlytics in 167. Choose the indication for the low risk group patient; 2. placement of inferior vena cava Contraindications to anticoagulants; filter: 3. Recurrent pulmonary embolism A. thrombosis despite adequate anticoagulation; 4. B. Recurrent pulmonary embolus Prophylactic placement in high-risk despite adequate anticoagulation therapy 26 C. Pulmonary embolism in a patient pulmonary embolism. Choose with a hypercoagulable condition WRONG statement concerning D. Pulmonary embolus due to deep superficial vein thrombophlebitis: vein thrombosis of the lower A. Superficial vein extremity that occurs 2 weeks thrombophlebitis is a life- postoperatively threatening condition with high E. Pulmonary embolus in a patient risk of pulmonary embolism with metastatic pancreatic B. The most common source of carcinoma trauma associated with superficial vein thrombophlebitis 168. Choose ALL risk factors for is an intravenous cannula development of superficial vein C. Migratory thrombophlebitis may thrombophlebitis: 1. Varicose veins: be associated with occult 2. Age > 60; 3. Obesity; 4. Tobacco malignancy smoking; 5. History of deep vein D. Mondor’s disease is a thrombosis. thrombophlebitis of A. 1, 2, 3 of the B. 1, 3, 4 brest and chest wall C. 2, 3, 4 E. Lesser saphenous vein superficial D. 1, 3, 5 vein thrombophlebitis may progress E. All are risk factors for superficial into popliteal deep vein thrombosis vein thrombosis 171. Which condition is most 169. The physical diagnosis of prothrombotic amoung all inherited superficial thrombophlebitis is thrombophilias? based on the presence of: 1. A. Antithrombin III deficiency Erythema in distribution of the B. Protein C deficiency superficial veins; 2. Pale lower C. Protein S excess extremity; 3. Tenderness in the D. Antiphospholipid antibodies distribution of the superficial veins; E. Hypocysteinemia 4. Palpable and painfull cord in distribution of the superficial veins; 172. The factors, which propagate 5. Pulsless lower extremity. thrombosis during pregnancy are all A. 1, 3, 4 except: B. 1, 2, 3 A. Increased fibrinogen level C. 2, 3, 4 B. Decreased fibrinolytic activity D. 2, 3, 5 C. Reduction in protein S level E. All are clinical signs of D. Increased levels of factors VII, superficial thrombophlebitis VIII, IX, X E. Increased level of protein C 170. Superficial vein thrombophlebitis is a relatively common disorder 173.Which statement concerning with a significant incidence of thrombotic predisposing factors, is recurrence and has potential wrong? morbidity from extension and 27 A. Women have a slight predilection over men 177. The laboratory evaluation for B. No racial predilection factor-related hypercoagulability C. Age may be a predisposing conditions includes measurement of all factor in both SVT, DVT the following, except: D. Oral contracept intake A. Prothrombin time, activated predispose for venous partial thromboplastin time thrombosis B. Protein C and protein S level E. Decreased total platelet count C. Antithrombin level D. Homocysteine level 174. Which statement is not correct? E. D-dimers A. Superficial thrombophlebitis usually occurs over a previous 178. All statements concerning heparin varicose vein are true, except: B. Pain associated with SVT is A. Causes inhibition of thrombin usually localized over the site of B. Prevents formation and/or thrombosis. extension of thrombus C. Pain associated with DVT is C. Allows recanalization of the generally diffuse blood vessel over time D. Recent surgery, immobilization D. Decrease the level of protein C are factors that can contribute to E. Can be used IV SVT or DVT E. Bilateral extremity swelling is 179. Oral anticoagulants are used: (1) suspectious for superficial to prevent recurrent thrombotic events, thrombophlebitis (2) for long-term outpatient therapy in patients, (3) to inhibite vitamin K 175. The classic findings of SVT are a metabolism, (4) to treat acute firm(1), tender(2), erythematous thromboflebitis, (5) for resolution of fibrous cord(3), evidence of heart thrombi. failure(4), positive Homan’s sign(5): A. 1,2,3 A. 1,2,3 B. 1,2,5 B. 1,3,5 C. 2,3,5 C. 2,3,5 D. 3,4,5 D. 3,4,5 E. 1,4,5 E. 1,4,5 180. What medicines have no 176. The signs of DVT are painfull significant role for therapy of venous calf(1), swollen leg(2), erythematous thrombosis? fibrous cord(3), evidence of heart A. Antiplatelet agents failure(4), positive Homan’s sign(5): B. Protein C concentrate A. 1,2,3 C. Thrombolytic agents B. 1,2,5 D. Heparin C. 2,3,5 E. Oral anticoagulants (Coumadin) D. 3,4,5 E. 1,4,5 28 181. A 24-year-old woman on oral extremity. Objectively: moderate contraceptive pills develops an episode edema of shin, brown induration of of deep vein thrombosis that is skin in the lower third of shin, varix adequately treated with dilatation of superficial shin veins are anticoagulation. She is at increased risk present. What is the most probable of developing which of the following? diagnosis? A. Recurrent foot infections A. Parkes-Weber syndrome B. Claudication B. Lymphedema of lower right C. Pulmonary embolism extremity D. Postphlebitic syndrome C. Acute thrombophlebitis of E. Superficial varicose veins superficial veins D. Postthrombophlebitic syndrome, 182. A 37 y.o. patient complains of varicose form pain in the right arm which increases E. Acute thrombosis of deep veins during motion, raised body temperature up to 39 0C. In the right cubital fossa 185. Venous insufficiency in the deep there is a trace of injection, hyperemia or superficial system causes all and thickening along the vein. Your except: diagnosis? A. pain A. Inflammation of lymph B. swelling B. Phlegmon C. skin changes C. Phlebit D. ulcerations D. Erysipelas E. intermittent claudication E. Abscess 186. Which statement is wrong? 183. A 28 y.o. woman comes to the A. Patients with deep venous Emergency Room with a slightly system insufficiency nearly reddened, painful "knot", 8 cm above always are symptomatic; the medial malleolus. Examination in B. Pain caused by venous the standing position demonstrates a insufficiency often is improved distended vein above and below the by walking or by elevating the mass. There are no other abnormalities legs; on physical examination. The most C. Warmth tends to aggravate the likely diagnosis is: symptoms of venous A. Cellulitis insufficiency, and cold tends to B. Early deep vein thrombosis relieve them; C. Subcutaneous hematoma D. Compression stockings usually D. Insect bite ameliorate or prevent the pain in E. Superficial venous thrombosis patients with chronic arterial insufficiency; 184. A 43 year old patient had right- E. Leg aching, heaviness, and sided deep vein thrombosis of soreness are the most common iliofemoral segment 3 years ago. Now subjective symptoms of venous he is suffering from the sense of insufficiency. heaviness, edema of the lower right 29 187. The most common signs of E. D-dimmer venous system insufficiency are: Pitting edema(1), 191. Which functional tests can Hyperpigmentation(2), Paresthesia(3), physician provide to evaluate both Ulceration(4), “Cold extremity”(5). deep and superficial venous A. 1,2,4 system: (1) venous refilling time, B. 1,3,4 (2) the maximum venous outflow, C. 1,3,5 (3) calf muscle pump ejection D. 2,3,5 fraction, (4) peak arterial velocity, E. 2,4,5 (5) D-dimmers. A. 1,2,3 188. Skin ulcerations localized on B. 1,4,5 the lateral aspect of the ankle are C. 1,3,5 more likely to be related to: prior D. 2, 4,5 trauma (1), perforating veins valve E. 2,3,4 failure (2), pure venous insufficiency (3), basal cell 192. What kind of prolonged carcinoma (4), chronic arterial activities are not allowed for insufficiency (5). patients with symptoms of venous A. 1,2,4 insufficiency and intact B. 1,4,5 functioning calf muscle pump. C. 1,3,5 A. Walking D. 2,3,5 B. Running E. 2,3,4 C. Bicycling D. Swimming 189. Nonhealing ulcers on the E. Standing medial part of the ankle are most likely due to underlying: 193. Patients with muscle pump failure A. venous stasis are not allowed for such prolonged B. previous trauma activities like: C. arterial insufficiency A. Walking D. basal cell carcinoma B. Running E. allergic reactions C. Bicycling D. Standing 190. What is the most sensitive and E. Not allowed everything above specific test for the assessment of deep and superficial venous 194. Name the contraindication for disease in the lower extremity and compression stockings wearing: pelvis, areas not accessible with A. arterial insufficiency other modalities. B. deep venous insufficiency A. Magnetic resonance C. hypocoagulation states venography (MRV) D. bleeding varicosity in past B. Doppler ultrasound E. varicose veins C. Duplex ultrasound D. 30 195. What are the contraindications A. Endoscopy for skin graft placement in patients B. Contrast esophagogram with ulceration due to venous C. Manometry insufficiency: (1) Uncorrected leg D. 24-hour pH-monitoring venous hypertension, (2) E. Endoscopic US Untreated arteriovenous malformation, (3) Congenital 200. A 69-year-old man is admitted to varicose veins, (4) Purulent ulcer the emergency department with an bottom, (5) Abnormal protein C & acute UGI hemorrhage following a S levels. bout of repeated vomiting. A. 1,2,4 Fiberoptic gastoscopy reveals three B. 1,4,5 linear mucosal tears at the GE C. 1,3,5 junction. What is the diagnosis? D. 2,3,5 A. Reflux with E. 2,3,4 ulceration B. Barrett’s esophagus 196. The most frequent cause of C. Carcinoma of the esophagus descending necrotizing D. Mallory-Weiss tear mediastenitis is: E. Scleroderma A. Odontogenic infection B. Retropharyngeal abscesses 201. A 79-year-old retired opera singer C. Iatrogenic pharyngeal injuries presents with dysphagia, which has D. Cervical lymphadenitis become progressively worse during E. Parotitis the last 5 years. He states that he is sometimes aware of a lump on the 197. Name the organism which didn’t left side of his neck and that he cause descending necrotizing hears gurgling sounds during mediastenitis: swallowing. He sometimes A. Staphylococcus regurgitates during eating. B. β-hemolytic Streptococcus What is the likely diagnosis? C. Pseudomonas A. Carcinoma of the esophagus D. Bacteroides B. Foreign body in the esophagus E. E.coli C. Plummer-Vinson (Kelly- Patteson) syndrome 198. What type of surgery is D. Zenker’s (pharyngoesophageal) complicated most oftenly by diverticulum postoperative mediastenitis? E. Scleroderma A. Endocrine B. Pulmonary 202. A symptomatic patient has a C. Cardiac barium swallow that reveals a 3-cm D. Esophageal Zenker’s diverticulum. The next E. Vascular step in management is? A. H2 blockers 199. The first step diagnostic test for B. Anticholinergic drugs esophageal diseases is: C. Elemental diet 31 D. Bougienage E. Surgery (cricopharyngeal 206. A 54-year-old clerk complains of myotomy and diverticulectomy) having had dysphagia for 15 years. The clinical diagnosis of achalasia 203. The gold standart test for making is confirmed by a barium study. diagnosis achalasia is? What is TRUE in this condition? A. Endoscopy A. The most common symptom is B. Contrast esophagogram dysphagia C. Manometry B. The dysphagia is not typical D. 24-hour pH-monitoring C. The incidence of sarcoma is E. Endoscopic US increased D. Recurrent pulmonary infections 204. A 38-year-old man develops are rare increasing dysphagia for solid food E. Endoscopic dilatation should be over many months. What is the avoided most likely cause of his clinical presentation? 207. Name the most common A. Carcinoma of the esophagus condition/disease of esophagus: B. Achalasia A. Achalasia C. Sliding B. Diverticulum D. Paraesophageal hernia C. Foreign bodies E. Esophageal diverticulum D. Mallory-Weiss syndrome E. Tumors 205. A 53-year-old moderately obese woman presents with heartburn 208. The esophagus is a muscular tube aggravated mainly by eating and lined with nonkeratinizing lying down in the horizontal squamous epithelium that starts as a position. Her symptoms are continuation of the pharynx and suggestive of gastroesophangeal ends as the cardia of the stomach. reflux disease (GERD). Which of Which of the following is NOT the following statements is TRUE? TRUE concerning anatomy of A. It is best diagnosed by an esophagus? anteroposterior (AP) and lateral film A. Significant mobility is a normal of the chest esophageal characteristics and B. It may be alleviated by certain pathological states can easily drugs, especially theophylline, displace the esophagus; diazepam, and calcium channel B. The esophagus is fixed only at its blockers upper and lower ends, the upper C. It is not relieved by cessation of end being firmly attached to the smoking cricoid cartilage and the lower D. If it is associated with dysphagia, end to the diaphragm; it suggest a stricture or motility C. The esophagus passes through disorder three compartments: neck, E. It should be immediately treated and abdomen; with surgery 32 D. Ingested foreign bodies tend to A. 1,2,3 lodge at the narrowest area - B. 1,2,5 gastroesophageal sphincter; C. 1,3,4 E. The lower third of esophageus D. 2,4,5 contains voluntary muscles. E. 1,2,4

209. In the adult male length of 212. Contrast esophagogram do NOT esophagus is from 22 to 28 cm and reveal (choose one option): averages 2 cm shorter in the female A. Diverticula Which statement, concerning B. Narrowing or strictures esophageal size is wrong? C. Achalasia A. The thoracic part of the D. Diaphragmatic esophagus is approximately 20 E. Gastroesophageal reflux disease cm long; B. The abdominal portion of the 213. Esophageal manometry is esophagus is approximately 2 cm essential to confirm the diagnosis of in length; primary esophageal motility C. The anatomical length of the disorders: 1. Achalasia; 2. Diffuse esophagus is the distance from esophageal spasm; 3. After- the cricoid cartilage to the gastric burnings esophageal strictures; 4. orifice; Hypertensive lower esophageal D. The length of the esophagus, sphincter; 5. Gastroesophageal according to endoscopical reflux disease. measure, is the distance from the A. 1,2,3 incisors to the gastric inlet; B. 1,3,5 E. Esophageal length doesn’t varies C. 1,2,4 with individual height. D. 2,4,5 E. 3,4,5 210. The esophagus receives blood from: 1. Superior thyroidal artery, 2. 214. In cases of ACHALASIA, the Both left and right pulmonary arteries, majority of patients presenting between 3. Bronchial arteries, 4. Middle the ages: mediastinal artery, 5.Left gastric artery A. 20-40 years A. 1,2,3 B. 10-20 years B. 1,3,5 C. 40-60 years C. 2,3,4 D. More than 60 years D. 2,4,5 E. No age predisposition E. 1,4,5 215. A 36-year-old male patient, with a 211. Nowadays assessment of 2 years history of dysphagia, esophageal function and structure underwent barium swallow, which includes such examinations: 1. reveals typical bird’s-beak Contrast esophagogram; 2. deformity in the distal esophagus Endoscopy; 3. Manometry; 4. with more proximal esophageal Irrigography; 5. Aortography. 33 dilatation. What is the most likely epiphrenic diverticulum; 4. diagnosis? Underlying achalasia 5. Failed A. Achalasia esophageal peristalsis. B. Foreign body of the esophagus A. 1,2,3 C. Hiatal esophageal hernia B. 1,3,4 D. Diverticula C. 1,2,5 E. Distal esophagitis D. 2,4,5 E. 3,4,5 216. The goal of medical treatment of achalasia is to relax smooth 220. Esophageal manifestations of muscles. Which of the following GERD includes: 1. Heartburn; 2. drugs are useless in case of Chest pain; 3. Dysphagia; 4. Weight achalasia? loss, 5. Bronchospasm. A. Calcium channel blockers A. 1,2,3 B. Opioids B. 1,3,4 C. Nitrates C. 1,2,5 D. Anticholinergics D. 2,4,5 E. H2 blockers E. 3,4,5

217. According to location esophageal 221. Extraesophageal manifestations of diverticula are classified as: 1. GERD include all of the following Retropericardiac, 2. EXCEPT: Pharyngoesophageal, 3. A. Chronic cough Retrosternal, 4. Midesophageal, 5. B. Laryngitis Epiphrenic. Choose the correct C. Dental damage combination. D. Regurgitation of undigested food A. 1,2,3 E. Chronic sinusitis B. 1,3,5 C. 1,2,4 222. The most common cause of D. 2,4,5 esophageal injury or perforation is: E. 3,4,5 A. Instrumental procedures B. Penetrating chest trauma, 218. What symptom is NOT typical for especially height fall pharyngoesophageal diverticulum C. Chest tube placement for tension A. Dysphagia B. Regurgitation of undigested food D. Caustic poisoning C. Enlarged cervical lymph nodes E. Foreign body ingestion D. Frequent aspiration E. Voice changes 223. “Pseudoachalasia”, caused by a mediastinal tumor, may be 219. Gastroesophageal reflux (GERD) distinguished from primary is a mechanical disorder that is achalasia using? caused by: 1. Defective lower A. Esophageal manometry esophageal sphincter; 2. Gastric B. Esophagogram emptying disorder; 3. Underlying C. Upper GI endoscopy 34 D. Physical examination E. Auscultation during meal 227. Which of the following, concerning gastroesophageal reflux 224. Which statement, concerning (GER) is WRONG? esophageal diverticula, is WRONG? A. GER is an extremely common A. Midesophageal diverticula are condition among esophageal rare and most commonly pathology associated with mediastinal B. Lower esophageal sphincter granulomatous disease dysfunction is the most common (histoplasmosis or tuberculosis) cause of GER B. A midesophageal diverticulum is C. H2-receptor antagonists are more typically asymptomatic and effective than PPIs in case of GER diagnosed incidentally D. Medical therapy is the first line C. Epiphrenic diverticulum most of management for GER commonly associated with E. Extraesophageal manifestations esophageal motor abnormalities of GER are generally pulmonary (achalasia, hypertensive LES) D. Epiphrenic diverticulum 228. The initial step in therapy for typically occurs within the distal patients with mild and intermittent 10 cm of the esophagus symptoms of gastroesophageal E. Midesophageal diverticula are reflux (GER) is: common conditions A. Lifestyle modifications B. Nissen procedure 225. All following conditions may C. Exclude from meal cause descending necrotizing D. α-adreno blockers except: E. BoTox injection A. Retropharyngeal abscesses B. Acute peritonitis 229. Assessment of thyroid gland C. Iatrogenic pharyngeal injuries functional condition is based on the D. Infections of the mandibular level of: molars A. Thyroglobulin E. Cervical lymphadenitis B. Liver function tests C. Hormones of hypophysis-adrenal 226. Clinical findings suspicious for system descending necrotizing mediastinitis D. Parathyroid hormone include following: 1. recent stomach E. Thyroid-stimulating hormone operation; 2. fever, 3. localized (TSH), thyroxine (T4) and cervical or oropharyngeal pain, 4. triiodothyronine (T3) purulent sputum, 5. respiratory distress. 230. Extent of surgical intervention on A. 1,2,3 thyroid gland does NOT depend B. 1,3,4 from: C. 2,3,5 A. Thyroid tissue malignant D. 2,4,5 transformation E. 1,4,5 35 B. Hormones level of hypophysis- A. Thyreoidectomy thyroid system B. Subtotal thyreoidectomy C. Hormones level of hypophysis- C. Hemithyroidectomy with adrenal system pretracheal lymphatic node D. Character of process in thyroid biopsy gland D. Hemithyroidectomy E. localization in thyroid E. Lymphadenectomy gland 236. Choose the best method of 231. “Cold” nodule on thyroid nodular toxic goiter treatment: scinigram is typical for: A. Methimazole treatment A. Autoimmune thyroiditis B. Radioactive iodine treatment B. Grave’s disease C. Surgical treatment C. Toxic nodular goiter D. Methimazole + glucocorticoids D. Thyroid cancer E. Methimazole + β-adrenoblockers E. Thyrotoxic adenoma 237. Extent of operation on thyroid 232. Inferior thyroid arteries are gland in patients with multinodular branches of: goiter is: A. a. catotis communis A. Thyreoidectomy B. a. thyroidea communis B. Hemithyroidectomy C. aortic arch C. Subtotal thyreoidectomy D. tr.thyrocervicalis D. Enucleation of nodes E. a. subclavia E. Isthmectomy

233. Superior thyroid artery is the 238. Thyrocalcitonin is produced by: branch of: A. Thyroid A-cells A. a. catotis communis B. Thyroid B-cells B. aorta C. Thyroid C-cells C. a. subclavia D. Thyroid D-cells D. tr.thyrocervicalis E. Thyroid A, B-cells E. a. carotis externa 239. In 6 hours after 234. Effeciency of levothyroxine hemithyroidectomy the wound therapy after thyroidectomy is hematoma arised and is increasing. controlled by: What is your further action? A. T3 level A. Continue hemostatic therapy B. T4 level B. Surgical exploration, ligation of C. T4 excretion with urine the bleeding vessel and D. Thyrotropin-releasing hormone removing of hematoma concentration C. Cold compress on wound area E. Thyroid-stimulating hormone D. Antibiotic therapy E. Hot compress on neck area 235. Standard operation for the thyrotoxic adenoma is: 36 240. Preparing the patient with 245. Ultrasonographic sign of thyroid thyrotoxic adenoma for surgical is: intervention we do NOT use: A. Size of the nodule – 1 cm A. Thyrostatic drugs B. Fluid inside the nodule B. β-blockers C. Peripheral vascularization of the C. Radioactive iodine nodule D. Sedative drugs D. Unclear circuit of the nodule E. Steroid hormones E. Multiple nodules

241. Ultrasonographic sign of 246. On second day after malignant thyroid nodule is: hemithyroidectomy the deep wound A. Size of the nodule less then 1 cm hematoma arised, but does not B. Fluid in the central area of the increase in size. What is your nodule further action? C. Peripheral vascularization of the A. No further intervention nodule B. Surgical exploration, removing D. Unclear circuit of the nodule wound hematoma and ligation of E. Multiple nodules the bleeding vessel C. Cold compress on wound area, 242. Thyrotoxic adenoma generally antibiotic therapy, hemostatic produces: drugs A. Triiodothyronine D. Antibiotic therapy, aspirin B. Thyroxin E. Hot compress on neck area C. Thyrocalcitonin D. Thyrotropin-releasing hormone 247. In patient with suspicion for E. Thyroid-stimulating hormone postoperative hypothyroidism first of all we should check: 243. The best surgical procedure for A. Iodine excretion with urine Grave’s disease is: B. Radioiodine absorption A. Resection of thyroid gland C. T4 B. Hemithyroidectomy B. T3 C. Subtotal thyroid resection E. TSH D. Isthmectomy E. Total thyroidectomy 248. Levothyroxine therapy after thyroidectomy in patient with 244. Ultrasonographic sign of colloid multinodular goiter is called: nodule is: A. Suppressive A. Size of the nodule – 1 cm B. Replacing B. Fluid in the central area of the C. Supportive nodule D. Stimulating C. Peripheral vascularization of the E. Adjuvant nodule D. Unclear circuit of the nodule 249. For last three months young E. Multiple nodules woman lost 8 kg of weight. She complains of heartbeats, neck 37 thickening, sensation of foreign 252. 28-year-old male, has been object during the swallowing, operated because of thyrotoxic fingers tremor, exophthalmos, low goiter. In 12 hours after operation grade body fever. On palpation body temperature increased to 39°C, diffusely enlarged thyroid gland. Ps – 160/min., BP – 180/110 The most likely diagnosis is? mm.Hg. What complication has A. Grave’s disease been developed? B. Hysteria A. Hypoparathyroidism C. Brain tumour B. Severe arterial hypertension D. Thyrotoxic adenoma C. Laryngeal recurrent nerve injury E. Endemic goiter (diffuse) D. Thyrotoxic crisis E. Pneumonia 250. 42-year-old woman for the FIRST time was diagnosed severe 253. Patient M., 28-year-old male, has thyrotoxicosis caused by Grave’s been operated because of thyrotoxic disease. Choose the right sequence goiter. On 2 day after operation the of treatment? patient`s condition worsen: A. Urgent operation convulsions of hands, legs, face B. Medical therapy, then resection muscels, Chvostek and Trousseau of thyroid gland signs are positive. Patient complains C. Medical therapy, then of chest pain. ECG: prolonged QT endocrinologist supervision interval. What complication has D. Medical therapy been developed? E. Medical therapy, then perform A. Hyperparathyroidism subtotal resection of thyroid B. Hypoparathyroidism gland C. Thyrotoxic crisis D. Laryngeal recurrent nerve injury 251. 46-year-old patient, has been E. Thyrotoxic myocardiodystrophy operated 12 years ago for diffuse thyrotoxic goiter. Now patient was 254. A 26-year-old woman was diagnosed the recurency of diagnosed: diffuse thyrotoxic goiter. thyrotoxicosis. One more operation The conservative treatment under is offered. It is necessary to find out endocrinologist supervision was not where the thyroid tissue is located, effective during 12 month. She US examination was not effective. agreed for surgical intervention. What diagnostic method we must What measures of preoperative use? preparation should be performed for A. Fine needle aspiration biopsy of prevention of thyrotoxic crisis in the thyroid gland postoperative period? B. CT A. Use of antithyroid agents to C. Thyroid radioactive iodine achieve euthyroid condition scintigraphy B. Minimally invasive surgery D. X-ray examination of neck C. Bed rest E. Contrast esophagogram D. Detoxication therapy E. Application of a-blockers 38 D. Total thyroidectomy 255. Patient K., 29-year-old male, has E. NSAIDs been diagnosed: diffuse thyrotoxic goiter. The conservative treatment 258. What laboratory test confirms the under endocrinologist supervision diagnosis of Graves disease? for last 3 years. Recently the tight A. free T3 nodule appeared in the right lobe of B. TSH-receptor’s antibodies thyroid gland, which is increasing in C. Thyroglobulin antibodies size. What diagnostic method will D. TSH help to exclude thyroid cancer E. Thyroperoxidase antibodies preoperatively? A. US 259. Levothyroxine therapy after B. Intraoperative lymphatic node thyroidectomy in patient with biopsy cancer of thyroid gland is called: C. X-ray examination of the neck A. Suppressive D. Thyroid radioactive iodine B. Replacing scintigraphy C. Supportive E. Fine needle aspiration biopsy D. Stimulating E. Adjuvant 256. A 36-year-old patient complains of permanent hoarseness of voice 260. * A patient of 32 y.o. complains and symptoms of respiratory of severe weakness, tremor of insufficiency after physical extremities. Objective examination: exercices. These complaints body weight loss, wet and warm appeared 2 years ago, after total skin. The thyroid gland is enlarged thyroidectomy. Name the up to the 2-nd degree, painless, complication which develops in this elastic. Ps – 108 bpm, BP – 160/55 case: mm Hg. There are no other A. Larynx cancer abnormalties. The diagnosis is: B. Laryngeal recurrent nerve injury A. Chronic autoimmune thyroiditis, C. Chronic laryngitis hypertrophic type D. Laryngophthysis B. Toxiferous adenoma of the E. Scleroma of vocal cords thyroid gland C. Diffuse toxic goiter of the 2-nd 257. A 40-year-old woman presents degree, thyrotoxicosis of the with weight loss, palpitations, and average degree exopthalmos. On physical D. Diffuse euthyroid goiter of the 2- examination, the thyroid gland is nd degree diffusely enlarged. Blood tests E. Chronic fibrous thyroiditis reveal hyperthyroidism. Which medication is not used for the 261. * A 63 y.o. patient was operated treatment of thyrotoxicosis? on account of big multinodular A. Methimazole euthyroid goiter. Despite of techical B. Lugols iodine difficulties a forced subtotal C. I131 resection of both parts of the thyroid 39 gland was performed. On the 4-th A. human body reaction to own day after the operation the woman thyroid proteins had cramps of face muscles and B. human body reaction to foreign upper extremities, stomach ache. proteins which get to organism Positive Chvostek's and Trousseau's from environment signs. What is the most probable C. influence of light irradiation to cause of such condition? human body A. Insufficiency of parathyroid D. increased blood level of glands thyrostimulating hormone B. Thyrotoxic crisis E. increased blood level of C. Injury of recurrent nerve thyrotropic hormone D. Postoperative hypothyroidism E. Tracheomalacia 265. *A 50 year old woman with a 2- year history of mild, diffuse, tender 262. * Examination of a 26 year old thyroid enlargement complains of female patient revealed a node in 10 pound weight gain and fatigu E. the right lobe of thyroid gland. The What is the most probable node appeared no earlier than 3 diagnosis? months ago. The patient associates A. Riedel's thyroiditis this node with . She doesn't B. Subacute thyroiditis complain either about pain or C. Papillary thyroid carcinoma enlargement of the node. Ultrasonic D. Suppurative thyroiditis scanning revealed a 2x2,5 cm large E. Hashimoto's thyroiditis node in the inferior part of the right lobe of thyroid gland. What 266. *Examination of a 12 year old treatment should be administered? child revealed diffuse thyroid A. Dynamic observation enlargement of the II degree. Heart B. No need for treatment auscultation revealed dullness of C. Radioactive iodine , heart rate was 64/min. D. Conservative therapy The child has frequent E. Surgical intervention constipations, anemia. Concentration of thyreoglobulin 263. What arrythmia most oftenly antibodies is increased. What develops in patients with toxic disease might have caused such goiter? symptoms? A. Atrial fibrillation A. Thyroid carcinoma B. Ventricle fibrillation B. Autoimmune thyroiditis C. AV block C. Thyroid hyperplasia D. Extrasystole D. Endemic goiter E. Thyroid hormones doesn’t cause E. Diffuse toxic goiter any heart problems 267. Thyroiditis includes the following 264. What is the most probable reason inflammatory diseases of the of autoimmune thyroid thyroid gland: 1. Acute suppurative inflammation initiation? thyroiditis; 2. Subacute thyroiditis; 40 3. Chronic thyroiditis; 4. D. Dysphagia Hashimoto's thyroiditis; 5. Riedel E. With everything thyroiditis A. 1, 2, 3 272. What is the main complication of B. 1, 3, 4 chronic autoimmune thyroiditis? C. 1, 2, 4 A. Hypothyroidism D. 2, 3, 4 B. Hyperthyroidism E. All are correct C. Hypoparathyroidism D. Abscess formation 268. What medicine may cause E. Descending necrotizing secondary thyroiditis? mediastinitis A. Amiodarone B. Cefuroxime 273. Which statement, concerning C. Potassium iodine acute thyroiditis is NOT TRUE? D. Digoxin A. The usual microorganism E. All can cause responsible for acute thyroiditis is Staphylococcus aureus 269. The history of acute thyroiditis B. Patients with acute thyroiditis include all of the following, generally maintain normal EXCEPT: thyroid function A. Fever C. Radioactive iodine thyroid B. Neck pain scanning is useless in patients C. Hoarseness with acute thyroiditis D. Dysphagia D. Thyroid ultrasonography is E. Viral infection useful in patients with acute thyroiditis 270. The history of subacute thyroiditis E. In acute thyroiditis, if diagnosis include: 1. Symptoms of is confirmed on the stage of hyperthyroidism; 2. Symptoms of infiltration, immediate surgery is hypothyroidism; 3. Previous viral required infection; 4. Previous bacterial infection; 5. Low grade fever; 6. 274. Which statement, concerning High grade fever subacute thyroiditis, is NOT A. 1,3,5 TRUE? B. 1,3,6 A. Subacute thyroiditis is generally C. 2,4,6 thought to be due to viral D. 2,3,6 infection E. 1,4,5 B. C-reactive protein levels are usually elevated in subacute 271. Chronic autoimmune thyroiditis is thyroiditis associated with which of the C. Treatmant of subacute thyroiditis following? is symptomatic A. High fever D. The most common symptom, in B. Previous bacterial infection case of subacute thyroiditis , is C. Thyroid gland enlargement fever 41 E. Signs of hyperthyroidism may be 278. The main effects of parathyroid present hormone are to increase the concentration of plasma calcium by 275. Which statement, concerning (choose the WRONG answer): chronic autoimmune thyroiditis, is A. Increasing the release of NOT TRUE? calcium and phosphate from A. Chronic thyroiditis is associated bone matrix with leukocytosis with a shift to B. Increasing calcium reabsorption the left and an increased ESR by the kidney B. Prevalence of autoimmune C. Increasing calcitonin thyroiditis in adults has a 90%- concentration female predominance D. Increasing renal production of C. Patients with autoimmune 1,25-dihydroxyvitamin D-3 thyroiditis frequently develop (calcitriol) hypothyroidism E. Increasing intestinal absorption D. Autoimmune thyroiditis is also of calcium frequently part of the polyglandular autoimmune 279. Name the most often cause of syndromes primary hyperparathyroidism: E. Treatment for chronic A. Single parathyroid adenoma autoimmune thyroiditis may B. Multiple parathyroid adenoma require replacement therapy with C. Parathyroid glands hyperplasia Levothiroxin D. Parathyroid carcinoma E. Medullary thyroid carcinoma 276. Name hormones and vitamin which maintain calcium plasma 280. Primary hyperparathyroidism is homeostasis: 1. Parathyroid the unregulated overproduction of hormone; 2. Gastrin; 3. Calcitonin; parathyroid hormone resulting in: 4. Vitamin K; 5. Vitamin D3. A. Hypocalcemia A. 1, 2, 3 B. Hypercalcemia B. 1, 3, 5 C. Hypokalemia C. 1, 3, 4 D. Hyperkalemia D. 1, 2, 5 E. Vitamin D deficiency E. 2, 3, 5 281. A 22-year-old patient is scheduled 277. Usually, parathyroid glands are to under go parathyroidectomy for situated posterior to the thyroid hyperparathryoidism associated gland. How many parathyroid with familial multiglandular glands usualy have human? syndrome. His sister developed A. 1 peptic ulcer disease secondary to a B. 2 Zollinger-Ellison C. 3 (hypergastrinemia) tumor of the D. 4 pancreas. On examination, a E. 5 swelling was noted over the

42 posterior aspect of the patient’s fifth 284. Which one of the following is rib. What is the most likely finding? not part of the management of a A. Metastasis from a parathyroid patient with hyperparathyroidism carcinoma A. Hydration with intravenous B. Osteitis fibrosa cystica (brown normal saline tumor) and subperiosteal B. Steroids resorption of the phalanges C. Exploration of the neck for C. Dermoid cyst parathyroidectomy D. Eosinophilic granuloma D. Parathyroid scan E. Chondroma E. Vitamin D

282. The clinical syndrome of primary 285. Each adrenal gland is supplied hyperparathyroidism is presented by by small arterial branches that (choose the WRONG answer): originate from three distinct A. Osteopenia -> osteoporosis sources. Name these sources: 1. B. Renal stones Inferior phrenic artery; 2. Celiac C. Peptic ulcer trunk; 3. Aorta; 4. Renal artery; 5. D. Colon polips Common hepatic artery. E. Acute A. 1, 2, 3 B. 2, 3, 4 283. A 56-year-old women complaints C. 3, 4, 5 for bone and joint pain, , D. 1, 3, 4 vomiting, abdominal pain, E. 1, 4, 5 constipation, weakness and easy fatigability, depression, inability to 286. Which statement is WRONG concentrate, and memory problems. concerning the venous and In past medical history she has lymphatic drainage of adrenal suffered from renal stones in both gland? kidneys, peptic ulcer disease. What A. The right adrenal gland usually changes in the biochemical analysis drains by one short vein, which of blood will diagnose primary empties directly into the vena hyperparathyroidism? 1. Elevated cava ionized calcium; 2. Hypercalcemia; B. Accessory adrenal veins are not 3. Elevated intact parathyroid infrequently present hormone level; 4. C. The right adrenal vein drains into Hypophosphatemia; 5. Elevated 24- the right renal vein hour urine calcium. D. Lymphatic drainage from the A. 1, 2, 3 adrenal glands drains into B. 2, 3, 4 periaortic and paracaval nodes C. 1, 3, 5 E. The left adrenal vein drains into D. 1, 2, 4 the left renal vein E. All changes diagnose primary hyperparathyroidism 287. The adrenal gland is composed of two distinct organs, the adrenal cortex and the adrenal medulla. 43 Choose the WRONG statement pheochromocytoma. Which about adrenal physiology. statement is WRONG concerning A. The cortex is divided into three adrenal tumor biopsy? functional zones: the outer A. Percutaneous biopsy of the glomerulosa, the intermediate adrenal gland can be performed fasciculata, and the inner reticularis. under either CT or ultrasound B. Zona glomerulosa produce guidance mineralocorticoids B. Biopsy should be performed in C. Zona fasciculata produce all cases of adrenal masses glucocorticoids C. The most common indication is D. Zona reticularis produce sex suspicion of metastatic disease to steroids the adrenal gland E. Only mineralocorticoids are D. A percutaneous biopsy cannot absolutely required for life reliably distinguish between an adrenal adenoma and an adrenal 288. The adrenal medulla is carcinoma embryologically analogous to a E. Biopsy should never be peripheral sympathetic ganglia performed in a patient until a and consists of: biochemical workup has been A. Chromaffin tissue completed to rule out a B. Connective tissue pheochromocytoma C. Chromogranin tissue D. Muscle tissue 292. A 26-year-old patient was E. Epithelial tissue performed CT scan to rule out pathology of pancreas. On CT 289. The adrenal medulla produces: scans 6 cm tumor of left adrenal A. Estrogen gland was localized. What B. Aldosteron questions should be answered for C. Cortisol appropriate management of D. Epinephrine incidentaloma: 1. Is it functional?; E. Androsteron 2. Is it likely to be a malignant adrenal tumor? 3. Is it metastatic? 290. Choose examinations which are A. 1, 2 used for localization of adrenal B. 1, 3 masses: 1. Ultrasound; 2. CT scan; C. 2, 3 3. MRI; 4. Radioisotope scan; 5. D. All of them should be answered Intraoperative ultrasound. E. There is no need to answer to any A. 1, 2, 3 question B. 1, 2, 4 C. 1, 2, 5 293. A 26-year-old patient was D. 1, 3, 4 performed CT scan to rule out E. All are used pathology of pancreas. On CT scans 6 cm tumor of left adrenal 291. Sudden death has been reported gland was localized. What size of following biopsy of unsuspected 44 incidentaloma is an indication for urination. Objectively: surgical treatment? hyperpigmentation of skin, gums, A. 0,5 cm internal surface of cheeks. BP – B. 1,0 cm 90/58 mm Hg. Blood count: RBC- C. 2,0 cm 3,1*10 12 /L, Hb - 95 g/L, C.I.- 0,92; D.3,0 cm leukocytes - 9,4*10 9/L, eosinophils E. 4,0 cm - 7, segmentonuclear leukocytes - 45, stab neutrophils - 1, 294. *A 49 y.o. female patient was lymphocytes - 40, monocytes - 7, admitted to the hospital with acute Na +- 115 mmol/L, К+- 7,3 mmol/L. attacks of headache accompanied by What is the preliminary diagnosis? pulsation in temples, BP rised up to A. Congenital adrenocortical 280/140 mm Hg. hyperplasia Pheochromocytoma is suspected. B. Primary hyperaldosteronism What mechanism of hypertensive C. Pheochromocytoma atack does this patient have? D. Primary adrenocortical A. Increasing of thyroxine excretion insufficiency B. Increasing of catecholamines E. Diabetes insipidus concentration C. Increasing of plasma renin 297. *A 38 y.o. woman suffers from activity paroxysmal BP rises up to 240/120 D. Increasing of aldosterone level in mm Hg accompanied by nausea, blood vomiting, tachycardia, excessive E. Increasing of vasopressin sweating. During the onset blood is excretion hyperglycemic. After the onset there is voluminous urination. Kidneys 295. *A 40 y.o. patient was diagnosed: sonography revealed accessory 1. Medular thyroid gland cancer. 2. mass bordering upon the upper pole Pheochromocytoma. What of the right kidney, presumably it operation should be performed at belongs to the adrenal gland. What first? laboratory test will allow to make a A. Subtotal resection of thyroid more precise diagnosis? gland and fascicular resection of A. Estimation of thyroxin and limphatic nodes thyrotropic hormon in blood B. Krail's operation B. Estimation of insulin and C- C. Operation on account of peptide content in blood pheochromocytoma C. Estimation of catecholamine and D. Vanach's operation vanillylmandelic acid excretion with E. Operation on thyroid gland urine D. Estimation of renin content in 296. *A 34 y.o. patient has been blood suffering from pulmonary E. Estimation of glomerular tuberculosis for 7 years; he filtration rate complains of muscle weakness, weight loss, diarrhea, frequent 45 298. A 57-year-old woman presents E. Abdominal ultrasonogram with vague abdominal pain. After a course of treatment with H2- 301. A 30-year-old primigravida blockers failed and abdominal complains of headaches, ultrasound was negative, she restlessness, sweating, and underwent a CAT scan of the tachycardia. She is 18 wk pregnant abdomen. The scan was negative and her blood pressure is 200/120 except for the presence of a 3-cm mm Hg. Pheochromocytoma was mass in the left adrenal gland. Her suspected. Appropriate treatment pain disappeared. Urine and serum might consist of biochemical studies for a A. Therapeutic abortion functioning adrenal tumor are B. Urgent excision of the tumor and negative. Her past medical history is a therapeutic abortion negative. The next step should be: C. Phenoxybenzamine and A. Adrenalectomy propranolol followed by a B. CT-guided percutaneous core combined cesarean section and needle biopsy excision of the tumor C. Arteriography D. Propranolol blockade followed D. MRI by a combined cesarean section E. Repeat CAT scan in 3 months and excision of the tumor E. Phenoxybenzamine and 299. A 40 y.o. patient was diagnosed: propranolol followed by a 1. Medular thyroid gland cancer. 2. combined vaginal delivery at Pheochromocytoma. Make the term and excision of the tumor diagnosis? A. MEN-1 302. Which of the following statements B. MEN-2 concerning Cushing syndrome C. Zolliger-Ellison syndrome secondary to adrenal adenoma is D. Watery diarrhea, hypokalemia, true? and (WDHA) A. Adrenal adenomas cause 80% of syndrome all cases of Cushing syndrome E. Werner’s syndrome B. CT scan is generally unsuccessful in lateralizing the 300. A 30-year-old primigravida tumors preoperatively complains of headaches, C. Exploration of both adrenal restlessness, sweating, and glands is indicated tachycardia. She is 18 wk pregnant D. For uncomplicated small tumors, and her blood pressure is 200/120 an open transperitoneal surgical mm Hg. Pheochromocytoma was approach is usually employed suspected. What examination E. Postoperative corticoid therapy is should be performed? required to prevent A. Exploratory hypoadrenalism B. Biopsy of adrenal mass C. Radioisotope scan 303. Primary aldosteronism is defined D. Abdominal CT scan as excessive secretion of 46 aldosterone. Choose WRONG statement concerning primary 306. Patients with endogenous hyperaldosteronism? Cushing’s syndrome caused by a A. Aldosteronomas occur in pituitary adenoma will have: 1. approximately 65% of patients with Elevated cortisol level; 2. Decreased primary aldosteronism cortisol level; 3. Elevated ACTH; 4. B. Idiopathic hyperaldosteronism is Decreased ACTH; 5. Hypokalemia. caused by bilateral adrenal A. 1, 4, 5 hyperplasia B. 1, 3, 5 C. Aldosteronomas are almost C. 2, 3, 4 always unilateral and are often less D. 1, 2, 3 than 2 cm in size E. All are correct D. Unilateral adrenalectomy in the setting of Idiopathic 307. Cushing’s syndrome can be hyperaldosteronism is curative caused by: 1. Adrenocortical E. Adrenal cortical aldosterone- adenoma; 2. Adrenocortical producing carcinoma is carcinoma; 3. Pituitary extremely rare and represents corticotroph adenoma; 4. Ectopic another surgically treatable form secretion of ACTH by of primary aldosteronism nonpituitary tumor; 5. Ectopic secretion of corticotropin- 304. Main clinical signs of primary releasing hormone. hyperaldosteronism are: 1. Arterial A. 1, 2, 3 hypertension; 2. Arythmia; 3. B. 1, 2, 4 Hypokalemia; 4. Increased C. 2, 4, 5 aldosterone level; 5. Decreased D. 1, 3, 5 plasma renin activity. E. All can cause A. 1, 2, 3 B. 1, 3, 5 308. Choose the main clinical signs C. 1, 4, 5 typical for Cushing’s syndrome: 1. D. 2, 3, 5 Centripetal obesity; 2. E. All are correct Hypertension; 3. Osteoporosis; 4. Acne; 5. Polyuria, polydipsia. 305. Patients with endogenous A. 1, 2, 3 Cushing’s syndrome caused by a B. 1, 2, 4 unilateral adrenal tumor will have: C. 2, 4, 5 1. Elevated cortisol level; 2. D. 1, 3, 5 Decreased cortisol level; 3. Elevated E. All are typical ACTH; 4. Decreased ACTH; 5. Elevated 24-h urinary free cortisol. 309. For Cushing’s syndrome is typical A. 1, 4, 5 deposition of adipose tissue in B. 1, 3, 5 characteristic sites: 1. Face C. 2, 3, 4 ("moon" facies); 2. Interscapular D. 1, 2, 3 area ("buffalo hump"); 3. E. All are correct Mesenteric bed ("truncal" 47 obesity); 4. Lower extremity; 5. E. abscess Upper extremity. A. 1, 2, 3 313. A 28-year-old female figure skater B. 1, 2, 4 presents several weeks after having C. 2, 4, 5 sustained an injury to her left breast. D. 1, 3, 5 She has a painful mass in the upper E. All are typical outer quadrant. Skin retraction is noticed, and a hard mass, 3–4 cm in 310. A 35-year-old professional dancer diameter, can easily be palpated. presents with a well-defined, tense, What is the most likely diagnosis? smooth mass in the upper outer A. Infiltrating carcinoma quadrant of the left breast. She B. Breast abscess states that the mass becomes larger C. Hematoma just before onset of her periods. D. Aspiration yields a clear yellow E. Sclerosing adenosis fluid and the mass disappears. The most likely diagnosis is: 314. A 35-year-old patient presents to A. Fibroadenoma in a cyst your office with chronic draining B Breast cyst subcutaneous periareolar abscesses, C Carcinoma in a cyst which have been incised and D. Lipoma drained many times in the past 5 E. years but keep recurring. What is the best treatment of choice? 311. An 23-year-old presents with a A. Repeat incision and drainage (I wellcircumscribed 2-cm mass in her and D) since the previous right breast. The mass is painless procedures were inadequate and has a rubbery consistency and B. Long-term antibiotics discrete borders. It appears to move C. Major duct excision freely through the breast tissue. D. Complete excision of the What is the likeliest diagnosis? drainage tract A. Carcinoma E. Tell the patient there is nothing B. Cyst to do and that this will C. Fibroadenoma eventually resolve with age D. Cystosarcoma phyllodes E. Intramammary lymph node 315. A patient presents 1 month after a benign right breast biopsy with a 312. , a milky discharge lateral subcutaneous cord felt just from the in nonpregnant under the skin and causing pain. women, is most likely to be The etiology of this condition is? associated with which of the A. Fat necrosis following? B. Infection A. Fibroadenoma C. Superficial thrombophlebitis B. Tubular adenoma D. Suture granuloma C. Pituitary adenoma E. Misdiagnosed breast cancer D. Hyperparathyroidism 48 316. A 36-year-old woman complains B. Previous history of benign of a 3-month history of bloody breast biopsies discharge from the nipple. At C. Atypia seen on pathology from examination, a small nodule is previous breast biopsy found, deep to the areola. Careful D. First-degree relative with history palpation of the nippleareolar of breast cancer complex results in blood arrearing E. Increasing age at the 3 O’clock position. Mammogram findings are normal. 319. A 46-year-old woman presents What is the likeliest diagnosis? with a mammogram that shows a 1- A. Intraductal papilloma cm cluster of fine calcification in B. Breast cyst the right breast. Following C. Intraductal carcinoma mammographic wire localization, D. the lesion is excised and the E. Fat necrosis pathology reported as ductal carcinoma in situ (DCIS) with 317. During a routine screening comedo features and free margins. , a 62-year-old What advice should be given to the teacher is informed that she has patient? changes on her mammography, and A. If untreated, about 30% of such she should consult her physician. lesions become invasive over a She can be reassured that the 10-year period findings that indicate a benign B. Comedo DCIS is less aggressive condition are which of the than noncomedo DCIS following? C. Bilateral mastectomy and A. Discrete, stellate mass radiotherapy are the preferred B. Fine, clustered calcifictions treatments C. Coarse calcifications D. Axillary node is D. Solid, clearly defined mass with always indicated irregular edges E. Total mastectomy carries a high E. Discrete, nonpalpable mass that (50%) risk of carcinoma has enlarged when compared recurrence. with a mass shown on a mammogram taken 1 year 320. Breast cancer most often occurred previously in age: A. 25-30 318. A 40-year-old lawyer comes into B. 30-40 your office after seeing some C. 40-50 information on the Internet relating D. 50-60 to breast cancer. Which of the E. 30-50 following factors has not shown to increase a woman’s risk for breast 321. A 50-year-old woman underwent cancer? wide excision of a 2.5-cm A. Smoking infiltrating ductal carcinoma of the breast with axillary lymph node 49 dissection followed by radiation and induration of the left mammary chemotherapy 2 years ago. The gland, body temperature rise up to patient now complains of RUQ 39 0С, headache, indisposition. abdominal pain. CT scan reveals Objectively: fissure of nipple, two masses in the right lobe of the enlargement of the left mammary liver. Select the most likely gland, pain on palpation. What diagnosis. pathology would you think about in A. Adenoma this case? B. Focal nodular hyperplasia A. Lactational C. Hemangioma B. Lacteal cyst with suppuration D. C. Breast cancer E. Metastatic carcinoma D. Fibrous adenoma of the left 322. A 52-year-old undergoes a left E. Phlegmon of mammary gland modified radical mastectomy for a 2-cm breast cancer. She should be 325. A 40 year old woman has changes informed that the factor which has of mammary gland. What is the the greatest impact on her prognosis most often symtomp that precede is? the malignization? A. The size of the primary tumor A. Painless movable induration B. The histological type of the B. Bloody discharges from the carcinoma nipple C. The number of axillary nodes C. Pure discharges from the nipple positive for metastasis D. Painful movable induration D. Hormonal receptor status of the E. Skin induration with inverted primary tumor nipple E. Positive findings on tests for the presence of the BRCA(breast 326. A parturient complains about pain cancer)1 gene in the mammary gland. Palpation revealed a 3 х4 cm large infiltration, 323. A young man has painful soft in the centre. Body temperature indurations in the peripapillary is 38,5 oC. What is the most probable regions of both mammary glands. diagnosis? The most reasonable action will be: A. Retention of milk A. To take an aspirate for bacterial B. Birth trauma inoculation and cytology C. Acute purulent mastitis B. To cut and drain them D. Pneumonia C. To administer steroids locally E. Pleuritis D. To remove them E. To leave these indurations 327. Blunt chest trauma can be caused untouched most oftenly by: 1. Motor vehicle crashe; 2. Stab wound; 3. Height 324. A woman consulted a doctor on falls; 4. Airplane crash; 5. Bullet the 14-th day after labor about wound: sudden pain, hyperemy and A. 1, 2, 3 50 B. 2, 3, 4 paradoxically with respiration. At C. 3, 4, 5 signs of D. 2, 3, 5 tensionpneumothorax absent. Make E. 1, 3, 4 the diagnosis? A. Hemothorax 328. Name the most common thoracic B. Sternal fracture injury in blunt trauma? C. Flail chest A. Hemothorax D. Esophageal injury B. Flail chest E. Aorta rupture C. Rib fracture D. Sternal fracture 332. A 70-year-old man is brought into E. Pericardial tamponade the emergency department following his injury as a passenger 329. A 45-year-old man skidded from in a car crash. He complains of right the road at high speed and hit a tree. side chest pain. Physical Examples of deceleration injuries in examination reveals a respiratory this patient include: rate of 42 breaths per minute and A. Aortic valve rupture multiple broken ribs of a segment of B. Aortic arch rupture the chest wall that moves C Posterior dislocation of shoulder paradoxically with respiration. At D. Hemothorax thoracentesis signs of E. Rib fracture tensionpneumothorax absent. What should the next step be? 330. Choose life-threatening injuries, A. Tube thoracostomy which should be identified B. Tracheostomy immediately: 1. ; C. Insertion of a nasogastric tube 2. Tension pneumothorax; 3. D. Endotracheal intubation Massive hemothorax; 4. Cardiac E. Intercostal nerve blocks tamponade; 5. Pulmonary contusion. 333. An 18-year-old man is brought to A. 2, 3, 4 the emergency department with a B. 1, 2, 3 stab wound just to the right of the C. 1, 3, 5 sternum in the sixth intercostal D. 1, 4, 5 space. His blood pressure is 80 mm E. 2, 3, 5 Hg. Faint heart sounds and are noted. Auscultation 331. A 70-year-old man is brought into of the right chest reveals markedly the emergency department decreased breath sounds. The initial following his injury as a passenger management of this patient should in a car crash. He complains of right be which of the following? side chest pain. Physical A. Aspiration of the right chest examination reveals a respiratory cavity rate of 42 breaths per minute and B. Analgesics multiple broken ribs of a segment of C. Echocardiogram the chest wall that moves D. Pericardial window 51 E. Insertion of central venous access 337. A 26-year-old man is stabbed in line the right intercostal space in the midclavicular line and presents to 334. An 18-year-old man presents to the emergency department. On the emergency department with a examination, subcutaneous gunshot wound to the left chest in emphysema of the right chest wall, the anterior axillary line in the sixth absent breath sounds, and a trachea intercostal space. His blood pressure shifted to the left are noted. What is is 120/70 mm Hg, pulse – 78 bpm. the most likely diagnosis? A sucking sound is audible during A. Pneumothorax inspiration. Immediate management B. Tension pneumothorax is which of the following? C. Massive hemothorax A. Exploratory laparotomy D. Hemopneumothorax B. Exploratory thoracotomy E. Chest wall laceration C. Pleurocentesis D. Closure of the hole with sterile 338. A 31-year-old man is shot in the dressing back of the left chest, and the bullet E. Insertion of central venous access exits the left anterior chest. The line patient’s blood pressure is 130/90 mm Hg, respiration rate is 28 335. While landing at the end of flight breaths per minute, and pulse is 110 a young man develops shortness of bpm. A chest x-ray reveals breath and rightsided pressure chest hemothorax. A chest tube is inserted pain. He is tall and thin. He has not and yields 800 mL of blood; the previously consulted a doctor. A first and second hour drainage is chest film is likely to show? 200 mL/h and 240 mL/h, A. Left respectively. What is the next step B. Spontaneous pneumothorax in management? C. Dilated stomach A. Place a second chest tube D. Hemothorax B. Collect the blood for E. Cardiomegaly autotransfusion C. Transfuse and observe drainage 336. While landing at the end of flight for another hour a young man develops shortness of D. Insert a Swan-Ganz catheter breath and rightsided pressure chest E. Perform a left thoracotomy pain. He is tall and thin. He has not previously consulted a doctor. The 339. A 31-year-old man is shot in the treatment is: back of the left chest, and the bullet A. Insertion of a chest tube exits the left anterior chest. The B. Immediate cardiology consult patient’s blood pressure is 130/90 C. Thoracentesis mm Hg, respiration rate is 28 D. Insertion of a nasogastric tube breaths per minute, and pulse is 110 E. Thoracoscopy bpm. A chest x-ray reveals hemothorax. A chest tube is inserted and yields 800 mL of blood; the 52 first and second hour drainage is 343. Which statement is wrong 200 mL/h and 240 mL/h, concerning sternal fracture? respectively. In the patient A. Is seen in 60% of patients with described above the most likely blunt trauma cause of the bleeding is injury to B. Is very uncommon injury which of the following? C. Needs large traumatic force A. Pulmonary artery D. Is caused by direct blow to front B. Lung parenchyma of the chest C. Internal thoracic (mammary) E. Is associated with high rate of and/or intercostals arteries myocardial contusion and cardiac D. Pulmonary vein tamponade E. Left atrium 344. A 25-year-old man is shot in the 340. Which statement concerning 1 st left lateral chest. In the emergency and 2 nd ribs fractures is wrong? department, his blood pressure is A. Require high force 120/90 mm Hg, pulse rate is 104 B. Frequently have injury to aorta beats per minute (bpm), and C. Frequently have injury to bronchi respiration rate is 36 breaths per D. May injure subclavian minute. Chest x-ray shows air and artery/vein fluid in the left . E. Causes pulsus paradoxicus Nasogastric aspiration reveals blood-stained fluid. What is the best 341. Most oftenly fracture of 11 th or step to rule out esophageal injury? 12 th ribs are associated with: A. Insertion of chest tube A. Flail chest B. Insertion of nasogastric tube B. Damage to underlying abdominal C. Esophagogram with gastrografin solid organs (liver, spleen, kidney) D. Esophagoscopy C. Injury to aorta E. Peritoneal lavage D. Injury to bronchi E. Pneumothorax 345. Because of his involvement in a motor vehicle accident, a 23-year- 342. In what cases patients with rib old football player has a chest wall fractures should be treated injury. The only abnormal findings immediately or monitored carefully: on clinical and radiologic 1. Elderly patients; 2. Patients with examination are a fracture of the left concomitant heart diseases; 3. fifth to seventh ribs and a small Patients with COPD; 4. Patients hemothorax. What should treatment with multiple rib fractures; 5. include? Patients with flail chest. A. Insertion of an intercostal drain A. 1, 2, 3 to avoid pneumothorax B. 2, 4, 5 B. Thoracotomy to treat a small C. 3, 4, 5 hemothorax in the left base D. 1, 2, 4 C. Insertion of a metal plate to fix E. All are correct the fracture

53 D. Administration of analgesic minute. She complains of right medication lower chest wall and severe right E. Administration of cortisone to upper quadrant (RUQ) tenderness. prevent callus formation Her breath sounds are questionably diminished. The immediate priority 346. A 25-year-old woman was stabbed is to perform which of the by her boyfriend in the left chest. following? On examination, she has a 1-cm A. Peritoneal lavage stab wound just inferior to her left B. Chest x-ray breast in the mid-clavicular line. C. CT scan of chest and abdomen There is jugular venous distension D. Thoracentesis with an 18-gauge and breath sounds are completely needle absent on the left side. She is E. Endotracheal intubation becoming extremely dyspneic and hypoxic. Make the diagnosis. 349. In the case of isolated A. pneumothorax the tube should be B. Tension pneumothorax placed in the: C. Massive hemothorax A. Second intercostal space, D. Flail chest anterior axillary line E. Rupture diaphram B. Second intercostal space, midclavicular line 347. A 45-year-old man was a C. Second intercostal space, mid passenger in a car when he was T- axillary line boned by a truck at a high speed. He D. Fifth intercostal space, is short in breath, complains of midclavicular line severe pain in the chest, and is E. Fifth intercostal space, mid hypoxic on the pulse oximeter. The axillary line breath sounds are diminished on the left and the percussion note is 350. In the case of hemothorax the tube completely dull. He rapidly should be placed in the: becomes tachycardic and A. Second intercostal space, hypotensive. anterior axillary line A. Cardiac tamponade B. Second intercostal space, B. Tension pneumothorax midclavicular line C Open pneumothorax C. Second intercostal space, mid D. Flail chest axillary line E. Massive hemothorax D. Fifth intercostal space, midclavicular line 348. A 40-year-old woman is brought E. Fifth intercostal space, mid to the emergency department axillary line following a car crash in which she was the driver. In the emergency 351. A 55-year-old man involved in an department, her blood pressure is automobile accident is unresponsive 80/60 mm Hg, pulse is 128 bpm, and is intubated at the scene. On and respiratory rate is 36 breaths per arrival in the emergency 54 department, he responds to painful A. Insert a chest tube and observe stimulation. His systolic BP is 60 for drainage. mm Hg, his HR is 140 bpm, his B. Perform an immediate right neck veins are distended, and his thoracotomy. breath sounds are absent on the left C. Perform an angiogram to rule out side. Immediate management great vessels injury. should involve which of the D. Perform median sternotomy with following? extension along with right anterior A. Insertion of a central venous line boarder of the sternocleidomastoid on the right side muscle. B. Insertion of an 18-gauge needle E. Perform a CAT scan with in the left second intercostal space contrast, to evaluate extent of C. Pericardiocentesis injury. D. Peritoneal lavage E. CT scan of head 354. A 31-year-old man is brought to the emergency room following an 352. During a car crash a young man automobile accident in which his suffers bilateral multiple fracture chest struck the steering wheel. ribs. He is alert and presents Examination reveals stable vital shortness of breath. His blood signs, but the patient exhibits pressure is 100/60 mm Hg and chest palpable 7 rib fractures from the is unstable. Treatment for this is: right side and paradoxical A. Prolonged intubation and movement of the right side of the ventilatory support until rib chest. Chest x-ray shows no fractures heal along with aggressive evidence of pneumothorax or bronchial toilette. hemothorax, but a large pulmonary B. Once the patient is stable, open contusion is developing. Proper rib fracture reduction and treatment would consist of which of stabilization with plates. the following? C. Fracture stabiliztion, with towel A. Tracheostomy, mechanical clips on ribs and attached to weights ventilation, and positive end- (external fixation). expiratory pressure D. Avoid intubation, control pain, B. Stabilization of the chest wall and perform aggressive bronchial with sandbags toilette. C. Stabilization with towel clips E. Temporary extracorporeal D. Immediate operative stabilization circulation to allow fractures to E. No treatment unless signs of heal. respiratory distress develop

353. A young man is shot at the level 355. Select the proper intervention for of the right sternoclavicular joint. a life-threatening injury of the chest: His blood pressure is 80/60 mm Hg, LARYNGEAL OBSTRUCTION pulse 120 bpm, and a chest xray A. Endotracheal intubation shows a right hydropneumothorax. B. Cricothyroidotomy The first step should be: C. Subxiphoid window 55 D. Tube thoracostomy associated with ribs fracture; 4. A E. Occlusive dressing massive air leak following chest tube insertion; 5. High rate of blood 356. 29. Select the proper intervention loss via the chest tube (>200 ml/h). for a life-threatening injury of the A. 1,2,3 chest: OPEN PNEUMOTHORAX B. 1,3,5 A. Endotracheal intubation C. 1,4,5 B. Cricothyroidotomy D. 2,4,5 C. Subxiphoid window E. 2,3,4 D. Tube thoracostomy E. Occlusive dressing 361. Name indications for immediate surgery in patients after chest 357. Select the proper intervention for trauma: 1. Traumatic disruption a life-threatening injury of the chest: with loss of chest wall integrity; 2. FLAIL CHEST Cardiac tamponade; 3. A. Endotracheal intubation Tracheoesophageal fistula; 4. A B. Cricothyroidotomy persistent thoracic duct C. Subxiphoid window fistula/; 5. Large vessel D. Tube thoracostomy injury. E. Occlusive dressing A. 1, 3, 4 B. 1, 2, 5 358. Select the proper intervention for C. 1, 3, 5 a life-threatening injury of the chest: D. 2, 4, 5 TENSION PNEUMOTHORAX E. 2, 3, 4 A. Endotracheal intubation B. Cricothyroidotomy 362. What is the cornerstone in C. Subxiphoid window management of patients with rib D. Tube thoracostomy fractures? E. Occlusive dressing A. Pain control B. Immediate surgery 359. Select the proper intervention for C. Endotracheal intubation a life-threatening injury of the chest: D. Ipsilateral chest tube placement PERICARDIAL TAMPONADE E. Diagnostic thoracotomy A. Endotracheal intubation B. Cricothyroidotomy 363. Concerning thoracic trauma which C. Subxiphoid window statement IS NOT true? D. Tube thoracostomy A. Rib fractures do not require E. Occlusive dressing surgery B. Flail chest is associated with 360. Name please indications for paradoxical motion of the flail immediate surgery in patients with segment chest trauma: 1. Traumatic C. Cardiac tamponade is an disruption with loss of chest wall indications for immediate surgery integrity; 2. A chronic clotted D. First and second rib fractures are hemothorax; 3. Athelectasis caused by excessive energy force 56 E. Isolated first and second rib D. 2,4,5 fractures require surgical therapy E. 2,3,4

364. What is the obvious sign of 367. All conditions belong to blunt diaphragmatic disruption on chest cardiac injuries, except: radiographs? A. Rupture of the valves, A. Abdominal visceral herniation B. Rupture of interventricular into the chest septum B. Absence of complete expansion C. Cardiac chamber rupture of the lung D. Cardiac tamponade C. Distended shade of the E. Atrioventricular stenosis D. Mediastinum dislocation to the 368. Which condition CAN NOT occur contralateral side in patients with chest trauma? E. Evidence of ipsilateral A. Pericardial tamponade pneumothorax B. Esophageal rupture C. Esophageal ahalasia 365. Which statement IS NOT true D. Heart contusion сoncerning thoracic trauma? E. Main bronchial disruption A. All patients with pneumothorax due to trauma need a tube 369. Trauma is the leading cause of thoracostomy death, morbidity, hospitalization, B. Open pneumothorax is caused by and disability in Americans in the lung tissue defect that is larger age: than the cross-sectional area of A. 0 – 1y the larynx B. 1 – 10y C. Treatment for an open C. 10 – 45y pneumothorax consists of D. 45 – 65y placing a 3-way occlusive E. More than 65y dressing over the wound D. Tension pneumothoraces are 370. What injury DOES NOT always life-threatening states compromise ventilation? E. Large, clotted hemothoraces may A. Lung contusion require an operation B. Open pneumothorax C. Painfull ribs fracture 366. What are the primary therapies for D. Flail chest pulmonary contusions? 1. Surgical E. Acute cardiac tamponade debridement; 2. Pain control, iv fluid restriction; 3. Immobilization 371. Measurement of serum creatine with a figure-of-eight dressing; 4. kinase isoenzyme (creatine kinase- Pulmonary toilet; 5. supplemental MB) levels is frequently oxygen. performed in patients with A. 1,3,4 possible: B. 1,3,5 A. Blunt myocardial injury C. 1,2,5 B. Multiple ribs fracture 57 C. Traumatic asphyxia sufficiently to drain its D. Flail chest developing contents, cavitation can occur E. Tension pneumothorax D. Lung abscesses occur in patients with oral or dental infections 372. What is the initial study of choice who sustained a depression in in patients with thoracic blunt their level of consciousness and trauma, suspicious on aspirated their oral secretions pneumothorax? E. Abscess is walled off the A. The chest radiogram infectious process B. 12-lead ECG C. Echocardiography 376. Name the microorganism, which D. CT scaning causes aspiration most E. Ventilation test oftenly: A. Anaerobic bacteria 373. Ultrasound examinations of B. Gram-positive bacteria thoracic cavities can be performed C. Gram-negative bacteria to confirm the diagnosis: D. Fungal infections A. Acute cardiac tamponade E. Protozoan infection B. Open pneumothorax C. Multiple ribs fracture 377. Choose incorrect statement D. Main bronchi disruption concerning instrumental diagnosis of E. Multiple athelectasis lung abscess: A. The diagnosis is confirmed with 374. Choose signs typical for a chest radiograph pneumonia: 1. Abnormal temperature; B. A chest radiograph demonstrate 2. White blood cells in an air–fluid level the sputum; 3. Abnormal white blood C. Bronchoscopy is essencial in cell count; 4. Infiltrate on chest making the diagnosis lung radiographs; 5. Crepitation on abscess auscultation: D. In patients with concomitant A. 1, 2, 3 lung diseases, most efficient in B. 1, 2, 4 diagnosing lung abscess is CT C. 1, 2, 5 scan D. 2, 4, 5 E. The goal of bronchoscopy is to E. All are correct determine the bacterial identity and sensitivities 375. Which statement is wrong concerning lung abscess? 378. Which statement concerning the A. Infecting organisms involved in treatment of patients with lung abscess a pneumonic process in the lung is wrong? will promote abscess formation A. Most lung abscesses will B. Most lung abscesses occur in the respond and resolve with upper lobe of lung appropriate antibiotic therapy C. If the abscess cavity erodes into and pulmonary toilet a bronchoalveolar space 58 B. Surgical therapy is reserved for C. Gram-negative bacteria those cases that fail to resolve D. Fungal infections with nonoperative management E. Protozoan infection C. Most lung abscesses will require surgery 382. Which diseases can cause D. Surgical therapy is reserved for empyema of the pleura: 1. those patients who develop Parapneumonic effusion; 2. Congestive severe hemoptysis, heart failure; 3. Lung abscess cavity bronchopleural fistula or erodes into a pleural space; 4. empyema Tuberculosis; 5. Collagen-vascular E. Pulmonary resection is disease. warranted if the abscess cavity is A. 1, 3, 4 larger than 6 cm in diameter for B. 1, 2, 4 more than 8 weeks of aggressive C. 1, 4, 5 antibiotic therapy D. 2, 3, 4 E. 3, 4, 5 379. After suffering a severe bout of pneumonia, a 46-year-old renal 383. Choose correct principles in transplantation patient develops a lung treatment of empyema: 1. Complete abscess. She has been receiving evacuation of the content of immunosuppression therapy since her infected space; 2. Elimination of last kidney transplantation 3 years ago. cavity; 3. Control of causative What is the most appropriate organisms/sterilisation; 4. Nutritional treatment? support; 5. Ventilation support: A. Needle aspiration A. 1, 2, 3 B. Urgent thoracotomy B. 2, 3, 4 C. Antituberculous therapy C. 1, 4, 5 D. Antibiotics and vigorous D. 3, 4, 5 attempts to obtain bronchial E. All are correct drainage E. Insertion of an intercostal pleural 384. Which microbiological aspect is drin not correct concerning empyema: A. 5-10% of parapneumonic 380. What minimal volume of pleural effusions become empyema effusion can be seen on chest film? B. A lack of detectable causative A. 50 cc organisms reported not more B. 70 cc then 10% C. 175 cc C. Blood cultures positive in only D. 600 cc 10% of patients E. 1000 cc D. Community acquired empyema most oftenly is caused by 381. Name the microorganism, which Streptococcus pneumoniae causes empyema most oftenly: E. Hospital-acquired empyema A. Anaerobic bacteria most oftenly is caused by B. Gram-positive bacteria Staphylococcus aureus 59 E. Chronic obstructive pulmonary 385. Immediately following a bout of deseases pneumonia, a young woman develops a large pleural effusion. A chest tube is 388. What is the most common inserted and 600 mL of thin pus is symptom associated with pleural obtained. A CAT scan shows effusion? incomplete drainage and multiple A. Dyspnea intrapleural loculations. Management B. Caugh of this empyema requires: C. Pain A. Insertion of multiple chest D. Diffuse edema tubes under CAT guidance to E. Anaemia drain either most or all loculations 389. Physical examination is NOT B. Treat the patient with informative until pleural effusion antibiotics and continue single exceeds: chest tube drainage A. 100 mL C. Treat patient with antibiotics B. 300 mL and continue single chest tube C. 30 mL drainage waiting for a thick D. 1000 mL peel to develop and then E. 2500 mL proceed with open total lung decortication 390. In the case of pleural D. Proceed with thoracoscopy and effusion physical findings include intrapleural toilette. Break the all of the following, EXCEPT: loculations and place drains A. Decreased breath sounds E. A thorough open total lung B. Dullness to percussion decortication immediately C. Decreased tactile fremitus D. Bronchial breath sounds 386. Which statements are NOT true? E. Pleural friction rub A. The lungs occupy most of the volume of each hemithorax 391. Transudates are caused by B. Each lung is divided into lobes a small, defined group of etiologies, C. The right lung has 3 lobes including the following: 1). D. The left lung has 2 lobes Congestive heart failure, 2). Spread E. Each segment is further divided TB infection, 3). Atelectasis due to into lobes. malignancy or pulmonary embolism, 4). Hypoalbuminemia, 387. The most often pleural effusion is 5). Congenital lung deseases. caused by all of the following, A. 1,2,3 EXCEPT: B. 1,3,4 A. Congestive heart failure C. 1,4,5 B. Malignancy D. 2,3,5 C. Infections E. 2,4,5 D. Pulmonary embolism 60 392. The most common causes of 4). Fistula formation; 5). Spleen exudates include the following: 1). or liver puncture Congestive heart failure, 2). A. 1,3,5 Pulmonary embolism, 3). B. 1,2,4 Esophageal perforation, 4). C. 1,3,4 Myxedema, 5). Chylothorax D. 2,3,5 A. 1,2,3 E. 2,4,5 B. 1,3,4 C. 1,4,5 396. The recommended limit D. 2,3,5 in a single thoracentesis procedure E. 2,4,5 is: A. 1000-1500 mL 393. Choose indications for diagnostic B. 150-200 mL thoracentesis from the following: 1). C. 300-450 mL Etiology of the effusion is unclear; D. 600-7500 mL 2). Effusion does not respond to E. 50-100 mL therapy as expected; 3). Small amoung of effusion in clinically 397. Pleurodesis is most stable patients; 4). Underlying oftenly used for: congestive heart failure; 5). A. Recurrent malignant effusions Suspected malignancy B. Spreading TB infection A. 1,2,5 C. COPD associated with pleural B. 1,2,3 effusion C. 1,4,5 D. If diagnosis of pyopneumothorax D. 2,3,4 is confirmed E. 3,4,5 E. Empiema associated with lung abscess 394. Name contraindications for diagnostic thoracentesis: 1). 398. Surgical intervention is ; 2). Small most oftenly required for: volume of fluid; 3). A. Parapneumonic effusions that Hypocoagulation; 4). Suspected cannot be drained adequately by malignancy; 5). Mechanical needle ventilation B. Lung abscess development after A. 1,2,5 pneumonia B. 1,2,4 C. Total hemithoracis effusion C. 1,3,4 without clear diagnosis D. 2,3,5 D. Multiple lung abscesses E. 3,4,5 E. Lung abscess with purulent sputum more than 100 mL/day 395. Complications of diagnostic thoracentesis include: 1). Internal 399. In the management of bleeding; 2). Systemic chylous effusions following action hypotension; 3). Pneumothorax; is efficient: A. Restrictions of fat intake 61 B. Restrictions of water intake 403. What condition DO C. Meal rich for proteins NOT cause secondary abscess of the D. Meal rich for carbohydrate lungs: E. All are correct A. Preexisting bronchial obstruction B. Infection spreading from an 400. Which statement extrapulmonary location concerning effusion, obtained by C. diagnostic thoracocentesis, is NOT D. Immunocompromised state of TRUE? the patient A. A putrid odor suggests an E. Heart failure anaerobic empyema. B. A milky, opalescent fluid 404. Most frequently, the lung suggests a chylothorax abscess is: C. All are correct A. complication of aspiration D. Frankly purulent fluid indicates pneumonia caused by mouth an empyema. anaerobes E. Grossly bloody fluid may result B. caused by Peptostreptococcus from trauma, malignancy species C. unrecognizable on chest X-ray 401. The fluid, obtained by film diagnostic thoracocentesis, is D. unexpected finding in healthy considered an exudate if (choose patients one correct sign): E. complication after acute A. Ratio of pleural fluid to serum bronchitis protein greater than 0.5 B. Low pleural fluid glucose 405. Which mechanism in concentration (< 50 mg/dL) pathogenesis of lung abscess C. Ratio of pleural fluid to serum formation is WRONG? lactate dehydrogenase (LDH) lesser A. Bacteremia than 0.1 B. Tricuspid valve endocarditis D. Any of them C. Lemierre syndrome (acute E. Pleural fluid pH less than 5.1 oropharyngeal infection followed by septic 402. Definition of lung abscess thrombophlebitis of the internal includes all, except: jugular vein) A. necrosis of the pulmonary tissue D. Pulmonary embolism B. formation of cavities containing E. Severe necrotic debris C. formation of cavities containing 406. What factors ARE NOT necrotic fluid associated with a poor prognosis in D. caused by microbial infection treatment of lung abscesses: E. obstructive changes in the small A. advanced age bronchi B. malnutrition C. immunodeficiency D. malignancy 62 E. mitral regurgitation E. 2,4,5

407. Which statements are 411. Name the most often NOT true? reason for poor response to A. Male sex predominates for lung treatment with antibiotics, in abscess patients with lung abscesses: B. Lung abscesses likely occur A. Bronchial obstruction with a more commonly in elderly patients foreign body C. Incidence of periodontal disease B. Tumor increase likelyhood of lung abscess C. Reccurent aspiration formation D. Fungial infection D. Alcoholism is preexisting factor E. Poor lung perfusion for abscess occurring E. Cigarette smoking is strong 412. The usual indications for underlying factor for lung abscess surgery, in patients with lung abscess, are : 1). coexsisting severe 408. The unusual symptom of obstructive lung desease, 2). acute lung abscess is: suspected , 3). congenital A. Fever lung malformation, 4). failure to B. Cough with sputum discharge respond to medical management, 5). C. Sweating amount of purulent sputum more D. Chest pain than 100ml/day E. Weight loss A. 1,2,3 B. 1,3,5 409. Objective findings in C. 1,4,5 patient with lung abscess are all, D. 2,3,4 EXCEPT: E. 2,4,5 A. Decreased breath sounds B. Dullness on percussion 413. Name the complications C. Bronchial breath sounds of pulmonary abscess: 1). Rupture D. Inspiratory crackles into pleural space causing E. Tachicardia associated with empyema, 2). Pulmonary bradypnoe hypertension, 3). Spreading bronchial sclerosis, 4). 410. Patients at the highest risk Bronchopleural fistula, 5). for developing lung abscess have Respiratory failure. the following risk factors: 1). A. 1,2,3 Periodontal disease, 2). Mild to B. 1,3,5 severe heart failure, 3). Alcohol C. 1,4,5 abuse, 4). Dysphagia, 5). Antibiotic D. 2,3,4 use in past. E. 2,4,5 A. 1,2,3 B. 1,3,4 414. Which of the following is C. 1,4,5 FALSE? D. 2,3,5 63 A. Postaspiration lung abscesses are indistinct outlines on the right associated with aerobic bacteria side at the level of the 4-th rib. In B. Prevention of aspiration is the centre of this opacity there is a important to minimize the risk of horizontal level and clearing of lung lung abscess tissue above it. What disease does C. Early intubation in disable this X-ray pattern correspond with? patients protects the airway from A. Right-sided hemothorax massive aspiration B. Abscess of the right lung D. The most common is aspiration C. Peripheral cancer of oropharyngeal contents. D. Right-sided pneumothorax E. Lung abscesses as a result of E. Tuberculoma of the right lung aspiration most frequently occur in the posterior segments of the 417. A 35 y.o. patient was admitted to upper lobes or the superior the local hospital a week after a segments of the lower lobes road accident with clinical picture of clotted hemothorax. What is the 415. Which statements, most appropriate treatment tactic for concerning investigations of prevention of acute pleural patients with lung abscesses, are empyema? TRUE? 1). A complete white blood A. Surgical removal of clotted cell count with differential may hemothorax reveal leukocytosis and a left shift; B. Treatment by pleural punctions 2). CT scan is the initiate step in C. Complex conservative therapy evaluation of the patient with lung D. Passive drainage of pleural abscess; 3). On chest X-ray lung cavity abscess is an irregularly shaped E. Active drainage of pleural cavity cavity with an air-fluid level inside; 4). Bronchoscopy has a very high 418. A 35 y.o. woman was admitted to sensitivity and specificity in thoracic surgery department with reaviling of lung abscesses; 5). fever up to 40 0C, onset of pain in Peripheral lung abscesses with the side caused by deep breathing, pleural contact or included inside a cough with considerable quantity of lung consolidation are detectable purulent sputum and blood with bad using lung ultrasonography at the smell. What is the most likely bedside. Choose the correct diagnosis? combination: A. Complication of liver A. 1,2,3 B. 1,3,5 B. Bronchiectatic disease C. 1,4,5 C. Pulmonary tuberculosis D. 2,3,4 D. Actinomycosis of lungs E. 2,4,5 E. Abscess of the lung

416. X-ray pattern of thorax organs 419. A 52 year old patient complains revealed a large intensive about pain in the right part of her inhomogeneous opacity with chest, dyspnea, cough with a lot of 64 foul-smelling albuminoid sputum in form of "meat slops". Objectively: the patient's condition is grave, cyanosis is present, breathing rate is 31/min, percussion sound above the right lung is shortened, auscultation revealed different rales. What is the most probable diagnosis? A. Chronic pneumonia

B. Pleura empyema C. Lung abscess D. Lung gangrene E. Multiple bronchiectasis

65 1. Overal mortality rate in case of acute D. Appendicular abscess; is: E. Peritonitis. A. 10-20%; B. 5-10%; 7. A 34-year-old female patient suffered from C. 0,2-0,8%; abdominal pain week ago; no other D. 1-5%; gastrointestinal problems were noted. On E. 25%. clinical examination, a mass of about 6 cm was palpable in the right lower quadrant, 2. Name the destructive form of appendicitis. appeared hard, not reducible and fixed to A. Appendicular colic; the parietal muscle. CBC: leucocyts – B. Superficial; 7,5*10 9/l, ESR – 24 mm/hr. Temperature C. hydrops; 37,4˚C. Triple antibiotic therapy with D. Phlegmonous; cefotaxime, amikacin and tinidazole was E. Catarrhal appendicitis. very effective. After 10 days no mass in abdominal cavity was palpated. What time 3. Koher sign is: term is optimal to perform appendectomy? A. Migration of the pain from the A. 1 week; epigastrium to the right lower B. 2 weeks; quadrant; C. 3 month; B. Pain in the right lower quadrant; D. 1 year; C. One time vomiting; E. 2 years. D. Pain in the right upper quadrant; E. Pain in the epigastrium. 8. What instrumental method of examination is the most efficient in case of portal 4. In cases of appendicular infiltration is pyelophlebitis? indicated: A. Plain abdominal film; A. Laparoscopic appendectomy; B. Barium meal; B. Concervative treatment; C. US; C. Open appendectomy; D. Termography; D. Draining; E. Doppler ultrasound. E. Laparotomy. 9. The complications of acute appendicitis 5. In cases of appendicular abscess is are all, except: indicated: A. Appendicular infiltration; A. Laparoscopic appendectomy; B. Appendicular abscess; B. Concervative treatment; C. Enzyme peritonitis; C. Open appendectomy; D. Pyelophlebitis; D. Draining, if possible - appendectomy; E. Sepsis. E. Ileo-cecal resection. 10.A 63-year-old male patient in reasonably 6. A 34-year-old female patient suffered from good health suddenly suffered from fever abdominal pain week ago; no other (>38°C) and a painful right iliac fossa gastrointestinal problems were noted. On tumefaction; no other gastrointestinal clinical examination, a mass of about 6 cm problems were noted. On clinical was palpable in the right lower quadrant, examination, a mass of about 5 cm was appeared hard, not reducible and fixed to the palpable in correspondence of right iliac parietal muscle. CBC: leucocyts – 7,5*10 9/l, fossa and appeared hard, not reducible and ESR – 24 mm/hr. Temperature 37,4˚C. Make fixed to the parietal muscle. Laboratory the diagnosis? data showed leucocytosis, shift to the left A. Appendicular colic; and elevated erythrocyte sedimentation B. Appendicular hydrops; rate (74 mm/hr) as pathological findings. C. Appendicular infiltration; Abdominal ultrasound examination 66 evidenced a fluid in the right lower and a gangrenous appendix is quadrant, with heterogenic echotexture and removed. The stump is inverted. Why a thickening of the ileocecal tract. does acute appendicitis in elderly Abdominal CT, confirmed the presence of patients and in children have a worse a complex, predominantly cystic, mass of prognosis? large size (6×8 cm) with heterogeneous, A. The appendix is retrocecal. mainly peripheral enhancement, the B. The appendix is in the preileal adjacent cecum had its wall thickened and position. it was not possible to differentiate the C. The appendix is in the pelvic position. appendix separately from the mass, D. The omentum and peritoneal cavity homolateral inguinal reactive appear to be less efficient in lymphadenopathy was also present. The localizing the disease in these age patient failed to respond to the initial groups. conservative management, which consisted E. The appendix is longer in these age of intravenous fluids and triple antibiotic groups. therapy with cefotaxime, gentamicin and metronidazole, without any improvement 13. A 12-year-old boy complains of pain in of pain and fever. At a further ultrasound the lower abdomen (mainly on the right examination, the mass appeared not side). Symptoms commenced 12 hours modified. Make the diagnosis? before admission. He had noted anorexia A. Appendicular infiltration; during this period. Examination revealed B. Appendicular abscess; tenderness in the right iliac fossa, which C. Peritonitis; was maximal 1 cm below McBurney’s D. Appendicular colic; point. In appendicitis, where does the E. Phlegmonous appendicitis; pain frequently commence? A. In the umbilical region and then 11. A 17-year-old female model presents to moves to the right iliac fossa the emergency room with a 1-day history B. In the back and moves to the right of lower abdominal pain. On examination iliac fossa she is most tender in the right lower C. In the rectal region and moves to the quadrant (RLQ) and also has pelvic right iliac fossa tenderness. White blood cell (WBC) D. In the right iliac fossa and remains count is 13x10 9/l and temperature is there 38,2°C. A provisional diagnosis of E. In the right flank uncomplicated appendicitis is made and laparoscopic appendectomy is offered. 14. On examination, patients presenting with Regarding laparoscopic appendectomy appendicitis typically show maximal which of the following is TRUE? tenderness over which of the following? A. It can be performed safely with A. Inguinal region minimal morbidity compared to B. Immediately above the umbilicus open technique. C. At a point between the outer one-third and B. Length of hospital stay is longer than inner two-thirds of a line between the with open technique. umbilicus and the anterior superior iliac C. Posthospital recovery can be shorter spine in uncomplicated appendicitis. D. At a point between the outer two-thirds D. Return to full feeding is less than and inner one-third of a line between the with open technique. umbilicus and the anterior superior iliac E. Wound complication rate is greater spine with open technique. E. At the midpoint of a line between the umbilicus and the anterior superior iliac 12. A79-year-old man has had abdominal pain spine for 4 days. An operation is performed, 67 15. A 29-year-old woman presents to her made. What morphological form acute physician’s office with pain in the right appendicitis we must suspect? iliac fossa. Examination reveals A. Gangrenous; tenderness in this region. Her last B. Superficial; was 2 weeks previously. C. Appendix hydrops; CBC: leukocyte count – 7,2x10 9/l, RBC D. Phlegmonous; – 2,9x10 12 /l, Hb – 105 g/l. Make the E. Appendicular colic. diagnosis. A. Acute superficial appendicitis 19. A 20-year-old man has undergone B. Ovarian apoplexy appendectomy for perforated C. Renal colic appendicitis with generalized peritonitis. D. Acute destructive appendicitis Seven days postoperatively, his E. temperature continues to spike to 39,5°C despite antibiotic therapy with 16. A 28-year-old man is admitted to the ampicillin, gentamicin, and emergency department complaining of metronidazole. Abdominal CT scan pain in the umbilical region that moves reveals a large pelvic abscess. Soon to the right iliac fossa. Which is a afterward, he has bleeding from the corroborative sign of acute appendicitis? mouth and nose with increasing oozing A. in the right side with from the surgical wound and all pressure on the left (Rovsing)sign intravenous puncture sites. What is the B. Increase of pain with testiculalr most likely diagnosis? elevation A. Anaphylactoid reaction to intravenous C. Relief of pain in lower abdomen with dye; extension of thigh B. Antibiotic-induced coagulopathy D. Relief of pain in lower abdomen with C. Disseminated intravascular coagulation internal rotation of right thigh D. ; E. Hyperesthesia in the right lower E. Congenital bleeding disorder. abdomen 20. A 20-year-old man has undergone 17. At open operation a normal appendix is appendectomy for perforated found, no other pathology in abdominal appendicitis with generalized peritonitis. cavity. What is the most common Seven days postoperatively, his procedure a surgeon should do if he temperature continues to spike to 39,5°C finds a normal appendix? despite antibiotic therapy with A. Evaluate the pelvis for tuboovarian ampicillin, gentamicin, and abscess pelvic inflammatory disease, metronidazole. Abdominal CT scan malignancy or etopic pregnancy reveals a large pelvic abscess. Soon B. Removal of appendix afterward, he has bleeding from the C. Evaluate the terminal ileum and mouth and nose with increasing oozing cecum for signs of regional or from the surgical wound and all bacterial intravenous puncture sites. What was a D. Evaluate the upper abdomen for trigger for coagulopathy? or perforated duodenal A. Sepsis; ulcer B. Antibioticotherapy; E. Evaluate for Meckel’s diverticulum C. Congenital disoder; D. Proteolysis; E. Operation. 18. M., 68-year old man, 14 hours ago appeared continuous pain in the RLQ, 2 21. All listed below locations of appendix hours ago pain decreased significantly. concerning cecum are correct, EXCEPT: The diagnosis of acute appendicitis was A. medial 68 B. retrocecal C. subhepatic 26. Typically, in the case of acute D. pelvic appendicitis, symptoms of the disease E. intermediate include: 1) rebound tenderness (Blumberg’s sign) in the RLQ, 2) 22. Appendicular artery is the branch of: hematuria, 3) pain in the RLQ, 4) A. a. ileocolica involuntary guarding, 5) normal body B. a. mesenterica inferior temperature. Choose the RIGHT C. a. hepatica communis combination. D. celiac trunck A. 1, 3, 4 E. a. iliaca interna B. 1, 2, 4 C. 1, 2, 5 23. What is the Clado ligament? D. 2, 3, 5 A. lig. appendico-ovaricum E. 3, 4, 5 B. lig. appendico-cecalis C. lig. appendico-transversum 27. Which sign reflects pelvic location of the D. lig. circularis appendicis appendix? E. lig. longitudinalis appendicis A. Rovsing’s sign B. Blumberg’s sign 24. Classification proposed by V.I. Kolesov C. psoas sign in 1972, is based on clinical course of D. obturator sign disease and includes FOUR items. They E. Koher’s sign are all, EXCEPT: A. Appendicular colic 28. You will perform all listed laboratory B. Simple appendicitis: superficial, tests to make the diagnosis and catarrhal differential diagnosis of acute C. Destructive appendicitis: appendicitis, EXCEPT? phlegmonous, gangrenous, A. white blood cell (WBC) count perforating B. urinanalysis D. Complications of acute C. serum liver enzymes and amylase appendicitis: appendicular levels infiltrate, appendicular abscess, D. serum HCG (human chorionic diffuse peritonitis gonadotropin) level in women of E. Purulent complications: abscesses, childbearing age sepsis E. serum protein

25. Which statement, concerning acute 29. Which statement, concerning acute appendicitis, is NOT TRUE? appendicitis, is WRONG? A. The tip of the appendix can be located A. Perforation of the appendix is anywhere in the right lower quadrant accompanied with a sharp of the abdomen or pelvis abdominal pain B. Appendicitis occurs rarely in very B. Acute appendicitis with pelvic young children and elderly persons. location of the appendix is always C. Luminal obstruction leads to secretion characterized by clear clinical of mucus and fluid with the rise in picture and typical course luminal pressure causing ischemia of C. Often the patients, with acute the wall of appendix appendicitis with pelvic location of D. In general, patients with appendicitis the appendix, have diarrhea and/or report nausea and loss of appetite dysuria E. Migration of the pain from epigastric or D. White blood cells (WBC) are paraumbilical region to right lower usually elevated in patients with quadrant is a sign of patient’s recovery acute appendicitis 69 E. Infiltrate is being formed on 3-5 B. Oral cholecystogram days of the disease C. Intravenous cholangiogram D. Abdominal x-ray 30. On the 10 th day after the admission to the E. ERCP surgical department patient develops suppuration of appendicular infiltrate. In 34. A 48-year-old woman is admitted to the this case, clinical picture will include all hospital with severe abdominal pain, symptoms, EXCEPT: tenderness in the right hypochondrium, A. pain in the region of abdominal and a WBC count of 12x10 9/l. Acute mass cholecystitis is established. After B. fatigue diagnosis, cholecystectomy should be C. headache performed within which of the following? D. hectic fever A. 20–60 minutes E. breathlessness B. The first 1-2 days following hospital 31. In attempting to minimize complications admission during cholecystectomy, the surgeon C. 8 days defines the triangle of Calot during the D. 3 weeks operation. The boundaries of the triangle E. 3 months of Calot (modified) are the common hepatic duct medially, the cystic duct 35. A 60-year-old diabetic man is admitted to inferiorly, and the liver superiorly. Which the hospital with a diagnosis of acute structure courses through this triangle ? cholecystitis. The WBC count is 28 A. Left hepatic artery x10 9/l, and a plain film of the abdomen B. Right renal vein and CT scan show evidence of intramural C. Right hepatic artery gas in the gallbladder. What is the most D. Cystic artery likely diagnosis? E. Superior mesenteric vein A. B. Acalculous cholecystitis 32. A 65-year-old woman is admitted with C. Cholangiohepatitis RUQ pain radiating to the right shoulder, D. Sclerosing cholangitis accompanied by nausea and vomiting. E. Emphysematous gallbladder Examination reveals tenderness in the RUQ and a positive Murphy’s sign. A 36. A 60-year-old woman is recovering from diagnosis of acute (uncomplicated) a major pelvic cancer operation and cholecystitis is made. What is the most develops severe abdominal pain and likely finding? sepsis. Acute cholecystitis is established, A. Serum bilirubin levels may be elevated laparotomy is performed. The gallbladder B. Cholelithiasis is present in 90% of all is severely inflamed and removed. There cases is no evidence of gallbladder stones C. Bacteria are rarely found at operation (acalculous cholecystitis). D. An elevated amylase level excludes this Cholecystectomy is performed. Which is diagnosis true of acalculous cholecystitis? E. A contracted gallbladder is noted on A. It is usually associated with stones in ultrasound the CBD. B. It occurs in 80–90% of cases of 33. A 38-year-old male lawyer develops cholecystitis. abdominal pain after having a fatty meal. C. It has a more favorable prognosis than Examination reveals tenderness in the right calculous cholecystitis. hypochondrium and a positive Murphy’s D. It is increased in frequency after sign. Which test is most likely to reveal trauma or operation. acute cholecystitis? E. It is characterized on US scan by A. US of the abdomen inlarged cystic duct. 70 C. Cholecystohepatic duct of Linka 37. Following recovery in the hospital from a D. Cholecystohepatic duct of Calot fracture of the femur, a 45-year-old E. Cholecystohepatic duct of Koher female patient develops RUQ abdominal pain and fever. She has tenderness in the 41. Different hormones regulate function of right subcostal region. There is evidence the gallbladder. Choose all of them: 1). of progressive sepsis and hemodynamic Vasoactive intestinal polypeptide; 2). instability. The WBC count is 24 x10 9/l. Cholecystokinin; 3). Thyroxine; 4). A bedside sonogram confirms the Lipase; 5). Motilin. presence of acalculous cholecystitis. A. 1, 2, 3 What should treatment involve? B. 1, 2, 5 A. Intravenous antibiotics alone C. 1, 3, 4 B. ERCP D. 3, 4, 5 C. Percutaneous drainage of the E. 2, 3, 4 gallbladder D. Urgent cholecystectomy 42. What is the most common symptom E. Elective cholecystectomy after 3 related to acute cholecystitis? months A. RUQ abdominal pain B. Jaundice 38. Following recovery in the hospital from a C. Fever fracture of the femur, a 75-year-old D. Nausea and vomiting nursing home female patient develops E. Weight loss RUQ abdominal pain and fever. She has tenderness in the right subcostal region. 43. Which condition does NOT predispose to There is evidence of progressive sepsis gallstone formation? and hemodynamic instability. The WBC A. Concentrated bile count is 24 x10 9/l. A bedside sonogram B. Increased gallbladder motility confirms the presence of acalculous C. Female sex cholecystitis. What should treatment D. Obesity involve? E. Previous truncal vagotomy A. Intravenous antibiotics alone B. Percutaneous drainage of the 44. Patient, who suffers from acute gallbladder cholecystitis, can have all listed below C. ERCP symptoms, EXCEPT: D. Urgent cholecystectomy A. Nausea and vomiting E. Elective cholecystectomy after 3 B. Anorexia months C. Fever D. Arterial hypotention 39. The gallbladder consists of following E. RUQ pain parts, EXCEPT: A. Fundus 45. Physical examination in patient with acute B. Body cholecystitis, can reveale all signs, EXCEPT: C. Infundibulum A. Guarding and rebound tenderness D. Neck in the RUQ E. Cardia B. Positive Murphy’s sign C. Painful mass in the RUQ 40. Sometimes small accessory may D. Positive Blumberg’s in the RUQ drain directly into the gallbladder. The E. Absence of liver dullness name of this duct is? A. Cholecystohepatic duct of Luschka 46. Which statement concerning imaging B. Cholecystohepatic duct of studies in patients with acute cholecystitis is Laschtuvka WRONG? 71 A. The most important value of plain C. Increased gallbladder motility abdominal film is exclusion of other D. Decreased liver function abdominal pathology (perforated E. Hyperglycemia peptic ulcer) B. Ultrasound of the abdomen is an 52. What type of bile stones didn’t exist? effective method for identifying A. in case of acute B. Pigment cholecystitis C. Mixed C. Gallstones and bile appear nearly D. Calcium isodense on CT E. Protein D. Cholangiography is the most accurate method the extrahepatic biliary tree 53. At abdominal examination: patient is visualization asked to breathe out and then physician gently E. All patients with acute cholecystitis place the hand below the costal margin on the should undergo MRI right side at the mid-clavicular line, the patient is then instructed to breathe in. If the 47. The most efficient noninvasive patient stops breathing in the test is instrumental method of examination in case considered positive. Name this test? of acute cholecystitis is: A. Murphy’s sign A. Oral cholecystography B. Kehr’s sign B. Ultrasonography C. Blumberg’s sign C. Termography D. Rovsing’s sign D. GIT X-ray examination E. Psoas sign E. IV cholangiography 54. Patient: 44 years female. Three weeks ago 48. Which signs are positive in patients with felt acute pain in right upper quadrant acute cholecystitis? after fatty food, spasmolitics were A. Ortner, Kehr, Murphy efficient. Complaints on admission: dull B. Kocher, Sitkovski, Rovzing pain in right upper quadrant. Examination: C. Pasternatski temperature 36,8ºC, soft mass palpated in D. Mayo-Robson Kehr point. CBC: WBC – 6,8x10 9/l. US: E. Cullen, Grey Turner very large gallbladder with thin wall, stone 15 mm in diameter obstructing it, 49. Normal diameter of CBD is: 7 mm in diameter. A. up to 25 mm Make the diagnosis? B. up to 15 mm F. Acute cholecystitis; C. up to 10 mm G. Biliary pancreatitis; D. up to 20 mm H. Gallbladder hydropsia; E. up to 5 mm I. Cholangitis; J. Mechanical jaundice. 50. Normal diameter of cystic duct is: A. 2 mm 55. The management in case of acute B. 3-4 mm cholecystitis complicated with C. 10-12 mm choledocholithiasis should be? D. 20 mm A. Conventional cholecystectomy; E. 8-9 mm B. Laparoscopic cholecystectomy; C. Cholecystostomy; 51. Choose the reasons which are responsible D. ECRP, sphincterotomy, for gallstones formation during the cholecystectomy; pregnancy: E. Concervative treatment. A. Hyperestrogenemia, B. Decraesed duodenal motility 72 56. To what complication can lead acute obstructive B. Complete the laparoscopic cholecystitis? cholecystectomy and repeat an A. Cancer of pancreatic gland; ultrasound postoperatively. Observe B. Pleural empiema; the patient if no CBD stone is C. Gallbladder empiema; visualized D. Perforative duodenal ulcer; C. Perform a CBD exploration either E. Acute bowle obstruction. laparoscopically or open along with a cholecystectomy 57. Which complication of acute cholecystitis D. Complete the laparoscopic develops, when the patient have jaundice, cholecystectomy, no further treatment high temperature (>38ºC), pain in upper is necessary right quadrant, swetting: E. Complete the laparoscopic A. Acute pancreatitis; cholecystectomy and plan for a B. Bile duct fistula; postoperative hydroxy iminodiacetic C. Gallbladder empiema; acid (HIDA) scan

Duodenal ulcer; D. 60. A 42-year-old accountant presents with E. Cholangitis. recurrent RUQ pain of 3-year duration. He had undergone a laparoscopic 58. An 85-year-old man is brought to the cholecystectomy 2-years ago for hospital with a 2-day history of nausea presumed symptomatic cholelithiasis, but and vomiting. He has not passed gas or the pain persisted. An upper GI endoscopy moved his bowels for the last 5 days. is normal. A sonogram and CT scan of the Abdominal films show dilated small abdomen are normal. An ERCP is bowel, no air in the rectum and air in the performed, and the pressure in the CBD is biliary tree. Which of the following 45-cm saline (normal bile duct pressure is statements is TRUE? 10–18- cm saline). What is the most likely A. Air in the biliary tree associated with diagnosis? small- suggests a A. Acalculous cholecystitis diagnosis of gallstone ileus; B. Emphysematous cholecystitis B. An enterotomy should be distal to the C. site of obstruction and the stone D. Cancer of the gallbladder should be removed; E. Myasthenia gravis C Gallstone ileus is more common in the young adults; 61. A 42-year-old accountant presents with D. Cholecystectomy is contraindicated; recurrent RUQ pain of 3-year duration. He E. Small-bowel obstruction usually occurs had undergone a laparoscopic in the distal jejunum. cholecystectomy 2-years ago for presumed symptomatic cholelithiasis, but the pain 59. An intraoperative cholangiogram is persisted. An upper GI endoscopy is normal. performed during an elective laparoscopic A sonogram and CT scan of the abdomen are cholecystectomy on a 30-year-old woman. normal. An ERCP is performed, and the She has no previous surgical history. pressure in the CBD is 45-cm saline (normal There is a 0.8-cm filling defect in the bile duct pressure is 10–18- cm saline). distal common bile duct (CBD). The Choose the best treatment option: surgeon should: A. Calcium channel blockers, if not A. Complete the laparoscopic effective an endoscopic cholecystectomy and check liver sphincterotomy function tests (LFTs) postoperatively. B. Antibiotics, if not effective an If they are normal, no further endoscopic sphincterotomy treatment is needed C. Analgesics, if not effective an endoscopic sphincterotomy 73 D. β-blockers, if not effective an E. Urobilin in urine endoscopic sphincterotomy E. No treatment 65. An 70-year-old male presents with a clinical diagnosis of acute cholangitis. 62. A 43-year-old woman undergoes open Which organism is most likely involved cholecystectomy. Intraoperative in the pathogenesis of ascending cholangiogram revealed multiple stones in cholangitis? the CBD. Exploration of the CBD was A. Clonorchis sinensis performed to extract gallstones. The CBD B. Escherichia coli was drained with a #18 T-tube. After 10 C. Salmonella days, a T-tube cholangiogram reveals a D. Staphylococcus aureus retained CBD stone. This should be E. Clostridia treated by which of the following? A. Laparotomy and CBD exploration 66. Following admission to the hospital for B. Subcutaneous heparinization intestinal obstruction, a 48-year-old C. Antibiotic therapy for 6 months and woman states that she previously had then reevaluation undergone cholecystectomy and D. ERCP, endoscopic sphincterotomy, choledochoduodenostomy. The most CBD exploration likely indication for the performance of E. Ultrasound crushing of the CBD stone the choledochoduodenostomy was: A. Hepatic metastasis were present 63. A 62-year-old woman who underwent B. Multiple stones were present in the cholecystectomy and gallbladder at the previous operation choledochoduodenostomy (CBD C. Multiple stones were present in the duodenal ) 5 years CBD at the previous operation previously is admitted to the hospital D. The common hepatic duct had a with a 3-day history of upper abdominal stricture pain, chills, fever, and dark urine. Make E. The small intestine was occluded the diagnosis: A. 67. In attempting to minimize complications B. Cancer of gallblader during cholecystectomy, the surgeon C. Acute defines the triangle of Calot during the D. Acute biliary pancreatitis operation. The boundaries of the triangle E. of Calot (modified) are the common hepatic duct medially, the cystic duct 64. A 62-year-old woman who underwent inferiorly, and the liver superiorly. cholecystectomy and Which structure courses through this choledochoduodenostomy (CBD duodenal triangle? anastomosis) 5 years previously is admitted to A. Left hepatic artery the hospital with a 3-day history of upper B. Right renal vein abdominal pain, chills, fever, and dark urine. C. Right hepatic artery What laboratory finding that supports D. Cystic artery diagnosis ascending cholangitis? E. Superior mesenteric vein A. Amylase elevation with normal findings on liver studies 68. A 55-year-old white female undergoes a B. Alkaline phosphatase elevation with laparoscopic cholecystectomy for normal or elevated normal bilirubin symptomatic cholelithiasis. The operation levels went well, and the patient was discharged C. Elevated serum glutamic home. One week later, she comes to your oxaloacetictransaminase (SGOT) office for a routine postoperative follow- levels up. The final pathology report shows an D. Altered urea/creatinine ratio incidental finding of a gallbladder 74 carcinoma confined to the mucosa. In Stricture of the CBD. Choose the correct further advising the patient, you should combination. inform her that A. 1, 2, 3 A. She should undergo radiation therapy B. 1, 3, 4 B She should undergo right hepatectomy C. 2, 4, 5 to remove locally infiltrating disease D. 1, 3, 5 C. She should undergo regional E. 2, 4, 5 lymphadenectomy D. She requires systemic chemotherapy 73. Which factors play important role in the E. She does not require any further pathogenesis of acute cholangitis? 1. Biliary therapy tract obstruction; 2. Elevated intraluminal pressure; 3. Decreased bile production by 69. Acute cholangitis is characterized by hepatocytes 4. Decreased serotonin serum three symptoms known as Charcot’s level; 5. Infection of the bile. Choose the triad. They are: 1). Weight loss; 2). correct combination. RUQ abdominal pain; 3). Jaundice; 4). A. 1, 2, 3 Nausea and vomiting; 5). Fever. Choose B. 2, 3, 4 the correct combination. C. 3, 4, 5 A. 1, 2, 3 D. 1, 2, 5 B. 1, 3, 4 E. 1, 4, 5 C. 1, 4, 5 D. 2, 3, 5 74. Which condition is NOT associated with E. 2, 4, 5 higher morbidity and mortality rate in patients with acute cholangitis: 70. Reynolds pentad known as symptoms in A. Hypotension patients with severe acute cholangitis. B. Acute renal failure Which symptom do NOT belong to C. Liver abscess Reynold’s pentad? D. Liver A. Altered mental status E. Age less than 50 years B. Arterial hypotension C. Jaundice 75. Which symptom together with Charcot's D. Nausea and vomiting triad is associated with acute cholangitis: E. Fever A. Coffee round vomitus B. Pruritus 71. Cholangiography is the most accurate and C. Hypertention sensitive method of biliary tree D. Diarrhea visualization. Diagnostic cholangiogram E. Constipation can be performed in different ways. Name them? 1). Percutaneously; 2). 76. Which of the following DOESN’T Endoscopically; 3). Intraoperatively; 5). increase the risk of cholangitis: Intravenous. Choose the most complete A. CBD stones confirmed on US answer. B. Recent cholecystectomy A. 1, 2, 3 C. Recent ERCP B. 1, 3, 4 D. History of cholangitis in the past C. 1, 4, 5 medical history D. All are correct E. Recent appendectomy E. 2, 4, 5 77. In patients with cholangitis at physical 72. Acute cholangitis is a bacterial infection examination you can reveal the following: 1. and most commonly is associated with: 1. RUQ tenderness; 2. Fever; 3. Mental status Gallstones; 2. Acute ; 3. Malignant changes; 4. Hypotension; 5. Tachycardia. tumor of CBD; 4. Truncal vagotomy; 5. Choose the correct combination. 75 A. 1, 2, 3 C. Cholangitis B. 2, 3, 4 D. Cholecystic-enteric fistula C. 3, 4, 5 E. Gallbladder hydropsia D. All are correct E. 1, 4, 5 83. Which disorder can cause gallbladder mucocele? 1. Impacted stone in the 78. Which statement, regarding acute gallbladder neck or cystic duct; 2. cholansitis, is NOT true? Spontaneously resolved acute cholecystitis; 3. A. Partial obstruction is associated Tumors of the CBD; 4. Extrinsic compression with a higher rate of infection than of cystic duct by lymph nodes; 5. Ascending complete obstruction cholangitis. Choose the correct combination. B. The bile is normally sterile A. 1, 2, 3 C. In case of cholangitis obstruction B. 2, 3, 5 of the common bile duct occurs C. 3, 4, 5 secondary to ascending infection D. 1, 2, 4 D. The male-to-female ratio is equal E. 1, 4, 5 in cholangitis E. The median age at presentation is 84. Which statement concerning gallbladder between 50 and 60 years. hydrops is WRONG? A. Acute inflammation is absent 79. Which tumor does NOT cause B. A large, painful gallbladder is found cholangitis? on physical examination A. Pancreatic cancer C. Laboratory test are normal B. Cholangiocarcinoma D. Ultrasonography of the RUQ shows an C. Ampullary cancer impacted stone in the neck of an D. Porta hepatis tumors enlarged gallbladder E. Hepatocellular carcinoma E. Intraoperatively, the aspirate from the gallbladder is clear fluid (white bile) 80. Which disease is NOT responsible for the development of cholangitis? 85. The gallbladder hydrops may develop all A. Strictures or stenosis of ampulla Vateri complication, listed below EXCEPT: B. Endoscopic manipulation of the CBD A. Empyema of the gallbladder C. AIDS cholangiopathy B. Perforation of the gallbladder D. Ascaris lumbricoides infections C. Gastric outlet obstruction E. Later D. Cholecystenteric fistula E. GIT bleeding 81. What changes in laboratory tests can be find in patients with acute cholangitis? 1. 86. Calculous cholecystitis may develop Elevated bilirubin; 2. Elevated alkaline biliodigestive fistula, which can cause phosphatase; 3. Elevated C-reactive protein; gallstone ileus. In this case fistula is a 4. Elevated ESR; 5. Leukocytosis connection between gallbladder and..? 1. A. All are present Duodenum; 2. Sigmoid colon; 3. Transversus B. 1, 2, 3 colon; 4. Stomach; 5. Bladder. Choose the C. 1, 3, 5 correct combination. D. 1, 2, 4 A. 1, 2, 3 E. 2, 4, 5 B. 1, 3, 4 C. 3, 4, 5 82. On X-ray film air in the biliary tree was D. 1, 2, 5 detected, which condition does NOT cause E. 2, 4, 5 this? A. Recent papillotomy 87. What hormone produces β-cells of B. Emphysematous cholecystitis pancreatic gland: 76 A. Somatostatin; E. CT. B. Somatotropin; C. Insulin; 93. A 60-year-old alcoholic is admitted to the D. Glucagon; hospital with a diagnosis of acute E. Pancreozymin. pancreatitis. Upon admission, his white blood cell (WBC) count is 21x10 9/l. His 88. Which ethiological factors causes acute pancreatitis lipase, lactate dehydrogenase and aspartate most oftenly? aminotransferase are elevated, blood A. Abdominal trauma, alimentary factor; glucose is 10 mmol/L. Which of the B. Gallstones, alcohol; following is TRUE? C. Hypercalcemia, hyperlipidemia; A. This patient is expected to have a D. Drugs, toxins; mortality rate of less than 5% E. ERCP, sphincter of Oddi disfunction. B. The patient’s lipase level is not important indication of prognosis 89. Which thesis is correct according to C. This patient requires immediate surgery pathogenesis of acute pancreatitis? D. A venous blood gas would be helpful in A. Intrapancreatic enzyme’s activation; assessing the severity of illness in this B. Pancreatic gland autolisys; patient C. Neutrophils chemoactivation, E. A serum calcium level of 1.7 mmol/L infiltration and inflamation; on the second hospital day is a bad D. SIRS; prognostic sign E. All are correct. 94. A 40-year-old alcoholic male is admitted 90. A 52-year-old male, without alcohol with severe epigastric pain radiating to history, admitted to hospital with acute the back. Serum amylase level is pain in epigastrium, radiating to the reported as normal (fluoroscopic back. Amylase level elevated. Which method), but serum lipase is elevated. radiological findings will help The serum is noted to be milky in diagnosing acute pancreatitis? appearance. A diagnosis of pancreatitis A. Changes in liver on CT scans; is made. The serum amylase is normal B. Choledocholithiasis on US; because: C. Stomach shifted anteriorly on contrast A. The patient has chronic renal failure examination of GIT; B. The patient has hyperlipidemia D. Cloiber cups on plain abdominal film; C. The patient has alcoholic cirrhosis E. Air below diaphragm on plain D. The patient has abdominal film. E. The diagnosis of pancreatitis is incorrect 91. What is the volume of pancreatic juice produced in 24 hours? 95. A 57-year-old woman is admitted to the A. 200-500 ml; hospital with abdominal pain. She B. 100-200 ml; reports that she alcohol only at C. 500-1000 ml; social occasions. The amylase is D. 700 ml; elevated to 120 mmol/l. Which E. 2500 ml following x-ray finding would support a diagnosis of acute pancreatitis? 92. What instrumental examination should A. Hepatic lesion on CT scan be performed to diagnose necrotizing B. Gas in abdominal cavity pancreatitis? C. Dilated colon transversum A. US; D. Large loop of colon in the RUQ B. ECRP; E Irregular cutoff of the CBD on C. Plain abdominal film; cholangiogram D. Blood gases; 77 96. Following a motor vehicle accident a epigastrium with irradiation to the back, truck driver complains of severe which develops after alcohol consumption. abdominal pain. Serum amylase level is Acute pancreatitis was diagnosed. The patient markedly increased to 400 mmol/l. Grey was performed CT scan: pancreatic Turner’s sign is seen in the flanks. enlargement. What is the stage of acute Pancreatic trauma is suspected. Which pancreatitis according to CT picture? statement is true of pancreatic trauma? A. Grade A A. It is oftenly caused by blunt injuries B. Grade B B. It is usually an isolated single-organ C. Grade C injury D. Grade D C. It often requires a total E. Grade E pancreatectomy D. It may easily diagnosed at operation 101. Acute pancreatitis is most rearly seen in: E. It is proved by the mildly elevated A. > 60 years old women amylase level B. 21-45 years old men C. 45-60 years old women 97. The severity of pancreatitis can be D. 45-60 years old men estimated with: E. Children A. Ranson Criteria B. Alvarado scale 102. How covers pancreatic C. US examination results gland? D. SOFA scale A. From all sides E. CXR B. From anterior and posterior sides C. From anterior and inferior sides 98. Clinical staging (Atlanta consensus) of D. Covers only anterior surface of the gland acute pancreatitis consists of the E. Covers anterior and posterior sides only of following forms of the disease: 1. the head of the gland Pseudotumorous pancreatitis. 2. Mild acute pancreatitis. 3. Sterile 103. For clinical picture of acute pancreatitis pancreonecrosis. 4. Infected are typical all signs except: pancronecrosis. 5. Pseudocyst of the A. Continuous severe pain located in the pancreas. Select the correct combination epigastrium and/or left upper quadrant of answers: B. The pain irradiates to the back A. 2,3,4 C. Multiple vomiting B. 1,2,3,5 D. Paralytic ileus C. 3,4 E. Absence of liver dullness D. 2,3,4,5 E. All correct 104. In differential diagnosis of acute pancreatitis and acute bowel obstruction will 99. Enzyme toxemia in case of pancronecrosis be usefull: 1. CBC; 2. Liver enzymes serum is caused by: 1. Trypsin. 2. levels; 3. Serum amylase and lipase levels; 4. Phospholipase A2. 3. Chymotrypsin. 4. Plain abdominal film; 5. Serum electrolytes. Elastase. 5. Enterokinase. Select the Choose best combination. correct combination of answers: A. 1, 2 A. 1,4 B. 2, 3 B. 2,3,5 C. 3, 4 C. 1,2,3,4 D. 1, 5 D. 1,3,4,5 E. 2, 5 E. All correct 105. Choose changes in biochemical test 100. A 45-year-old male addmitted to the typical for patients with acute pancreatitis: 1. surgical department with severe pain in Hypertriglyceridemia; 2. Hyperglycemia; 3. 78 Hypercalcemia; 4. Hypotriglyceridemia; 5. D. ; Hypocalcemia. E. Cancer of the head of the pancreas. A. 1, 2, 5 B. 1, 3, 4 110. The early complications of acute C. 2, 3, 4 pancreatitis are all, except: D. 3, 4, 5 A. Pancreatic shock; E. 1, 2, 4 B. Acute respiratory distress syndrome; C. Enzyme peritonitis; 106. Most common cause of death in early D. Renal failure; acute pancreatitis is? E. . A. Renal failure B. Cardiac failure 111. When infection complications of acute C. Respiratory failure pancreatitis develops most oftenly? D. Uncontrolled coagulopathy A. On 7-th day of disease; E. Sepsis B. After 2 weeks; C. On 4-th day of disease; 107. Severity of acute pancreatitis correlate D. After 6 months; with levels of all of the following except E. On 2-nd day of disease. А. Glucose B. Amylase 112. Treatment of acute pancreatitis includes C. Transaminase all EXCEPT: D. Calcium A. Diurhetics; E. WBC level B. Antibiotics; C. Intravenous hydration; 108. A 60-year-old female treated in D. Analgesics; surgical department for acute pancreatitis. E. Nothing per os. On 30 day of treatment in epigastrium appeared mass 10 cm in diameter. CBC: 113. A 24-year-old college student recovers RBC – 3,7*10 12 /l, WBC – 10*10 9/l. US from a bout of severe pancreatitis. He examination: fluid collected near the head has mild epigastric discomfort, sensation of the pancreatic gland. What of bloating, and loss of appetite. complication of acute pancreatitis Examination reveals an epigastric developed in this case? fullness that on ultrasound is confirmed A. Postnecrotic pseudocyst; to be a pseudocyst. The swelling B. Retroperitoneal phlegmone; increases in size over a 3-week period of C. Enzyme peritonitis; observation to 15 cm in diameter. What D. Pancreatic abscess; should be the next step in management? E. Cancer of the head of the pancreas. A. Percutaneous drainage of the cyst B. Laparotomy and internal drainage of 109. A 45-year-old male treated in surgical the cyst department for acute pancreatitis during C. Excision of pseudocyst 28 days. He started to complain for the D. Total pancreatectomy temperature elevation to 39°C, E. Administration of pancreatic enzymes weakness, fatigue. CBC: RBC – 3,5*10 12 /l, WBC – 21*10 9/l. CT: 11 cm 114. A 42-year-old woman with a history of in diameter fluid collected near the head chronic alcoholism is admitted to the of the pancreatic gland. What hospital because of acute pancreatitis. complication of acute pancreatitis The bilirubin and amylase levels are in developed in this case? the normal range. An ultrasound reveals A. Postnecrotic pseudocyst; cholelithiasis (stones in the gallbladder, B. Retroperitoneal phlegmone; CBD 6 mm in diameter). The symptoms C. Enzyme peritonitis; 79 decreased on the fifth day after abdominal pain, nausea, vomiting. admission. What should she be advised? Body temperature 36,8°C. At palpation: A. To start on a low-fat diet at home defined tumor in the epigastrium. B To increase the fat content of her diet Laboratory tests: pancreatic amylase C. To undergo immediate normal. On the second day in the hospital cholecystectomy CT was held: 9 cm pseudocyst of D. To undergo cholecystectomy during pancreas was found. Which statement the same hospital stay as well as an regarding this patient is correct? assessment of her bile ducts А. Pseudocyst can cause compression of E That she will be discharged and should stomach and biliary ducts undergo elective cholecystectomy B. Spontaneous resorption never met after 3 months C. Pseudocyst can be seen only in case of acute pancreatitis 115. A 26-year-old woman with a known D. In this case pseudocyst doesn’t need history of chronic alcoholism is admitted any treatment to the hospital with severe abdominal E. Malignant degeneration occurs in 25% pain due to acute pancreatitis. The serum of cases, if untreated and urinary amylase levels are elevated. On the day following admission to the 119. Which drug is used to suppress the hospital, there is no improvement, and secretion of the pancreas? she has a mild cough and slight dyspnea. A. Aprotinin What is the most likely complication? B. Octreotide A. Pulmonary atelectasis C. Papaverine B. Bronchitis D. Imipenem/cilastatin C. Pulmonary embolus E. Acetaminophen D. Afferent loop syndrome E. Pneumonia 120. Which drug is used to inhibite proteases? 116. The most often cause of death in case of A. Aprotinin destructive pancreatitis is: B. Octreotide A. Pulmonary embolism C. Papaverine B. Mechanical jaundice D. Imipenem/cilastatin C. Peritonitis E. Acetaminophen D. Septic complications E. Renal failure 121. Which statement concerning medical treatment of patient with acute 117. 30-year-old man hospitalized with a pancreatitis on early phase (first 3-5 days) severe epigastric pain. During is WRONG? examination: hypoxemia, dehydration. A. Patients are kept “nothing per os” Laboratory tests: increased levels of B. Aggressive fluid resuscitation is amylase and lipase in the blood. CT critically important confirmed severe acute pancreatitis. C. Antibiotics should be used in any case Which antibiotic will reduce the risk of of acute pancreatitis infection? D. Imipenem/cilastatin is indicated to A. Ampicillin prevent infication in patients with B. Erythromicin pancreatic necrosis C. Gentamicin E. Early initiation of enteral nutrition is D. Ceftriaxon important in treatment and infection E. Imipenem/cilastatin prevention

118. 45-year-old man who abused alcohol, admitted to the hospital complaining of 80 122. Name the source of infection in patients with necrotizing pancreatitis? 126. Which sign is not associated with peptic A. Pulmonary infection ulcer perforation: B. Bacterial translocation from gut A. Sudden onset; C. Skin infection B. Positive Blumberg sign; D. Urinary infection C. Free air below diaphragm on plain E. Nosocomial infection abdominal film; D. Cloiberg caps; 123. A 44-year-old woman is admitted to the E. Intolerable abdominal pain. hospital because of acute pancreatitis. Biochemical test: bilirubin – 45 mcmol/l. 127. Which ethiological factors causes peptic ulcer An ultrasound reveals stones in disease most oftenly? gallbladder and CBD 14 mm in A. Abdominal trauma, alimentary factor; diameter. MRCP: 8 mm stone in the B. H. pylori, NSAID's; distal part of CBD, which is dilated to 14 C. H. pylori, hyperlipidemia; mm. What should she be the first step in D. Drugs, toxins; the treatment of patient? E. NSAID's, . A. Whipple procedure B Distal resection of the pancreas 128. A 30-year-old male is operated on for C. Sphincterotomy and stone extraction peptic ulcer perforation, in 2,5 hours D. Only medical treatment after the beginning of the disease. E Urgent cholecystectomy Which operation will be most efficient (radical operation for peptic ulcer)? 124. A 47-year-old man with a known history A. Simple closure and highly selective of chronic alcoholism is admitted to the vagotomy; hospital with severe abdominal pain due B. Simple closure with a Graham patch to acute pancreatitis. His general using omentum; condition worsens in first 48 hours. CBC: C. Simple closure; RBC – 4,4*10 12 /l, WBC – 11,2*10 9/l. On D. Antrumectomy; second series of CT scans rapid increase E. Stomach resection. in retroperitoneal fluid accumulation was found. Which complication of necrotizing 129. A 42-year-old male, with previous ulcer pancreatitis develops in this case? history and typical clinical picture of A. Enzyme peritonitis peptic ulcer perforation, on examination, B. Retroperitoneal bleeding in 4 hours after the beginning of the C. Pancreatic duct disruption disease, discomfort in right upper D. Retroperitoneal phlegmon quadrant, heart rate – 74/min., mild E. Pancreatic abscess abdominal wall muscles rigidity, negative Blumberg sign. Free air below 125. Choose the WRONG statement diaphragm on X-ray abdominal film. concerning pseudocyst definition: What is you diagnosis? A. Peripancreatic fluid collection A. Peptic ulcer recurrence; persisting for more than 4 weeks B. Acute cholecystitis; B. Pseudocyst lack an epithelial layer C. Covered peptic ulcer perforation; C. Most of them are filled with necrotic D. Chronic cholecystitis; debris E. Acute appendicitis (subhepatic D. Intervention (minimally invasive or location). conventional) should be performed in case of pseudocyst complications 130. Most oftenly perforated peptic ulcers are development located at: E. In all cases pseudocysts should be A. Posterior wall of antrum; treated with conventional surgery B. Cardiac part of stomach; 81 C. Fundus of stomach; A. It results in a lower incidence of D. Posterior wall of duodenal bulb; ulcer recurrence E. Anterior wall of duodenal bulb. B The complication rate is lower C. It reduces acid secretion to a greater 131. Penetrated ulcer can cause such extent complications: 1). Abdominal abscess; D. It benefits patients with antral ulcers the 2). Portal pyelophlebitis; 3). Stomach- most organ fistula; 4). Acute pancreatitis; 5). E. It includes removal of the ulcer Bleeding. A. 1, 2, 3; 135. A63-year-old woman is admitted to the B. 2, 3, 5; hospital with severe abdominal pain of 3- C. 3, 4, 5; hour duration. Abdominal examination D. 1, 3, 5; reveals board-like rigidity, guarding, and E. All. rebound tenderness. Her blood pressure is 90/50 mm Hg, pluse 110 bpm (beats per 132. A 45-year-old man complains of burning minute), and respiratory rate is 30 breaths epigastric pain that wakes him up at per minute. After a thorough history and night. The pain is relieved by eating or physical, and initiation of fluid using over-thecounter antacids and H2 resuscitation, what diagnostic study blockers. Diagnosis is best confirmed by should be performed? which of the following? A. Supine abdominal x-rays A. Urea breath test B. Upright chest x-ray B. Serum gastrin levels C. Gastrograffin swallow C. Barium meal examination D. Computerized axial tomography (CAT) D. Upper endoscopy scan of the abdomen E. Upper endoscopy and biopsy E. Abdominal sonogram

133. A 44-year-old dentist complains of 136. A frail elderly patient is found to have an burning epigastric pain that wakes him up anterior perforation of a duodenal ulcer. at night caused by a recurrent duodenal He has a recent history of nonsteroidal ulcer. He has shown considerable anti-inflammatory drug (NSAID) use and improvement following operative no previous history of peptic ulcer treatment by a truncal vagotomy and disease. A large amount of bilious fluid is pyloroplasty, 10 years prior to this found in the abdomen. What should be incident. Which is TRUE of truncal the next step? vagotomy? A. Lavage and omental patch closure of A. It is performed exclusively via the the ulcer thorax B. Lavage of the peritoneal cavity alone B. It can be performed in the neck C. Total gastrectomy C. If complete, it will result in increased D. Lavage, vagotomy, and acid secretion gastroenterostomy D. It requires a gastric drainage procedure E. Laser of the ulcer E. It has been abandoned as a method to treat ulcer disease. 137. Name 3 main signs of perforated peptic ulcer? 134. A 42-year-old executive has refractory A. Abdominal pain, multiple vomiting, chronic duodenal ulcer disease. His distention of abdominal cavity physican has suggested several surgical B. Increasing abdominal pain, meteorism, options. The patient has chosen a parietal peritonitis (highly selective) vagotomy instead of a C. Ulcer in past medical history, sudden truncal vagotomy and antrectomy abdominal pain, board-like rigidity because? 82 D. Ulcer in past medical history, pain in B. Gastric outlet obstruction; the epigastrium, which gradually increases, C. Penetration; peritonitis D. Ulcer malignization; E. Abdominal pain, pale and cold skin, low E. In all cases. BP 142. During endoscopy on the proximal 1/3 of 138. A 63-year-old male patient admitted to lesser curvature of stomach the large hospital with complaints of general ulcer (3x3 cm) with profuse arterial weakness, dizziness, temporary loss of bleeding was visualized. Name the consciousness. For last three years patient bleeding vessel. was complaining of pain in epigastric A. Right gastric artery; area, specially at night, heartburn. Medical B. Left gastric artery; examination was not performed. 2 weeks C. Left gastroepiploic artery; before hospitalization, patient complaint D. Right gastroepiploic artery; of intense pain in epigastrium. On the day E. Short gastric arteries. of hospitalization encountered severe weakness, nausea, dizziness, twice black 143. 53-year-old patient, alcoholic, stool with a rotten odor and twice lost of complaining for that consciousness. CBC: hemoglobin – 91 follows episodes of intense vomiting. g/l, RBC – 2,3x10 9/l. Make the diagnosis? During endoscopy: liner tears below the A. Bleeding from duodenal ulcer; gastroesophageal junction. Make the B. Stomach cancer complicated with diagnosis? bleeding; A. Peptic ulcer complicated with C. Acute pancreatitis; bleeding; D. Myocardial infarction; B. Mallory-Weiss syndrome; E. . C. Esophageal varices; D. Dieulafoy’s lesion; 139. 63-year-old patient admitted with massive E. Gastritis. vomiting with blood, which started suddenly. Previous history of viral 144. Name most often cause of GIT hepatitis. On examination: , bleeding? enlarged liver and spleen. What is the A. Peptic ulcer; cause of bleeding? B. Liver cirrhosis; A. Erosive gastroduodenitis; C. ; B. Peptic ulcer disease complicated with bleeding; D. Colon diseases; C. Esophageal varices; E. Esophagus diseases. D. Mallory-Weiss syndrome; E. Gastric cancer. 145. The main complication of GIT bleeding is? 140. Choose the most efficient diagnostic A. Renal insufficiency; method in case of peptic ulcer disease B. Liver failure; complicated with bleeding: C. CNS hypoxia; A. Abdominal plain film; D. Hypovolemic shock; B. Ultrasound; E. Centralization of circulation. C. CT scanning; D. Endoscopy; 146. What is the most efficient and safe E. . method of bleeding stress ulcers treatment? 141. In patient suffering from peptic ulcer A. Stomach resection; disease the risk of bleeding is highest in B. Sengstaken-Blackmore tube; case of: C. Combination of hemostatic and anti- A. Ulcer perforation; acid treatment; 83 D. Endoscopic coagulation; that stopped spontaneously. She did E. Gastrotomy and suturing of the not require transfusion. She had ingested vessels. large amounts of aspirin in the past 4 months to relieve the pain caused by 147. Peptic ulcer complicated with bleeding severe rheumatoid arthritis. Endoscopy is associated with such clinical signs: 1). confirms the presence of a duodenal Increase of the pain; 2). Coffee ground ulcer. A biopsy is done. What is the next vomitus; 3). Decrease of the pain; 4). step in the management of a duodenal Bradycardia; 5). ulcer associated with a positive biopsy A. 1, 2, 3; for H. pylori ? B. 2, 3, 5; A. H2 blockers C. 3, 4, 5; B. Bipolar electrocautery of the ulcer D. 1, 3, 4; C. Triple therapy E. All. D. Photocoagulation E. Elective surgery 148. A 68-year-old woman has been diagnosed with a benign ulcer on the 151. An 80-year-old grandfather gets greater curvature of her stomach, 5 cm admitted to the hospital for a UGI bleed. proximal to the antrum. After 3 months of He undergoes upper endoscopy and standard medical therapy, she continues to bleeding ulcer is visualized. Attempts at have guaiac positive stool, anemia, and endoscopic cauterization and abdominal pain with failure of the ulcer to epinephrine injection are unsuccessful at heal. Biopsies of the gastric ulcer have not stopping the bleeding. A previous identified a malignancy. The next step in attempt at angioembolization was also management is which of the following? unsuccessful. What is the next definitive A. Treatment of the anemia and repeat all step in therapy? studies in 6 weeks A. Elective surgery B. Endoscopy and bipolar electrocautery B. PPI IV or laser photocoagulation of the gastric C. Blood transfusion ulcer D. Repeated attempts at bipolar C. Admission of the patient for total electocanter parenteral nutrition (TPN), treatment of E. Emergency surgery anemia, and endoscopic therapy D. Surgical intervention, including partial 152. A 54-year-old man presents with a gastric resection massive UGI bleed. After resuscitation, E. Surgical intervention, including total endoscopy is performed. No esophageal gastrectomy varices, gastritis, or gastric ulcers are seen. After irrigation, a pinpoint lesion is 149. Ahealthy 75-year-old man bleeds from a seen near the gastro-esophageal junction. duodenal ucler. Medical management Make the diagnosis? and endoscopic measures fail to stop the A. Mallory-Weiss syndrome bleeding. What is the next step in B. Dieulafoy’s lesion management? C. Carcinoma A. Continued transfusion of 8 U of blood D. Gastro-intestinal stromal tumor B. Oversewing of the bleeding point, E. vagotomy. and pyloroplasty C. Oversewing of the bleeding point 153. A 54-year-old man presents with a D. Administration of norepinephrine massive UGI bleed. After resuscitation, E. Hepatic artery ligation endoscopy is performed. No esophageal varices, gastritis, or gastric ulcers are 150. A73-year-old woman is admitted to the seen. After irrigation, a pinpoint lesion is hospital with a mild UGI hemorrhage seen near the gastro-esophageal 84 junction.What can be said about this A. Intussusception; disease? B. Paralytic ileus; A. It is a carcinoid C. ; B. It is related to alcohol use D. Hernia incarceration; C. It is exclusively a mucosal lesion E. Spastic bowel obstruction. D. Surgery if first-line therapy E. Bleeding is from a submucosal vessel 159. For low large bowel obstruction are characteristic all signs except: 154. The most efficient noninvasive A. Slow beginning; diagnostic method for small bowel B. Abdominal distension; obstruction is: C. Cloiberg caps on plain abdominal A. Plain abdominal film; film; B. Laparoscopy; D. Absence of stool; C. Irrigography; E. Acute dehydration. D. Colonoscopy; E. Ultrasound examination. 160. Surgical treatment in case of acute bowel obstruction is indicated in cases: 1. 155. Medical therapy will be effective in cases Electrolyte disbalance. 2. Abdominal cramps. of bowel obstruction caused by: 3. Strangulation. 4. Obstruction has not 1.Volvulus; 2.Cancer of colon; 3.Paralytic resolved within 24-48 hours of concervative ileus; 4.Spastic bowel obstruction; treatment 5. Multiple air-fluid level on plain 5.Coprostasis. abdominal film. A. 1, 2, 3; A. 1, 2; B. 2, 3, 4; B. 3, 4; C. 3, 4, 5; C. 3, 5; D. 1, 2, 5; D. 4, 5; E. 1, 2, 4. E. 2, 3.

156. The signs of strangulation are: 1.Acute 161. A 42-year-old woman is admitted to the onset of disease; 2.Severe abdominal pain; emergency department with severe colicky 3.Slow beginning; 4.Dull pain; 5. Dilated pain, vomiting, and abdominal distention. small intestine loops with air/fluid levels on She has not passed stools or flatus for 48 plain abdominal film. hours. X-rays of the abdomen confirm the A. 4, 5; presence of smallbowel obstruction. What B. 1, 3, 4; is the most likely cause of small-bowel C. 3, 4, 5; obstruction in the patients? D. 1, 2, 5; A. Adhesions, inflammatory diseases; E. 2, 3, 5. B. Helminths, tumors; C. Groin hernia incarceration, adhesions; 157. The signs of obturation are: 1.Acute D. Gallstones, tumors; onset of disease; 2.Severe abdominal pain; E. Tumors, groin hernia. 3.Slow beginning; 4.Dull pain; 5. Dilated air- filled colon on plain abdominal film. 162. Most oftenly small bowel obstruction is A. 1, 2, 3; caused by: B. 3, 4, 5; A. Adhesions, inflammatory diseases; C. 1, 4, 5; B. Helmints, tumors; D. 2, 3, 5; C. Groin hernia incarceration, adhesions E. 1, 2. D. Gallstones, tumors; E. Tumors, groin hernia. 158. Patient with bowel obstruction is complaining for blood in the stool. What is 163. An elderly nursing home patient is you diagnosis? brought to the hospital with recent onset of 85 colicky abdominal pain, distension and 167. Name the scoring system which helps obstipation on examination, the abdomen surgeon to make the prognosis in case of is markedly distended and tympanitic. peritonitis? There is no marked tenderness. Plain A. Manheim index; abdominal x-ray shows a markedly B. APACHE II; distended loop located mainly in the right C. ONTARIO Score; upper quadrant. The likely diagnosis is: D. EuroSCORE; A. Small-bowel obstruction; E. SOFA. B. Chron’s disease; C. Gallstone ileus; 168. A 68-year-old woman is admitted with D. Mesenteric vascular occlusion; an acute surgical abdomen. After E. Sigmoid volvulus. resuscitation with IV crystalloids fluids and administration of 164. 56-year-old patient admitted to the antibiotics, she is taken for an surgical department with complaints of immediate laparotomy. Perforated abdominal pain, repeated vomiting, which of the sigmoid colon is does not bring relief. The pain starts 2 found. The sigmoid colon is inflamed hours before admission, after consumption but mobile and the mesentery contains of large amout of food. Patient anxious, a perforated abscess. The best pale skin, , pulse 120 bpm, operation for this patient would be: BP 90/60 mmHg. Abdomen moderately A. Insertion of a drain in the abscess; distended in the epigastric region, in the B. Sigmoid resection including the lower parts – sink in. On palpation: abscess and primary colon-to-colon tenderness in the epigastrium. On anastomosis; percussion: tympanic sound in the C. Sigmoid loop colostomy; epigastic region, increased peristalsis. On D. Left hemicolectomy; plain abdominal film dillated small E. Sigmoid resection and end sigmoid intestinal loops. Make diagnosis? colostomy and oversew the rectum A. Peptic ulcer perforation (Hartmann procedure). B. Groin hernia incarceration C. Acute pancreatitis 169. Which statement is wrong concerning D. Small intestine volvulus primary microbial peritonitis? E. Bowel obstruction caused by the tumor A. Occurs without perforation of a hollow of large intestine viscus; B. Occurs with perforation of a hollow 165. Which sign is characterized by rebound viscus; tenderness over the site of abnormality in C. Caused by direct seeding of patients with peritonitis? microorganisms; A. Kocher’s sign; D. Seeding of microorganisms via B. Blumberg's sign; bacterial translocation from the gut; C. Murphy’s sign; E. Seeding of microorganisms via D. Pasternatski’s sign; hematogenous dissemination. E. Cullen’s sign. 170. Which statement is wrong concerning 166. What is the leading etiological factor secondary microbial peritonitis? causing spontaneous bacterial peritonitis? A. Occurs subsequent to perforation of a A. Candida albicans; hollow viscus; B. Streptococcus pyogenes; B. Endogenous microbes spill out into the C. Staphylococcus aureus; peritoneal cavity; D. Escherichia Coli; C. Even after timely surgical intervention E. Pseudomonas aeruginosa. and preemptive antibiotic therapy, about 15-30% of patients demonstrate 86 ongoing infection consisting of domination; recurrent secondary microbial D. Staphylococcus; peritonitis, intraabdominal abscess, or E. Streptococcus tertiary microbial peritonitis. D. Perforation of the colon is associated 176. Choose a reason for the use of with lower infection rates; metronidazole as a component of E. Patients who develop secondary antibacterial therapy of patients with microbial peritonitis should undergo diffuse peritonitis? surgery to alleviate the source of A. Elimination of anaerobic bacteria; ongoing peritoneal soilage. B. Elimination of gram-positive flora; C. Elimination of gram-negative flora; 171. Which examination would be most D. Elimination of fungal infections ; efficient in finding the cause for E. Antiprotozoal antibiotoc secondary bacterial peritonitis? A. Laparoscopy; 177. For the clinical course of acute B. Plain abdominal film; peritonitis three stages are typical. C. Ultrasound; Choose the corect combination: 1. D. CT scan; Subclinical; 2. Reactive; 3. Toxic; 4. E. Level of CRP. Septic; 5. Terminal: A. 1, 2, 3; 172. One of the listed below diseases didn’t B. 2, 3, 4; cause secondary peritonitis C. 1, 2, 5; A. Acute cholecystitis; D. 3, 4, 5; B. Destructive appendicitis; E. 2, 3, 5 C. Acute cholangitis; D. Bowel obstruction; 178. For clinical picture of subdiaphragmatic E. Mesenteric infarction abscess is typical everything except: A. Reduction of pulmonary respiratory 173. Choose the clinical sign, not typical for excursions; acute peritonitis: B. Elevation of diaphragm dome; A. Tachycardia; C. Reactive pleural effusion; B. Positive Blumberg sign; Leukocytosis; C. Muscles rigidity; D. D. Paralytic ileus; E. Hematemesis E. Positive Ortner sign 179. Name the walls of inguinal canal: 1. 174. Select the method of instrumental Inguinal ligament; 2. External oblique examination, which is not suitable for abdominal m. aponeurosis; 3. localization of intra-abdominal abscesses: Transversalis fascia; 4. Round ligament of A. US; the ; 5. M. rectus abdominis; 6. B. Colonoscopy; Spermatic cord; 7. M. transversus C. CT; abdominis , internal oblique abdominal m.. D. MRI; A. 1, 2, 4, 5; E. Laparoscopy B. 1, 2, 3, 7; C. 2, 3, 4, 5; 175. Specify the main microorganisms, which D. 3, 4, 5, 6; are identified in the abdominal cavity of E. 4, 5, 6, 7. patients with purulent peritonitis: A. Monomicrobial; 180. A 60-year-old male presents with an B. Gram-positive microorganisms of recent onset. Which of the domination; following statements are TRUE? C. Gram-negative microorganisms 87 A. The hernia is more likely to be direct A. Incarcerated; than indirect; B. Irreducible; B. Presents through the posterior wall of C. Sliding; the inguinal canal, lateral to the deep D. Richter’s; inguinal ring; E. Interstitial. C. Is located lateral to epigastric vessels; D. Is more likely than a to 185. Which nerves can be injured during strangulate; surgery for repair of inguinal hernia: 1). E. The sac is congenital. Ilioinguinal n., 2). Pudendal n., 3). Iliohypogastric n., 4). Femoral n., 5). 181. A 70-year-old cigarette smoker presents Popliteal n., 6). Genitofemoral n.. with a right inguinal mass that has enlarged A. 1, 3, 6 and has caused discomfort in recent months. B. 2, 3, 5 The swelling, which does not extend to the C. 1, 2, 3 scrotum, reduces when resting. What is the D. 3, 5, 6 likely diagnosis? E. 1, 5, 6 A. Strangulated indirect inguinal hernia; 186. Which of the following structures would B. Direct inguinal hernia be encountered during repair of an inguinal C. Hydrocele hernia in a male? D. Aneurysm of the femoral artery; A. Spermatic cord; E. Cyst of the cord. B. Round ligament; C. Obturator nerve; 182. Name the best tension-free method for D. Symphysis pubis inguinal hernia repair. E. Nerve to the adductor muscles of the A. Bassini repair; thigh. B. Shouldice repair; C. Stoppa repair; 187. In repair of a femoral hernia, the D. Lichtenstein repair; structure most vulnerable to major E. McVay (Cooper’s ligament) repair. injury lies: A. Medially; 183. A 62-year-old male presents with an B. Laterally; irreducible swelling and severe pain in the left C. Anteriorly; groin. He had a known reducible hernia for 15 D. Posteriorly; years prior to this. He had a bowel movement E. Superficially. while in the emergency room. At surgery, a Richter’s hernia was found. Which of the 188. A 28-year-old professional football following statements is TRUE? player has sudden pain and swelling in A. It presents lateral to the rectus sheath; the right groin when attempting to B. It presents through the lumbar intercept a pass. He is admitted to the triangle; local emergency department. On C. It presents through the obturator examination, there is a tender swelling foramen; in the right groin. The scrotum and D. It contains a Meckel’s diverticulum; penis show no abnormality. What is E. It may allow normal passage of stool. the next step in management? A. Needle aspiration to exclude hematoma; 184. At surgery for a right inguinal hernia, a B. Forceful manual reduction; 72-year-old man is found to have a hernia sac C. Laparotomy within 20 minutes; that is not independent of the bowel wall. The D. Preoperative preparation and cecum forms part of the wall of the sac. Such exploration of the groin with hernia a hernia is properly referred to as which of the repair; following? E. Morphine and reevaluation within 12 88 hours B. It is protective against gastric carcinoma. 189. A 45-year-old man complains of burning C. It is associated with chronic gastritis. epigastric pain that wakes him up at night. D. It causes gastric ulcer but not duodenal The pain is relieved by eating or using over- ulcer. the-counter antacids and H2 blockers. E. It cannot be detected by the urea breath Diagnosis is best confirmed by which of the test. following? A. Urea breath test 193. A 68-year-old woman has been B. Serum gastrin levels diagnosed with a benign ulcer on the greater C. Barium meal examination curvature of her stomach, 5 cm proximal to D. Ultrasound the antrum. After 3 months of standard E. Upper endoscopy and biopsy medical therapy, she continues to have guaiac positive stool, anemia, and abdominal pain 190. A 44-year-old patient has refractory with failure of the ulcer to heal. Biopsies of chronic duodenal ulcer disease. His physican the gastric ulcer have not identified a has suggested several surgical options. The malignancy. The next step in management is patient has chosen a parietal (highly selective) which of the following? vagotomy instead of a truncal vagotomy and A. Treatment of the anemia and repeat all antrectomy because? studies in 6 weeks A. It results in a lower incidence of ulcer B. Endoscopy and bipolar electrocautery or recurrence. laser photocoagulation of the gastric B. It benefits patients with antral ulcers the ulcer most. C. Admission of the patient for total C. It includes stomach resection. parenteral nutrition (TPN), treatment D. It left the nerve supply to pylorus intact of anemia, and endoscopic therapy (nerves of Latarjet). D. Surgical intervention, including partial E. It includes removal of the ulcer. gastric resection E. Surgical intervention, including total 191. A frail elderly patient is found to have an gastrectomy anterior perforation of a duodenal ulcer. He has a recent history of nonsteroidal anti- 194. Over the past 6 months, a 60-year-old inflammatory drug (NSAID) use and no woman with long standing duodenal ulcer previous history of peptic ulcer disease. A disease has been complaining of anorexia, large amount of bilious fluid is found in the nausea, weight loss and repeated vomiting. abdomen. What should be the next step? She recognizes undigested food in the A. Lavage of the peritoneal cavity alone vomitus. Examination and workup reveal B. Lavage and omental patch closure of the dehydration, hypokalemia, and ulcer hypochloremic alkalosis. What is the most C. Total gastrectomy likely diagnosis? D. Lavage, vagotomy, and A. Carcinoma of the fundus gastroenterostomy B. Penetrating ulcer E. Laser of the ulcer C. Pyloric obstruction due to cicatricial stenosis of the lumen of the duodenum 192. A 37-year-old man has had recurrent D. ZES (Zollinger Ellison Syndrome) symptoms suggestive of peptic ulcer disease E. Anorexia nervosa for 4 years. Endoscopy reveals an ulcer located on duodenum buld. A mucosal biopsy 195. A 2-cm ulcer on the greater curvature of reveals Helicobacter. pylori . What is TRUE the stomach is diagnosed in a 70-year-old about H. pylori ? woman by a barium study. Gastric analysis to A. Active organisms can be discerned by maximal acid stimulation shows achlorhydria. serology. What is the next step in management? 89 A. Antacids, H2 blockers, and repeat C. Superior mesenteric vein, barium study in 6 to 8 weeks inferior mesenteric vein B. Proton pump inhibitor (e.g., omeprazole D. Splenic vein + inferior C. Prostoglandin E (misoprostol) and mesenteric vein, superior repeat barium study in 6 to 8 weeks mesenteric vein D. Immediate elective surgery E. Splenic vein, superior E. Upper endoscopy with multiple biopsies mesenteric vein, inferior (at least 8 or 9) for the ulcer mesenteric artery

196. A 9-year-old girl had multiple episodes 200. Budd-Chiari syndrome develops due to: of upper GI bleeding. Contrast enhanced CT A. Thrombosis of hemorrhoidal scan showed multiple cavernous veins malformation surrounding the portal vein. She B. Thrombosis of hepatic veins is admitted with severe hematemesis and C. Thrombosis of superior melena. At birth, she had developed an mesenteric vein infection around the umbilicus. What is the D. Thrombosis of inferior most likely site of bleeding? mesenteric vein A. Mallory-Weiss tear of the E. Thrombosis of splenic vein lower end of the esophagus B. Duodenal varices 201. The portal vein drains blood from the C. Peptic ulcer following organs, except: D. Esophageal varices A. liver E. Meckel's diverticulum B. spleen C. pancreas 197. A 43-year-old man with chronic . small intestines and liver cirrhosis is admitted with upper GI E. stomach bleeding. He has marked ascites and shows multiple , liver palmar 202. What is the most common complication erythema, and clubbing. A diagnosis of of portal hypertension: bleeding esophageal varices secondary to A. portal hypertension is made. Portal pressure is B. obstructive jaundice considered elevated when it is above which of C. the following? D. liver cirrhosis A. 0.15 mm Hg E. variceal hemorrhage B. 1.5 mm Hg C. 10 mm Hg 203. Determining the cause of portal D. 40 mm Hg hypertension involves all except (choose one E. 105 mm Hg wrong answer): A. chronic fever 198. For how many segments the liver is B. history of alcohol abuse divided? C. umbilical infection A. 11 D. history of blood transfusions, B. 9 intravenous drug use (. 8 B and C) D. 7 E. history of jaundice E. 5 204. Signs of portosystemic collateral 199. Name the main vessels which forms formation include the following: 1. portal vein Dilated veins in the anterior abdominal A. Azygos veins wall (umbilical epigastric vein shunts) 2. B. Hepatic veins Venous pattern on the flanks (portal- parietal peritoneal shunting) 3. 90 Paraumbilical hernia 4. Rectal C. US investigation with 5. Ascites - Shifting dullness Duplex-Doppler and fluid wave 6. Caput medusa (tortuous D. upper GI endoscopy collaterals around the umbilicus).Choose E. hepatic venous pressure the correct combination of answers: gradient (HVPG) measurement A. 2, 3, 5, 6. B. 1, 2, 3, 5, 6. 209. Patient with 10-year history of hepatitis C. 1, 2, 4, 6. C is admitted to emergency department with D. 2, 4, 5, 6. massive GI bleeding. Within admission there E. 1, 2, 4, 5, 6. were signs of rebleeding, but patient complains on severe weakness and dizziness. 205. US features suggestive of hepatic What complications can develop in this case : cirrhosis with portal hypertension include the 1. , 2. bronchial following: 1. nodular liver surface or nodular aspiration, 3. renal failure, 4. portal vein regenerative hyperplasia. 2. enlarged IVC. 3. thrombosis, 5. acute liver necrosis. splenomegaly. 4. "portal" gastropathy. 5. A. 1, 3, 4. ascitis. 6. portal vein cavernous formation. B. 1, 3, 5. Exclude false answers: C. 1, 2, 3. A. 3, 4, 6. D. 2, 3, 5. B. 2, 4. E. 2, 4, 5. C. 2, 4, 6. D. 1, 2, 4. 210. 62-year old man with long history of E. 1, 3, 5. liver cirrhosis developed massive variceal bleeding. Physician performed upper 206. Name scoring system used to evaluate endoscopy and variceal bleeding was severity of liver cirrhosis: confirmed. First attempt of endoscopic A. APACHE II variceal ligation failed. Balloon-tube B. Glasgow score tamponade was considered to stop the C. Child-Pugh scoring system bleeding. For how long Balloon-tube D. Ranson Criteria tamponade is allowed? E. Alvorado score A. 6-9 h B. up to 24 h 207. To evaluate liver function abnormalities C. only within resuscitation all listed below laboratory test should be D. 1-2 h performed, except: E. 2-3 days A. serologies B. Platelet count 211. Choose wrong statement concerning C. Prothrombin time blood supply of liver: D. Albumin A. The liver receives a dual blood supply E. Liver function tests from both the portal vein and the hepatic artery 208. All patients with cirrhosis should be B. 75% of flow from the portal vein and considered for the presence of varices at the 25% from the hepatic artery time of the initial diagnosis of cirrhosis. C. Venous drainage is via the hepatic Gastroesophageal varices confirm the veins, which drain directly into the diagnosis of portal hypertension. What inferior vena cava examination is the most reliable in revealing Portal flow is increased by food intake the varices: D. A. antegrade cholangiogram E. The liver receives blood supply only B. liver-spleen scan (with from the proper hepatic artery technetium sulfur colloid) 91 212. Under the normal conditions per 24 are used to PREVENT initial bleeding by hours the liver produces bile in the volume: lowering the portal preassure? A. 50-100 ml A. α-blockers B. 150-200 ml (phenoxybenzamine , C. 1500-2000 ml phentolamine ) D. 600-1000 ml B. β-blockers (propranolol, E. 2000-2500 ml nadolol) C. antibiotics (cefazolin, 213. Choose the reasons which cause ceftriaxone) presinusoidal (prehepatic) portal D. Lactulose hypertension: 1. Budd–Chiari syndrome; 2. E. Diuretics (furosemide) Viral hepatitis C; 3. Thrombosis of portal vein or one of it’s major branches; 4. Thrombosis 217. A 49-year-old man with a history of of inferior vena cava 5. Alcohol liver cirrhosis is admitted with significant cirrhosis; 6. Umbilical infection in infants. hematemesis. There is jaundice and clubbing A. 1, 3 of the fingers. His extremities are cold and B. 3, 6 clammy, and the systolic blood pressure drops C. 1, 4 to 84 mm Hg. After BP stabilization, which D. 3, 5 drugs should be used to STOP bleeding? E. 4, 6 A. phenoxybenzamine or phentolamine 214. Choose the reasons which cause B. propranolol or nadolol sinusoidal (hepatic) portal hypertension: 1. C. furosemide Budd–Chiari syndrome; 2. Viral hepatitis C; D. vasopressin or octreotide 3. Thrombosis of portal vein or one of it’s E. prednisolon or major branches; 4. Thrombosis of inferior hydrocortisone vena cava 5. Alcohol liver cirrhosis; 6. Umbilical infection in infants. 218. A 49-year-old man with a history of A. 1, 3 cirrhosis is admitted with significant B. 3, 6 hematemesis. There is jaundice and clubbing C. 1, 4 of the fingers. His extremities are cold and D. 3, 5 clammy, and the systolic blood pressure drops E. 2, 5 to 84 mm Hg. After BP stabilization, which intervention should be performed primarily to 215. Choose the reasons which cause STOP bleeding? postsinusoidal (posthepatic) portal A. Urgent endoscopy and endoscopic hypertension: 1. Budd–Chiari syndrome; 2. bleeding management (band ligation Viral hepatitis C; 3. Thrombosis of portal vein or sclerotherapy) or one of it’s major branches; 4. Thrombosis B. Open surgery to stop the bleeding of inferior vena cava 5. Alcohol liver C. Sengstaken-Blakemore tube cirrhosis; 6. Umbilical infection in infants. Emergency lienorenal shunt A. 1, 5 D. B. 3, 6 E. Vagotomy C. 1, 4 D. 3, 5 219. A 42-year-old woman with a known E. 4, 6 history of esophageal varices secondary to hepatitis and cirrhosis is admitted with 216. The high mortality associated with first severe hematemesis from esophageal variceal bleeding episodes has led to the varices. Bleeding persists after investigation of a variety of methods of vasopressin therapy, after endoscopic prevention of initial bleeding. Which drugs bleeding management (band ligation and sclerotherapy). What should be the next 92 step in management? E. Laboratory tests are performed A. Emergency portocaval shunt to establish the etiology of liver B. Emergency lienorenal shunt cirrhosis C. Splenectomy D. Vagotomy 222. To determine the etiology of liver E. Transjugular intrahepatic cirrhosis laboratory test should be performed. portasystemic shunt (TIPS) Which tests will be positive or increased in case of cholestatic liver cirrhosis? Choose 220. A 25-year-old man from rural district on correct combination: 1. Alcohol screening; 2. US was diagnosed a cyst 60 mm in diameter, Gamma glutamyl transpeptidase; 3. ALT is serological test for antigens specific for greater than AST; 4. AST is greater than Echinococcus granulosus was positive. Which ALT; 5. Viral serology (HBV, HCV); 6. statement is wrong concerning treatment of Alkaline phosphatase. this patient? A. 1, 2 A. Systemic antihelminthic agents are B. 2, 3 generally very effective against C. 2, 4 human Echinococcus D. 2, 5 B. Surgery remains the standard E. 2, 6 approach for hydatid disease of the liver 223. To determine the etiology of liver C. Surgery is indicated in all patients cirrhosis laboratory test should be performed. with symptomatic disease Which tests will be positive or increased in D. Surgical treatment should be case of alcohol liver cirrhosis? Choose correct considered in patients with combination: 1. Alcohol screening; 2. Gamma asymptomatic disease discovered glutamyl transpeptidase; 3. ALT is greater accidentally, when the cyst is large than AST; 4. AST is greater than ALT; 5. (>5 cm) Viral serology (HBV, HCV); 6. Alkaline phosphatase. To be curative, surgical therapy E. A. 1, 2 must remove all the living parasite B. 1, 3 and leave no viable daughter C. 1, 4 or protoscolices D. 1, 5

E. 1, 6 221. Which statement is wrong concerning liver cirrhosis? 224. To determine the etiology of liver A. Cirrhosis is a histologic diagnosis, cirrhosis laboratory test should be performed. based on three essential criteria: Which tests will be positive or increased in diffuse disease, presence of case of viral liver cirrhosis? Choose correct fibrosis, and replacement of combination: 1. Alcohol screening; 2. Gamma normal architecture by abnormal glutamyl transpeptidase; 3. ALT is greater nodules than AST; 4. AST is greater than ALT; 5. B. Cirrhosis is a reversible process Viral serology (HBV, HCV); 6. Alkaline C. Morphologicaly liver cirrhosis can phosphatase. be devided into two groups: A. 1, 5 macronodular (>3 mm) and B. 2, 5 micronodular (<3 mm) C. 3, 5 D. Clinicaly cirrhosis is classified to D. 4, 5 two stages: compensated and E. 5, 6 decompensated 225. *In 50-year-old patient., suffering from Budd-Chiari`s syndrome, appeared progressive pain in right subcostal area, 93 jaundice, varicose veins of the esophagus, B. US rectum, abdominal wall, ascites, C. Radioisotope scanning splenomegaly. Liver cirrhosis was D. Needle biopsy with histological diagnosed. What was the mechanism of examination liver cirrhosis development? E. Biochemical blood test A. Cholestatic hepatitis B. Portal hypertension 230. A patient delivered to the emergency C. department with obstructive jaundice. Choose D. Toxic hepatitis clinical signs which are typical for obstructive E. Viral hepatitis jaundice caused by choledocholisiasis: 1. Upper right quadrant pain; 2. Itch; 3. Back 226. *In patient M. for a long time suffering pain; 4. Vomiting; 5. Discolouration of stool: from liver cirrhosis, recently appeared A. 1, 2, 5 complaints for moderate pain in the B. 2, 3, 5 epigastric area, constant bloating, which C. 1, 4, 5 aggravated after eating. Objective: caput D. 3, 4, 5 medusae on abdominal wall, signs of free E. 1, 2, 3 fluid in the abdominal cavity, enlarged liver and spleen. US: expansion of portal 231. 67-year-old man was made diagnosis cancer of vein, increased liver and spleen. Which pancreas, tumor is located in the head of the gland complication of liver cirrhosis we can and causes obstructive jaundice. Which think about? clinical signs are typical for obstructive A. Portal vein thrombosis jaundice caused by cancer of the pancreas: B. Hepatocellular failure 1. Upper right quadrant pain; 2. Nausea; 3. C. Portal hypertension Palpable enlarged gallblader; 4. Weight D. Peritonitis loss; 5. Discolouration of stool: E. Intestinal dysbacteriosis A. 1, 3, 4 B. 2, 3, 4 227. Choose the symptom of portal C. 1, 2, 3 hypertension: D. 3, 4, 5 A. Tongue papilla atrophy E. 2, 3, 5 B. Jaundice C. Ascites 232. A 58-year-old man with a 20-year D. Erythema of palms history of alcoholism and pancreatitis is E. Increased levels of liver admitted to the hospital with an elevated enzymes in serum bilirubin level to 65 µmol/L, acholic stools, and an amylase level of 120 U*H/L. 228. Select the causes of ascites in patients Obstructive jaundice in case of chronic with liver cirrhosis: 1. Portal hypertension pancreatitis usually results from which of the 2. AST and ALT high levels 3. following? Secondary hyperaldosteronism 4. A. Sclerosing cholangitis Hypoalbuminemia 5. High level of B. Common bile duct compression bilirubin in the blood serum. caused by inflammation A. 1, 3, 4 C. Alcoholic hepatitis B. 1, 2, 3 D. Biliary dyskinesia C. 2, 3, 5 E. Splenic vein thrombosis D. 1, 3 E. All are correct 233. A 62-year-old man is admitted with abdominal pain and weight loss of 5 kg over 229. Which method is the most efficient in the past month. He has continued to consume diagnosing liver cirrhosis? large amounts of rum. Examination reveals A. CT icteric sclera. The indirect bilirubin level is 50 94 µmol/L with a total bilirubin of 65 µmol/L. should be undertaken as the next step in An ultrasound shows a 4-cm fluid collection therapy? near the head of the pancreas. What is the A. Should be discharged home under most likely cause of jaundice in a patient with observation alcoholic pancreatitis? B. Should be observed in the hospital A. Alcoholic hepatitis C. Undergo surgical exploration of the B. Carcinoma of pancreas CBD C. Intrahepatic cyst D. ERCP with sphincterotomy and D. stone removal E. Hemolytic anemia E. Anticoagulants

234. A 66-year-old man with obstructive 238. A 49-year-old African American woman jaundice is found on ERCP to have born in New York is admitted with RUQ pain, periampullary carcinoma. He is otherwise in fever, and jaundice (Charcot’s triad.) A excellent physical shape and there is no diagnosis of ascending cholangitis is made. evidence of metastasis. What is the most With regard to the etiology of ascending appropriate treatment? cholangitis, which of the following is TRUE? A. External radiotherapy A. It usually occurs in the absence of B. Local excision and radiotherapy jaundice C. Radical excision (Whipple B. It usually occurs secondary to CBD procedure) when possible stones D. Internal radiation seeds via catheter C. It occurs frequently after E. Stent and chemotherapy choledochoduodenostomy D. It does not occur in patients with 235. Which instrumental examination is cholangiocarcinoma considered to the most efficient and safe in E. It is mainly caused by the liver finding the cause of obstructive jaundice? insufficiency A. CT B. US 239. A surgeon is removing the gallbladder of C. Plain abdominal film a 35-year-old obese man. One week D. MRCP previously the patient had recovered from E. ERCP obstructive jaundice and at operation, numerous small stones are present in the 236. A 67-year-old woman is evaluated for gallbladder. In addition to cholecystectomy, obstructive jaundice. Which biochemical test, the surgeon should also perform which of the except bilirubin level, helps in diagnosing following? obstructive jaundice? A. No further treatment A. Alkaline phosphatase B. Liver biopsy B. AST C. Intraoperative cholangiogram C. CRP D. Removal of the head of the pancreas D. Protein E. CBD exploration E. Amilase 240. Choose the diseases for which Courvoisier 237. A 40-year-old man underwent sign is typical: 1. Chronic calculous laparoscopic cholecystectomy 2 years earlier. cholecystitis; 2. Pancreatic cancer; 3. He remains asymptomatic until 1 week before Acute pancreatitis; 4. Cancer of papilla admission, when he complains of RUQ pain Vatteri; 5. Liver cirrhosis. and jaundice. He develops a fever and has А. 1, 2, 5; several rigor attacks on the day of admission. В. 3, 5; An ultrasound confirms the presence of С. 2, 5; gallstones in the distal CBD. The patient is D. 2, 3, 5; given antibiotics. Which of the following Е. 2, 4. 95 A. Gallstones 241. Name the indications for Intraoperative B. Cholangitis cholangiography: 1. Detection of stones C. Chronic pancreatitis in CBD during palpation; 2. Suspicion D. CBD malignancy for scar narrow of papilla Vatteri; 3. E. Acute hepatitis Jaundice before surgical intervention; 4. Diameter of CBD more than 10 mm; 5. 247. Extrahepatic biliary tree consists of all Jaundice during surgical intervention. except: А. 1, 2, 3, 4; A. Wirsung’s duct В. 1, 3, 4; B. Common hepatic duct С. 3, 4; C. Cystic duct D. 1, 3, 4, 5; D. Gallbladder Е. All are correct. E. Common bile duct

242. Choose the most serious complications of 248. What is the most common benign causes mechanical jaundice: of obstructive jaundice? А. violation of absorption in small A. Gallstones intestine; B. Strictures В. reduction of protein-synthesizing C. Chronic alcohol abuse function of liver; D. Primary sclerosing cholangitis С. hepatic-renal failure; E. Liver cirrhosis D. gallbladder empyema; Е. gallbladder hydrops. 249. What is NOT the additional source of bilirubin production? 243. Choose the indications for choledochotomy: 1. A. Ineffective erythropoiesis Biliary dyskinesia; 2. Acute cholangitis; B. Myoglobinemia 3. Gallbladder perforation and peritonitis; C. Cytochromes metabolism 4. Obstructive jaundice; 5. Detection of D. Large hematoma lysis stones in CBD during palpation. E. Gallstones dissolving А. 1, 3, 4; В. 2, 3, 4; 250. All diseases can cause hemolysis, С. 2, 3, 4, 5; except: D. 2, 4, 5; A. Autoimmune disorders Е. All are correct. B. Hypersplenism C. Defects in hemoglobin structure 244. Which disease did not cause obstructive D. Sickle cell disease jaundice? E. Iron deficiency anemia A. 1,6 cm stone in gallbladder; B. Cancer of papilla Vatteri; 251. What are the most common causes of C. Residual stone in CBD; hepatitis? 1. Portal hypertention. 2. Alcohol. D. Pancreatic cancer; 3. Autoimmune disorders. 4. Cholelithiasis. 5. E. Choledocholithiasis. Viruses. A. 2,3,5 245. Prehepatic jaundice is caused by: B. 1,2,4 A. Hemolysis C. 3,4,5 B. Gallstones D. 1,3,4 C. Pancreatic cancer E. 1,2,5 D. Renal fealure E. Pancreatic preudocyst 252. Chronic alcohol abuse may result in: 1. Fatty liver (steatosis). 2. 246. Posthepatic jaundice can be caused by all infection. 3. Gallstone formation. 4. Cirrhosis. except: 5. Hepatitis. 96 A. 2,4,5 D. Pancreatic cancer is associated B. 1,4,5 with positive Courvoisier's sign C. 2,3,4 E. Common bile duct is divided into two D. 1,2,3 parts: supraduodenal and E. 1,3,5 retroduodenal

253. Classicaly cholangitis is diagnosed 258. All diseases listed below can lead to clinically as syndrome known as Charcot’s extrahepatic biliary obstruction, triad. EXCEPT: A. Fever, hypotension and jaundice A. Choledocholithiasis B. Fever, RUQ pain and hypotension B. Acute virual hepatitis C. Pruritus, RUQ pain and jaundice C. Ampullary cancer D. Pruritus, hypotention, and jaundice D. Pancreatic cancer E. Fever, RUQ pain, and jaundice E. Biliary strictures

254. Which enzymes are markers of biliary 259. Decompression of extrahepatic biliary hypertension? 1. Alkaline phosphatase. 2. obstruction can be achieved by three gamma glutamyl transpeptidase. 3. methods: 1. Surgical bypass. 2. Enterocinase, 4. Acid phosphatase. 5. AST. Percutaneous insertion of stents. 3. A. 2,4 Endoscopic insertion of stents. 4. TIPS B. 1,5 procedure. 5. Nasogastric decompression. C. 2,3 A. 2,4,5 D. 1,2 B. 1,4,5 E. 3,5 C. 2,3,4 D. 1,2,3 255. Bile is produced by the liver and E. 1,3,5 contains all components, EXCEPT: A. Bile salts. 260. Which statements, concerning jaundice, B. Water and electrolytes. is NOT true? C. Cholesterol. A. High levels of circulating bile salts are D. Bilirubin. associated with pruritus E. Vitamins A, K, D B. Jaundice, dark urine, discoloration of stool and pruritus are the hallmark of 256. ERCP and percutaneous transhepatic obstructive jaundice cholangiography are invasive and can be C. History of fever, and associated with following complications, intermittent jaundice may be EXCEPT: suggestive of cholangitis or A. Cholangitis choledocholithiasis B. Biliary leakage – biliary peritonitis D. Weight loss, abdominal mass, pain C. Pancreatitis radiating to the back and progressively D. Bleeding deepening jaundice may be suggestive E. Obstructive jaundice of pancreatic cancer E. A palpably enlarged gall bladder in a 257. Which statement is NOT true? jaundiced patient is also suggestive of A. Intrahepatic disorders can lead to chronic alcohol abuse unconjugated-conjugated hyperbilirubinemia 261. Complications of obstructive jaundice B. Posthepatic disorders can cause are all of the following, EXCEPT: conjugated hyperbilirubinemia A. Sepsis caused by cholangitis C. Bilirubin is a breakdown product of B. Biliary cirrhosis heme C. Pancreatitis D. Renal and liver failure 97 E. Respiratory failure adjacent hollow viscera; 4. Exocrine insufficiency; 5. Diabetus mellitus. 262. Chronic pancreatitis (CP) is A. 1, 2, 5 characterized by (choose correct clues of CP B. 2, 3, 4 pathogenesis): 1. Parenchymal fibrosis; 2. C. 3, 4, 5 Ductal strictures; 3. Atrophy of acinar and D. 1, 2, 3 islet tissue; 4. Pseudocyst formation; 5. CBD E. 1, 2, 4 compression. A. 1, 2, 3 268. Longitudinal pancreaticojejunostomy is B. All correct also known as: C. 2, 3, 4 A. Puestow procedure D. 3, 4, 5 B. Whiple procedure E. 1, 2, 4 C. Billroth reconstruction D. Bassini’s operation 263. Name main forms of CP: 1. Proliferative; E. Hartman’s procedure 2. Calcific; 3. Degenerative; 4. Obstructive; 5. Inflamatory. 269. The most common complication of CP A. 1, 2, 3 is: B. 2, 3, 4 A. Mechanical jaundice C. 2, 4, 5 B. Compression of duodenum D. 3, 4, 5 C. Pseudocyst E. 1, 4, 5 D. Diabetus mellitus E. Exocrine isufficiency 264. The most often reason causing CP is: A. Alcohol ingestion 270. A 40-year-old woman with severe B. Gallstones chronic pancreatitis is scheduled to C. Pancreatic cancer undergo an operation, because other forms D. Autoimmune disorders of treatment have failed. The ultrasound E. Trauma of the pancreas shows no evidence of pseudocyst formation or cholelithiasis and endoscopic 265. Name main clinical signs of CP: 1. retrograde cholangiopancreatogram Epigastric pain; 2. Vomiting; 3. Poor appetite; (ERCP) demonstrates dilated main 4. Weight loss; 5. Positive Blumberg sign in pancreatic duct (12 mm) with multiple RUQ stricture formation. Which operation is A. 1, 2, 3 suitable to treat this condition? B. 2, 3, 4 A. Gastrojejunostomy C. 3, 4, 5 B. Pancreaticojejunostomy D. 2, 4, 5 C. Cholecystectomy E. 1, 3, 4 D. Splenectomy E. Subtotal pancreatectomy 266. Which instrumental examination is a reference standard diagnostic method for 271. A 45-year-old patient with chronic chronic pancreatitis? pancreatitis is suffering from malnutrition A. US and weight loss secondary to inadequate B. CT pancreatic exocrine secretions. Which is C. MRCP TRUE regarding pancreatic secretions? D. Plain abdominal film A. Pancreas releases fluid poor in E. ERCP enzymes B. Pancreas releases fluid rich in 267. Choose indications for surgical treatment enzymes and bicarbonate of CP: 1. Pseudocyst formation; 2. Disabling C. Cholecystokinin doesn’t have abdominal pain; 3. Disabling obstruction of influence into pancreatic secretion 98 D. All pancreatic enzymes are secreted B. 1, 3 in an active form C. 1, 4 E. The pancreas produces proteolytic D. 1, 5 enzymes only E. 2, 5

272. Which statement is WRONG concerning 276. All problems that compromise blood anatomy of the pancreas? flow listed below can cause acute visceral A. Pancreas is located mesoperitoneally ischemia, except? B. Pancreas consists of head, neck, A. Acute embolic occlusion body and tail B. Acute thrombotic occlusion C. The head of the gland lies nestled in C. Nonocclusive mesenteric ischemia the C-loop of the second part of the D. Splanchnic artery aneurysm duodenum E. Mesenteric veins thrombosis D. The tail of the gland extends obliquely into the hilum of the 277. Which clinical sings are typical for early spleen stages of acute visceral ischemia? E. The neck of the pancreas overlies the A. Severe abdominal pain, vomiting, spine, where it is susceptible to diarrhea, leukocytosis injury in blunt abdominal trauma B. Mild abdominal pain, constipation C. Pulsating abdominal mass 273. The islets of Langerhans are small D. No typical clinical signs islands of endocrine cells within a sea of E. Bloody stool, signs of peritonitis exocrine tissue, they does NOT consists of: 278. Name later manifestations of acute A. A-cells visceral ischemia? B. B-cells A. Severe abdominal pain, vomiting, C. C-cells diarrhea, leukocytosis D. PP-cells B. Mild abdominal pain, constipation E. D-cells C. Pulsating abdominal mass D. No typical clinical signs 274. The pancreas has a major exocrine E. Bloody stool, signs of peritonitis function in the production of digestive enzymes. These include: 279. Which examination is considered to be a A. Amylase, which functions in the gold standard for diagnosis of acute visceral breakdown of starches ischemia? B. Lipase, which functions to hydrolyze A. Ultrasound fatty acids B. CT C. Trypsin and chymotrypsin, which C. Selective mesenteric angiography function to degrade proteins D. Duplex ultrasonography D. Enterokinase, which functions to E. MRI hydrolaze proteins E. Nucleases such as deoxyribonuclease 280. Which examination is considered to be and ribonuclease, which function to the best for screening of acute visceral break down DNA and RNA, ischemia caused by thrombotic ischemia or respectively venous thrombosis? A. Ultrasound 275. Choose TWO main etiological factors B. CT causing chronic pancreatitis: 1. C. Selective mesenteric angiography Alcohol; 2. Autoimmune disorders; 3. D. Duplex ultrasonography Hyperlipidemia; 4. Pancreatic duct E. MRI obstruction; 5. Idiopathic. A. 1, 2 99 281. A 60-year-old man with a history of 286. The intestine is viable in over 90% of atrial fibrillation is found to severe abdominal patients if the duration of mesenteric ischemia pain, vomiting, diarrhea, WBC=23*10 9/l. The symptoms lasts: embolus is most probably originating from A. 12 hours or less which of the following? B. 24 hours or less A. An atherosclerotic plaque C. 36 hours or less B. An abdominal aortic aneurysm D. 48 hours or less C. Heart E. 72 hours or less D. Lungs E. Paradoxical embolus 287. Superior mesenteric artery embolism can be caused by: 1. Aneurysm of the left 282. A 60-year-old man with a history of ventricle after myocardial infarction; 2. atrial fibrillation is found to severe abdominal Atrial fibrillation; 3. Bacterial pain, vomiting, diarrhea, WBC=23*10 9/l. endocarditis; 4. Right ventricle What is the most appropriate surgical hypertrophy; 5. Pulmonary artery treatment for this patient? stenosis. Choose correct combination: A. Embolectomy А. 1, 3, 4; B. Lumbar sympathectomy B. 1, 2, 3; C. Bypass surgery C. 2, 3, 4; D. Intestine resection D. 2, 3, 5; E. Heparinization E. All are correct.

283. Name the most often cause for 288. What can cause infarction of the mesenteric thrombosis? intestine: 1. Superior mesenteric artery A. Blunt abdominal trauma embolism; 2. Superior and inferior mesenteric B. Arteriosclerotic plaque artery embolism; 3. Superior mesenteric C. Mesenteric artery aneurysm artery thrombosis; 4. Superior mesenteric vein D. Embolus thrombosis; 5. Prolonged spasm of small E. Vasospasm intestine arteries. Choose the BEST combination. 284. A 66-year-old woman is admitted for Z. 1, 2, 3 hyperalimentation due to malnutrition AA. 2, 3, 4 consequent to massive small-bowel BB. 3, 4, 5 resection. What is the most likely CC. All diseases can cause intestine condition that leads to the need to infarction perform a massive resection? DD. None of these diseases can A. Autoimmune disease cause intestine infarction B. Mesenteric ischemia C. Mesenteric adenitis 289. In a patient with superior mesenteric D. Cancer artery embolism in stage of bowel infarction E. Pseudomyxoma peritonei (part of small intestine necrotised) should be performed the following operation: 285. Name the form of acute mesenteric ischemia A. Thrombectomy which has the highest mortality rate? B. Isolated embolectomy A. Acute embolic occlusion C. Embolectomy and resection of B. Acute thrombotic occlusion necrotised intestine C. Nonocclusive mesenteric ischemia D. Total colectomy D. Splanchnic artery aneurysm E. Periarterial sympathectomy E. Mesenteric vein thrombosis 290. A 42-years-old patient, who suffers from mitral stenosis and atrial fibrillation, 6 hours ago appeared severe abdominal pain, 100 vomiting, diarrhea. On examination: Septic shock; 4. Hemorrhagic shock; 5. tenderness in mesogastrium, negative Cardiac decompensation. Blumberg sign. CBC: Leukocytes – 21*10 9/l. A. 1, 2, 3 What causes acute mesenteric ischemia in this B. 2, 3, 5 case? C. 2, 3, 4 A. Acute embolic occlusion of D. None of these diseases cause superior mesenteric artery NOMI B. Acute thrombotic occlusion of E. All these diseases cause NOMI superior mesenteric artery C. Nonocclusive mesenteric ischemia 295. In patients with acute mesenteric D. Portal vein thrombosis ischemia due to mesenteric embolism, E. Mesenteric vein thrombosis which of the following statements is correct? 291. Which parts of the GI tract will be A. Most oftenly embolization to ischemic in case of thrombosis of the orifice inferior mesenteric artery is of superior mesenteric artery? observed A. Stomach and duodenum B. Embolus most oftenly origins B. Stomach, duodenum and ileum from right heart C. Small intestine, cecum, colon C. Thrombolytic therapy may be ascendance attempted in patients without D. Colon and rectum signs of bowel infarction or E. All parts of small and large gastrointestinal bleeding intestine D. Arteriography usually reveals the embolus lodged at the 292. Which operations we can perform in case orifice of the superior of embolic occlusion of superior mesenteric mesenteric artery artery (choose the best combination): 1. E. At the time of exploration in Embolectomy; 2. Embolectomy and resection case of superior mesenteric of part of small intestine; 3. Embolectomy artery embolism, ischemia is and left hemicolectomy; 4. Embolectomy and most severe in the left colon right hemicolectomy; 5. Total excision of ileum, jejunum and right hemicolectomy. 296. A 68-year-old man is admitted to the A. 1, 2, 3 coronary care unit with an acute B. 2, 3, 4 myocardial infarction. His postinfarction C. 1, 4, 5 course is marked by congestive heart D. 1, 2, 4 failure and intermittent hypotension. On E. All operations can be the fourth hospital day, he develops performed severe midabdominal pain. On physical examination, blood pressure is 90/60 293. Most often cause of acute mesenteric mm Hg and pulse is 110 beats/min and ischemia is: regular; the abdomen is soft with mild A. Embolisation to the superior generalized tenderness and distention. mesenteric artery Bowel sounds are hypoactive; stool B. Thrombosis of superior mesenteric hematest is positive. The next step in this artery patient’s management should be which C. Nonocclusive mesenteric ischemia of the following? D. Portal vein thrombosis A. Barium enema E. Mesenteric vein thrombosis B. Upper gastrointestinal endoscopy 294. What can cause nonocclusive mesenteric C. Angiography ischemia (NOMI)? 1. Severe mesenteric D. Ultrasonography vasoconstriction; 2. Myocardial infarction; 3. E. Celiotomy 101 D. 2, 4, 5 297. The earliest symptom of CP is E. All can be used abdominal pain, choose WRONG statement concerning the pain in case of 300. In patients with chronic pancreatitis chronic pancreatitis: surgical treatment is indicated in: 1. A. In CP, two pain patterns have been Disabling pain, which interferes with the described: continuous and patient’s ability to work, is refractory to intermittent pancreatic enzyme therapy, requires high B. When pain is intermittent, episodes doses of oral narcotics; 2. Diabetes may be separated by pain-free mellitus; 3. ; 4. ERCP intervals of months or years evidence of a dilated pancreatic duct (> C. Episodes of continuous pain last 6 mm); 5. Obstruction of CBD. from daily to 2- to 3-days per week A. 1, 2, 3 for at least 2 months B. 1, 3, 4 D. Pain appeares in the epigastric C. 1, 4, 5 region at the beginning of disease D. 2, 3, 4 with a subsequent shift to the right E. All are indications for surgical iliac region treatment E. Pancreatic pain is felt in the epigastrium or upper abdomen, 301. In diagnosing CP in its early stages, with penetration to the back or clinicians should recognize that: radiation to the left intercostal A. Pain is present, to some degree, in region all patients B. Pain may be absent in 10% and more 298. With advancing disease, patients develop patients exocrine and endocrine insufficiency and C. Direct pancreatic secretory thus lose the ability to digest protein and assessment remains the gold fat. Which statement is WRONG? standard for diagnosing pancreatic A. The patien manifests diabetes insufficiency and is performed mellitus and malabsorption on early frequently (revesible) stage D. Radiographic imaging confirms B. Protein malabsorption results in pancreatitis in early stages of the creatorrhea disease C. Since malnutrition impairs E. Endoscopic US is gold standard immunity, incidence of infection is instrumental examination for likely to rise among affected making the diagnosis chronic patients pancreatitis D. With the destruction of insulin- producing pancreatic cells, the 302. Name the most often patient may develop diabetes mellitus complication of chronic E. Fat malabsorption results in pancreatitis: A. Left sided pleural effusion 299. Choose the combination of instrumental B. Disseminated intravascular examinations, which can be used in patients with chronic pancreatitis? 1. coagulation Enoscopic US; 2. CT; 3. MRCP; 4. Plain C. Splenic vein thrombosis abdominal film 5. ERCP. A. 1, 2, 5 D. Pancreatic pseudocyst B. 1, 3, 5 E. Pancreatic ascities C. 1, 4, 5 102 C. All layers of the bowel wall were 303. All statements concerning Chron’s involved disease are true, except? D. Skip lesions were noted A. Affects all parts of GI (from the E. The preoperative GI series showed a oropharynx to the anus) narrowing string like stricture in the B. Nonspecific inflamation ileum (string sign) C. Superficial (mucosal) inflamation D. Ileocolic region affected most 308. A 35-year-old man has known ulcerative oftenly . Which of the following is an E. Etiology is not known indication for total proctocolectomy? A. Occasional bouts of colic and 304. All statements concerning Ulcerative diarrhea colitis (UC) are true, except? B. Sclerosing cholangitis A. Affects colon and rectum C. Toxic B. Transmural inflamation D. Arthritides C. 20-30% of patients with UC have E. Iron deficiency anemia another family member with the disease D. Etiology is not known 309. A 54-year-old man with diarrhea is E. Smokers are in the risk group for UC found to have . Colectomy should be advised in patients with 305. The typical complications of Chron’s ulcerative colitis who have symptoms that disease are all, except: persist for more than which of the A. Abdominal abscesses following? B. Internal fistula A. 1 month C. Intestinal obstruction caused by B. 6 months strictures C. 1 year D. Sclerosing cholangitis D. 10–20 years E. Hemorrhage E. More than 25 years

306. A 26-year-old man is present with mild 310. A 48-year-old woman develops colon clinical signs of Chron’s colitis. What primary cancer. She is known to have a long history treatment this patient should be of ulcerative colitis. In ulcerative colitis, recommended: which of the following is a characteristic of A. Right hemicolectomy colon cancer? B. Left hemicolectomy A. Occurs more frequently than in the C. Treatment of anemia rest of the population. D. Total coloproctectomy B. Is more likely to occur when the E. Medical treatment with ulcerative disease is confined to the aminosalicylates left colon. C. Occurs equally in the right and left 307. A 43-year-old woman undergoes side. investigation for colitis. In her history, it is D. Has a synchronous carcinoma in 4– noted that 20 years earlier she underwent a 5% of cases. surgical procedure on the large intestine. Is E. Has an excellent prognosis because the diagnosis more likely to be ulcerative of physician awareness. colitis rather than Crohn’s disease because at the previous operation? 311. A 40-year-old man with a long history of A. The serosa appeared normal on bloody diarrhea presents with increased inspection, but the colon mucosa was abdominal pain, vomiting, and fever. On extensively involved examination, he is found to be dehydrated B. There was evidence of fistula and shows tachycardia and hypotension. formation The abdomen is markedly tender with 103 guarding and rigidity. What is the most 316. What imaging study is necessary to likely cause? make diagnosis uncomplicated perirectal A. in ulcerative colitis abscess fistula disease? B. Small-bowel perforation from A. Sinogram regional enteritis B. Transrectal US C. Perforated carcinoma of the sigmoid C. CT colon D. No imagine study D. Volvulus of the sigmoid colon E. MRI E. Acute perforated diverticulitis 317. The most effective treatment, which is 312. A 25-year-old man has recurrent, successful in healing 90% of anal fisures, indolent fistula in ano. He also complains includes: 1. Stul softeners; 2. Lexatives; 3. of weight loss, recurrent attacks of diarrhea Antibiotics; 4. NSAIDs; 5. Sitz bath. with blood mixed in the stool, and A. 1, 2, 3 tenesmus. Proctoscopy revealed a healthy, B. 1, 2, 4 normal-appearing rectum. What is the most C. 2, 3, 4 likely diagnosis? D. 3, 4, 5 A. Colitis associated with acquired E. 1, 2, 5. immunodeficiency syndrome (AIDS) B. Ulcerative colitis 318. Uncomplicated interanal hemorrhoids C. Amoebic colitis typicaly are acossiated with: D. A. Anorectal pain E. Crohn’s colitis B. Pain after defecation C. Thrombosis 313. Perirectal abscess fistulous disease is D. Perirectal abscesses most oftenly: E. Bright-red bleeding per rectum A. Associated with a specific systemic disease 319. Interanal and external hemorrhoids can B. Associated with specific infection develop all complications except: disease A. Incarceration C. Cryptoglandular in origin B. Necrosis D. Associated with hemorrhoids C. Perianal condylomas E. Etiology is not known D. Thrombosis E. Bleeding 314. According to their location perirectal abscesses are classified to (one answer is 320. Protoscopy reveals nonbleeding grade I not correct): hemorrhoids. Indications for surgical A. Superficial treatment are all except: B. Supralevator A. III-IV grade hemorrhoids C. Ischiorectal B. Severe bleeding D. Intersphincteric C. Thrombosis E. Perianal D. I-II grade hemorrhoids E. Necrosis 315. The typical complications of perirectal abcess are all, except: 321. Why during hemorrhoidal bleeding the A. Internal fistula blood is bright-red? B. Hemorrhoids A. Hemorrhoidal vein have lots of C. Sphincter injury shunts with rectal arteries D. Perineal sepsis B. Hemorrhoids never bleed E. Chronic fistula C. Hemorrhoids develops from arteries D. Bleeding is associsted with coagulopathy 104 E. Most oftenly upper parts of colon are D. Streptococcus pyogenes bleeding, which are richy vascularized E. Escherichia coli

328. Which of the following is clearly 322. A 44-year-old man has recurrent demarcated? hemorrhoids. What treatment modality is A. Furuncle not indicated in case of recurency? B. Carbuncle A. Conventional surgery C. Cellulitis B. Minimaly invasive treatment D. Necrotizing fasciitis C. Increasing dietary fiber E. Erysipelas D. decreasing constipating , 329. 6-years-old boy was brought to the out- E. decreasing time spent on the toilet patient surgical department by parents with complaints for a rash on the face. On 323. List the layers of skin from the most examination: thin-walled vesicles and superficial to the deepest layer: 1. dermis, yellow crusts on the chin, enlarged 2. epidermis, 3. deep fascia, 4. regional lymph nodes, temperature – subcutaneous tissue, 5. superficial fascia. 37,8C.̊ What is the most probable A. 1 2 3 4 5 diagnosis? B. 2 3 4 5 1 A. Furuncle C. 2 1 5 4 3 B. Carbuncle D. 4 5 3 1 2 C. Impetigo E. 1 2 3 5 4 D. Necrotizing fasciitis E. Erysipelas 324. Which is the most commonly identified infectious agent causing skin and soft tissue 330. Which skin and soft tissue infection have infection? highest mortality rate? A. Staphylococcus aureus A. Furuncle B. Streptococcus pyogenes B. Carbuncle C. Pseudomonas aeruginosa C. Cellulitis D. Clostridium perfringens D. Necrotizing fasciitis E. Escherichia coli E. Erysipelas

325. Erysipelas most oftenly is caused by: 331. Choose WRONG thesis concerning A. Staphylococcus aureus necrotizing fasciitis. B. Streptococcus pyogenes A. Necrotizing fasciitis is more C. Pseudomonas aeruginosa commonly polymicrobial disease D. Clostridium perfringens B. The most commonly affected sites are E. Escherichia coli the extremities C. Necrotizing infections typically 326. A 23 year old patient fell ill 3 weeks ago progress more rapidly (within 24–48 when she noticed a very painful hr) than more superficial cellulitic induration in her axillary area, 4-5 days processes later it burst and discharged a lot of pus. D. Signs and symptoms are diffuse After this some new infiltrations appeared swelling of the affected area without around the affected area. The patient has well-demarcated borders and pain out never suffered from skin diseases before. of proportion to physical findings What is the most probable diagnosis? E. CT and MRI may show air in the A. Hydradenitis tissues or enhancement with B. Streptococcal impetigo intravenous contrast and these signs C. Furuncle are specific to necrotizing fasciitis D. Mycosis E. Carbuncle 332. Choose CORRECT thesis for the treatment of skin and soft tissue infection 327. Which of the following is a β-haemolytic (SSTI). group of bacteria that commonly cause A. Most SSTIs can be managed on an purulent skin and soft tissue infection? inpatient basis A. Staphilococcus aureus B. Superficial infections typically do not B. Pasteurella multocida require systemic treatment and usually C. Haemophilus influenzae respond to topical agents 105 C. SSTI caused by healthcare-associated C. Congenital disoder; MRSA have better prognosis than D. Proteolysis; community-associates MRSA E. Operation. D. In the case of furuncles and carbuncles, incision and drainage of 335. The diagnosis of pseudomembranous colitis is aided by: abscesses is required A. Positive blood culture for Clostridium E. Impetigo is typicaly managed with difficile topical agents B. Raised antibody levels in blood to Clostridium difficile toxin 333. A 63-year-old male patient in reasonably C. High C-reactive protein level good health suddenly suffered from fever D. Detection of Clostridium difficile toxin (>38°C) and a painful right iliac fossa in the stool tumefaction; no other gastrointestinal E. Isolation of Clostridium perfringens problems were noted. On clinical from the stool examination, a mass of about 5 cm was palpable in correspondence of right iliac 336. The following is cause of fossa and appeared hard, not reducible and pseudomembranous colitis in man: fixed to the parietal muscle. Laboratory A. Clostridium histolyticum data showed leucocytosis, shift to the left B. Clostridium septicum and elevated erythrocyte sedimentation C. Clostridium sporogenes rate (74 mm/hr) as pathological findings. D. Clostridium perfringens Abdominal ultrasound examination E. Clostridium difficile evidenced a fluid in the right lower quadrant, with heterogenic echotexture 337. 30-year-old man hospitalized with a and a thickening of the ileocecal tract. severe epigastric pain. During Abdominal CT, confirmed the presence of examination: hypoxemia, dehydration. a complex, predominantly cystic, mass of Laboratory tests: increased levels of large size (6×8 cm) with heterogeneous, amylase and lipase in the blood. CT mainly peripheral enhancement, the confirmed severe acute pancreatitis. adjacent cecum had its wall thickened and Which antibiotic will reduce the risk of it was not possible to differentiate the infection? appendix separately from the mass, A. Ampicillin homolateral inguinal reactive B. Erythromicin lymphadenopathy was also present. The C. Gentamicin patient failed to respond to the initial D. Ceftriaxon conservative management, which E. Imipenem/cilastatin consisted of intravenous fluids and triple antibiotic therapy with cefotaxime, 338. Which statement concerning medical gentamicin and metronidazole, without treatment of patient with acute any improvement of pain and fever. At a pancreatitis on early phase (first 3-5 days) further ultrasound examination, the mass is WRONG? appeared not modified. Make the A. Patients are kept “nothing per os” diagnosis? B. Aggressive fluid resuscitation is A. Appendicular infiltration; critically important B. Appendicular abscess; C. Antibiotics should be used in any case C. Diffuse peritonitis; of acute pancreatitis D. Appendicular colic; D. Imipenem/cilastatin is indicated to E. Phlegmonous appendicitis; prevent infication in patients with pancreatic necrosis 334. A 20-year-old man has undergone E. Early initiation of enteral nutrition is appendectomy for perforated appendicitis important in treatment and infection with generalized peritonitis. Seven days prevention postoperatively, his temperature continues to spike to 39,5°C despite 339. Which of the following is not a В-lactam antibiotic therapy with ampicillin, antibiotic? gentamicin, and metronidazole. A. Meropenem Abdominal CT scan reveals a large pelvic B. Ceftriaxone abscess. Soon afterward, he has bleeding C. Benzylpenicillin from the mouth and nose with increasing D. Gentamicin oozing from the surgical wound and all E. Monobactam intravenous puncture sites. What was a trigger for coagulopathy? 340. What is the commonest cause of A. Sepsis; Antibiotic-associated diarrhea (choose best B. Antibiotic therapy; answer)? 106 A. Malabsortion Evisceration; 4). Grade IV B. Allergic reactions C. Reversible renal impairment on liver injury; 5). Grade V spleen accumulation injury. Choose the best D. Clostridium difficile E. Bone marrow depression combination. a. 1, 2, 3 341. What is the commonest side-effect of B- lactams? B. 1, 3, 4 A. Malabsortion C. 1, 4, 5 B. Allergic reactions C. Reversible renal impairment on D. 2, 3, 5 accumulation E. All conditions are indications D. Clostridium difficile E. Bone marrow depression for laparotomy

342. What therapy is based on predicted susceptibility of likely pathogens and local 346. Which diagnostic procedure antimicrobial policy? should be performed at first in a A. Targeted Therapy hemodynamically stable patient B. Etiological therapy C. Suceptibility-guided therapy with penetrating abdominal D. Empiric therapy trauma? E. Symptomatic therapy A. Ultrasound (Focused

343. Which diagnostic procedure abdominal sonography for should be performed at first in a trauma)

hemodynamically unstable patient B. СТ

with blunt abdominal trauma? C. Laparoscopy

A. Ultrasound (Focused abdominal D. Local wound exploration sonography for trauma) E. Diagnostic peritoneal lavage B. Triple contrast CT C. Diagnostic peritoneal lavage 347. Which diagnostic procedure D. Laparoscopy should be performed secondly in E. Plain abdominal film a hemodynamically stable patient with penetrating abdominal 344. Which diagnostic procedure trauma?

should be performed at secondly in A. Ultrasound (Focused a hemodynamically unstable patient abdominal sonography for with blunt abdominal trauma? trauma)

A. Ultrasound (Focused abdominal B. СТ

sonography for trauma) C. Laparoscopy

B. Triple contrast CT D. Local wound exploration

C. Diagnostic peritoneal lavage E. Diagnostic peritoneal lavage D. Laparoscopy E. Plain abdominal film 348. Which organs are injured most oftenly in patients with blunt 345. The indications for laparotomy in abdominal trauma: 1). Stomach; patient with penetrating 2). Duodenum; 3). Liver; 4). abdominal trauma are: 1). Spleen; 5). Small intestine; 6). Hemodynamic instability; 2). Large intestine. Positive peritoneal signs; 3). A. 1, 2 107 B. 2, 5 350. Laparoscopy in abdominal C. 3, 4 trauma may be indicated in which of D. 5, 6 the following? E. 1, 6 A. To exclude diaphragmatic injury 349. A 7-year-old boy was involved in B. In patients with multiple a motorcycle crash while seated in previous abdominal the back of a minivan without operations restraints. His vital signs in the C. If there is limited emergency room are stable but he is cardiovascular reserve complaining of left upper quadrant D. If severe diffuse peritonitis abdominal pain. The FAST scan exists shows scanty fluid around in the left E. In hemodynamically colic gutter. An abdominal and unstable patients pelvic CT scan with iv and po contrast is performed and the 360. A 30-year-old restrained driver radiologist suggests a “blush” was involved in a motor-vehicle (arterial extravasation) in the splenic crash. He is hemodynamically parenchyma. The spleen itself stable and has a large seat belt sign sustained a deep parenchymal tear on the abdomen. His abdomen is and is classified as a grade III tender to palpation. In this patient injury. The child remains one should be most concerned hemodynamically stable. What is about: recommended next? A. Liver and spleen injury A. Continuous hemodynamic B. Transection of the head of the monitoring, celiac pancreas angiogram, and angio C. Renal pedicle avulsion embolisation of splenic D. Hollow-viscus injuries artery. E. Pelvic fracture B. Immediate exploration in the operation room 361. A 20-year-old unrestrained driver C. If hemodynamic instability was involved in a motor-vehicle develops, aggressive fluid crash. A computed tomography resuscitation including a (CT) of the abdomen revealed a repeated bolus of 20 large hematoma in the second mL/kg lactated Ringer’s portion of duodenum. The rest of solution followed by a the abdomen is normal. The initial liver spleen scan management of this duodenal D. Monitoring only hematoma should be: E. Pneumovax and elective A. Operative evacuation splenectomy in 6 weeks B. Nasogastric decompression, intravenous fluids, and 108 gradual resumption of oral pulse is 120 bpm, and respiration diet rate is 28 breaths per minute. Her C. Endoscopic retrograde abdomen has a stab wound in the cholangiopancreatogram anterior axillary line at the right (ERCP) costal margin. Two large-bore D. Laparotomy, pyloric intravenous lines, a nasogastric exclusion, and tube, and a Foley catheter are gastrojejunostomy inserted. The blood pressure rises to E. Octreotide 85 mm Hg after 2 L of Ringer’s lactate. The appropriate 362. In a patient who had a motor-cycle management is which of the crash, a CT of the abdomen following? revealed a peripancreatic hematoma A. Peritoneal lavage and indistinct pancreatic border. B. Ultrasound of the abdomen The most definitive test for a C. Laparoscopic assessment pancreatic injury requiring operative of the peritoneal cavity intervention is: D. Exploratory laparotomy A. ERCP E. CT of the head B. Ultrasonography C. CT scanning 365. A 22-year-old woman presents to D. Operative exploration the emergency department with a E. Amylase test of lavage fluid chief complaint of severe left upper quadrant (LUQ) pain after being 363. A 25-year-old man fell down from punched by her husband. Her blood his bicycle and hit a concrete wall pressure is 110/70 mm Hg, pulse is on his left side. An ultrasound 100 bpm, and respiration rate is 24 examination showed free fluid in breaths per minute. The best means the abdomen. A CT scan confirmed to establish a diagnosis is which of a grade III splenic injury. The most the following? important contraindication for a A. FAST nonoperative management of the B. Physical examination splenic injury is: C. CT of the abdomen A. Hemodynamic instability D. Peritoneal lavage B. Active bleeding on CT scan E. Upper gastrointestinal (GI) C. Adult patient series D. Lack of availability of blood for transfusion 366. A 60-year-old man is attacked E. Extensive associated injuries with a baseball bat and sustains multiple blows to the abdomen. He 364. A 17-year-old girl presents to the presents to the emergency emergency department with a stab department in shock and is brought wound to the abdomen and a blow to the operating room (OR), where a to the head that left her groggy. Her laparotomy reveals massive blood pressure is 80/0 mm Hg, and a stellate 109 fracture of the right and left lobes of C. Postoperatively, if the the liver. Which of the following patient develops fever techniques should be used D. Postoperatively, based on immediately? culture and sensitivity of A. Pringle’s maneuver fecal contamination found B. Packing the liver at the time of surgery C. Suture ligation E. Intraoperatively, if any D. Ligation of the right hepatic hollow viscus is found to artery be injured E. Ligation of the proper hepatic artery 369. A 70-year-old woman is hit by a car and injures her midabdomen. 367. A 23-year-old man is shot with a The best way to rule out a rupture of handgun and found to have a the second part of the duodenum is through-and-through injury to the by which mode? right transverse colon. There is little A. Repeated physical fecal contamination and no bowel examinations devascularization. At operation, B. Ultrasound what does he require? C. Repeated amylase levels A. Right hemicolectomy with D. CT with oral and ileotransverse colon intravenous contrast anastomosis E. Peritoneal lavage B. Right hemicolectomy with ileostomy and mucous fistula 370. A 15-year-old girl had an injury to C. Debridement and closure of the right retroperitoneum with wounds with exteriorization duodenal contusion. What is the of colon test required to exclude a rupture D. Debridement and closure of of the duodenum? wounds A. Serum amylase E. Segmental resection with B. Dimethyliminodiacetic acid primary anastomosis (HIDA) scan C. Gastrografin study 368. A 20-year-old woman presents to D. Upper GI with barium the emergency department with a E. ERCP stab wound to the abdomen. There is minimal abdominal tenderness. 371. A 33-year-old man presents to the Local wound exploration indicates emergency department with a that the knife penetrated the gunshot injury to the abdomen. At peritoneum. What is the ideal use of laparotomy, a deep laceration is antibiotic administration? found in the pancreas just to the A. Preoperatively left of the vertebral column with B. Intraoperatively, if a colon severance of the pancreatic duct. injury is found What is the next step in management? 110 A. Intraoperative cholangiogram B. Debridement and drainage of defect C. Distal pancreatectomy D. Closure of abdomen with J-P drains E. Vagotomy